Вы находитесь на странице: 1из 214

1.

An 16 year-old girl diagnosed with Turner syndrome in utero and as an infant presents at your clinical
reporting spontaneous menstrual activity. Hormonal workup demonstrates normal ovulatory cycles and a
biopsy of an ovaries demonstrates primary follicles. On physical examination, you note a short webbed
neck, numerous nevi on the skin, and modest lymphedema of the hands and feet. While these are classic
signs of Turner syndrome, the girl is in the 15th percentile for height, presumably due to growth hormone
therapy that was initiated at age 10. Pubic hair on the labia majora is adult-like, though not extending to
the thighs, the areola and papilla project distinctly from the surrounding breast tissue, and you note that
she is Tanner stage IV of sexual development. Although estrogen replacement therapy was initiated at
age 12, the girl's development was noted to be in Tanner stage 1, the infantile stage, just a year prior to
this visit.

The most likely explanation for these developments includes which of the following:

the diagnosis during infancy was in error

an estrogen-secreting ovarian tumour

45,XO/46, XY mosaicism

45,XO/46, XX mosaicism

successful estrogen replacement therapy

Explanation

The correct answer is choice D.

Classically, Turner syndrome results from the absence of one of the X chromosomes in a female, whose
karyotype thus is 45, XO. It also can result from the absence of several genes of the short arm of one of
the X chromosomes, in which case the karyotype is 46, XXiq. While the 45, XO karyotype can be present in
all of the cells of a girl with Turner syndrome, many embryos with this karyotype actually are aborted
spontaneously. Indeed, it has been estimated that up to 15 percent of spontaneous abortions may be of
Turner syndrome embryos. For this reason many girls with Turner syndrome have the condition in mosaic
form.

Mosaicism is a condition in which some of the patient's cells have a certain genetic makeup, while others
have a different one. The karyotype 45, XO/46 XX means that some portion of a girl's cells have a
completely normal chromosome pattern. In contrast 45, XO/46, XY (choice C) would indicate a mosaic
karyotype in which some cells of have a male pattern. This is known as intersex syndrome and most
certainly it would lead to normal female development beginning spontaneously. Most likely, cytogenetic
analysis of tissue biopsied from this girl's ovaries would show some proportion of cells with the normal 46,
XX pattern and this is the reason why sexual maturation has begun.

Girls and women affected by Turner syndrome tend to be of short stature and also exhibit physical signs,
some of which result from a deficiency or lack of ovarian hormones, others which may be related in a more
complex way to certain X genes being present only in one copy.

Lymphedema, especially of the hands and feet, often is noticeable in infancy, or even in utero during
ultrasound testing. Lymphedema occurring in utero is also what causes the notorious webbed neck of
Turner syndrome as well as a low hairline that is common to the condition as well. A high number of nevi
also is typical of Turner syndrome. Given these signs, misdiagnosis at birth (choice A) is rather farfetched.

The Tanner scale is used to assess development in children and adolescents. It consists of five stages
which define the development pubic hair in male and females, the genitals in males, and breasts in
females. While an estrogen producing ovarian tumour (choice B) could indeed stimulated breast growth,
the fact that the breasts are developing appropriately and in the same Tanner stage as the public hair
development suggest that normal physiology is responsible, not a tumour. The fact that she has reached

1
the penultimate stage, suggest that she will reach the last stage and be fully-developed as a woman.

When Turner syndrome is recognized either at birth or early in childhood, growth hormone therapy is
initiated prior to puberty to prevent extremely short stature, as it has been proven effective. This is why
this girl has reached the 15th height percentile; without it, girls with Turner syndrome tend to be below the
5th percentile for height.

Estrogen replacement therapy (choice E), beginning in ages 12-15, reduces the risk of developing
osteoporosis as well as heart disease. However, it does not help much with sexual development. Thus
women with a pure (not mosaic) 45, XO karyotype are infertile.

Despite the apparently normal ovarian function due to mosaicism, having Turner syndrome, this girl still
needs to be monitored for associated physical conditions. One such condition is hypertension. This can
result from coarctation of the aorta or from kidney anomalies, or it may be isolated. Although the girl in
this case does not seem to have signs of hypertension, her blood pressure should be monitored as
hypertension may develop at any time. In addition to aortic coarctation, other cardiovascular anomalies
may be present. Thus, on physical examination, murmurs may be noted. With or without cardiovascular
signs, all patients with Turner syndrome should be referred for consultation with a cardiologist.

Middle ear issues are common as well and this can lead to deficits in hearing or balance. In the
gastrointestinal tract, ulcerative colitis and Crohn's disease both have elevated incidence in those afflicted
with Turner syndrome.

2.

A 22-year-old primigravida woman arrives at the hospital at term at the onset of labor. Despite an
uneventful pregnancy, on taking a history you learn that she has experienced unusually long menstrual
periods, some lasting 9-10 days. You order a complete blood count (CBC) which reveals a red blood cell
count and hematocrit that are normal for pregnancy, but you find that her bleeding time is prolonged at
16 minutes (normal bleeding time is up to 9.5 minutes). Tests for von Willebrand antigen and activity
reveal that she has a mild form of von Willebrand disease.

Management of this case may include all of the following except:

treatment with desmopressin

transfusion of cryoprecipitate

transfusion of whole blood

Cesarean section

careful monitoring for postpartum hemorrhage

Explanation

The correct answer is choice C.

A history of a prolonged menstrual period, or menorrhagia, reported for this woman together with the
prolonged bleeding time suggests that she may have von Willebrand disease. Affecting an estimated one
percent of the population, von Willebrand disease is the most common hereditary inherited bleeding
disorder. It results from a deficiency or abnormality of von Willebrand factor (vWF), a glycoprotein
secreted by capillary endothelial cells that acts as a carrier for clotting factor VIII (FVIII). This protects
FVIII from degradation; thus, if vWF is abnormal, the amount of FVIII is low and the patient's coagulation
ability is reduced. Usually symptoms are fairly mild; in many cases menorrhagia may be the only symptom

2
a woman experiences.

Assays for vWF antigen test for the presence and concentration of vWF in the blood. vWF is the antigen
that is detected by antibodies using various methods. While a low or absent vWF indicates von Willebrand
disease (specifically vWD type 1, a quantitative deficiency) a normal vWF antigen test cannot be used to
rule out the disease. This is because abnormally functioning vWF may bind the antibodies used in the assay
just as well normal vWF binds them. For this reason, a von Willebrand factor activity test is performed.
Also known as a ristocetin cofactor test, this assay indicates how well a patient's vWF actually functions. If
a patient has a normal concentration vWF antigen, but an abnormal vWF activity test, this indicates von
Willebrand disease type 2, a functional deficit.

While levels of FVIII and vWF often increase during the second and third trimesters, causing symptoms of
von Willebrand disease to subside, the fact that this patient has a prolonged bleeding time puts her at risk
of the condition is not managed.

Generally, people with mild von Willebrand disease are treated with desmopressin (choice A), derivative of
the hormone arginine vasopressin, which stimulates capillary endothelial cells to secrete of vWF. Since the
effects of this drug are swift, it is an appropriate treatment for a patient who is about to give birth.

Transfusion of cryoprecipitate (choice B) also would be useful, since such preparations contain vWF, along
with FVIII. However, transfusion of whole blood (choice C) is not necessary, since whole blood contains
many products, especially red blood cells (RBCs) that the patient does not need, as she is not anemic.

Since risk of fetal hemorrhage is elevated in cases in which the mother is affected by von Willebrand
disease, trauma to the fetus, as often occurs during vaginal delivery, should be minimized. Thus, delivery
by Cesarean section (choice D) is recommended. Following the delivery, the mother should be monitored
carefully for possible hemorrhage (choice E) as the risk for this also has been shown to be elevated in
women with von Willebrand disease, regardless of whether or not their symptoms subside during
pregnancy.

3.

A 42-year-old woman presents as a new patient to your office. Her family history is notable for ovarian
cancer in a paternal grandmother and breast cancer in a paternal uncle. Her father died in a motor vehicle
accident at age 36; he had no known illnesses at the time. During the counseling portion of the visit, you
communicate your concerns about the potential for harboring a hereditary predisposition to cancer.

Your concern for a hereditary predisposition to cancer in this patient is particularly related to which of the
following genes?

BRCA1

BRCA2

MLH1

MSH2

PMS1

Explanation

3
The correct answer is Choice B.

BRCA1 and BRCA2 are human genes that belong to a class of genes known as tumor suppressors.
Mutations in these genes have been linked to the development of hereditary breast and ovarian cancer.

A woman’s lifetime risk of developing breast and/or ovarian cancer is greatly increased if she inherits a
mutation in BRCA1 or BRCA2. Men with harmful BRCA1 mutations also have an increased risk of breast
cancer and, possibly, of pancreatic cancer, testicular cancer, and early-onset prostate cancer. However,
male breast cancer, pancreatic cancer, and prostate cancer appear to be more strongly associated with
BRCA2 gene mutations than BRCA1 mutations. For this question, the fact that her paternal uncle had
breast cancer is likely related to the BRCA2 (choice B) gene and not the BRCA1 gene for the reasons listed
above.

The likelihood that a breast and/or ovarian cancer is associated with a harmful mutation in BRCA1 or
BRCA2 is highest in families with a history of multiple cases of breast cancer, cases of both breast and
ovarian cancer, one or more family members with two primary cancers (original tumors that develop at
different sites in the body), or an Ashkenazi (Eastern European) Jewish background.

It is important to note that not every woman who has a BRCA1 or BRCA2 mutation will develop breast
and/or ovarian cancer. A woman who has inherited a harmful mutation in BRCA1 or BRCA2 is about five
times more likely to develop breast cancer than a woman who does not have such a mutation. Risk
estimates for ovarian cancer among women in the general population indicate that 1.4 percent (14 out of
1,000) will be diagnosed with ovarian cancer compared with 15 to 40 percent of women (150–400 out of
1,000) who have a harmful BRCA1 or BRCA2 mutation.

The MLH1 and MSH2 genes are members of a set of genes known as the mismatch repair (MMR) genes.
Mutations in these genes predispose an individual to the development of colorectal cancers as well as
several other tumor types. PMS1 is also a mismatch repair gene that is sometimes found to be mutated in
colorectal and other cancers.

4.

A 30 year old monogamous female has experienced episodic lower abdominal pain and dyspareunia for
the past year. A bimanual pelvic examination reveals no abnormal findings, and she denies having had
sexually-transmitted diseases. A Pap smear is normal. A urinalysis is unremarkable and a urine pregnancy
test is negative. Pelvic ultrasound reveals no masses. Her stool is negative for occult blood. Her
hemoglobin is 14.7 g/dL with hematocrit 44.1% and MCV 94 fL.

Which of the following conditions is most likely to be present:

Pelvic inflammatory disease

Endometriosis

Malakoplakia

Diverticulitis

Adenomyosis

Explanation

4
The correct answer is Choice B.

Pelvic pain is the most common presenting symptom of endometriosis. In addition to pelvic pain, other
common symptoms include dyspareunia (as evident in this case), dysmenorrhea, infertility or subfertility,
chronic fatigue, and bladder or bowel symptoms that correlate with the menstrual cycle.

Physical findings are variable in women with endometriosis, and the physical examination may not reveal
any specific findings, as in this case. In other cases, palpable and/or tender adnexal masses, palpable
pelvic nodules, uterine tenderness, a fixed uterus, cul-de-sac tenderness, or other findings may be
observed, depending upon the extent and localization of the endometriosis implants.

Adenomyosis and diverticular disease are both included in the differential diagnosis of endometriosis and
may produce pelvic pain. In this case the negative stool occult blood test suggests that diverticulitis is less
likely, and although pelvic pain can occur with adenomyosis, heavy menstrual bleeding and painful
menstruation are more common symptoms of adenomyosis.

In this case there is no evidence or history of a sexually-trsnamitted disease, and in a monogamous


woman without evidence of pelvic infection, pelvic inflammatory disease is a less likely cause of her pain.

Malakoplakia is a rare inflammatory condition that often affects the urogenital tracts but is not a common
cause of pelvic pain and dyspareunia.

5.

An infant is born to a 30 year-old mother at 36 weeks of gestation. One minute Apgar score is 4, five
minute Apgar score is 6, but by ten minutes has increased to 8. The infant, a female, is 2,400 grams
(5.25 pounds) at birth and is admitted to the neonatal intensive care unit in respiratory distress. The
umbilical cord is found to contain only one umbilical artery, instead of two. Examining the infant, you
mote multiple congenital anomalies that include a small head (microcephaly), small eyes (micropthalmia),
a prominent occipit, and malformed ears. Echocardiography reveals a ventricular septal defect with the
aorta. Although the respiratory distress resolves with surfactant treatment, laboratory studies reveal
significant chromosomal abnormalities.

Which of the following is likely in association with this case?:

chromosome 22q11 deletions

tall stature, if the child survives infancy

karyotype 47, XX, +21

elongated skull

webbed neck

Explanation

The correct answer is choice D.

The case presented is suggestive of Trisomy 18; thus, The karyotype of this patient would be 47, XX, +18.
Also known as Edwards syndrome, it was described first in 1960, not only by J.H. Edwards and colleagues,
but independently by D.W. Smith and colleagues in that same year. Among live births, Edwards syndrome
is the second leading trisomy of an autosome, after Down syndrome (Trisomy 21) whose female karyotype

5
is 47, XX, +21 (choice C). Like Down syndrome, Edward syndrome manifests with a range of features that
include mental retardation along with abnormalities of multiple organ systems.

Trisomy of a chromosome results from a nondisjunction of sister chromatids during the first anaphase of
meiosis. After DNA of a germ cell is replicated, each chromosome exists in duplicate in structures called
chromatids. As the cell prepares to divide, sister chromatids of each pair are pulled toward opposite poles
of the cell. Anaphase is the phase during which the chromatids actually separate from one another.
Because errors occur, however, sometimes a chromatid or a piece of a chromatid remains attached to its
counterpart, resulting in a gamete that has an extra chromosome.

Although it has an estimated incidence of 1 in 6,500 live births, screening of pregnant women has revealed
that Trisomy 18 actually is present in 1 of 400 first trimester concepti, most of which are eliminated
through spontaneous abortion. Of those which make it to term, only 5-10 percent survive beyond one year
of age.

Infants born with Edward syndrome frequently present with overlapping digits in the hands and a plethora
of other structural signs, including microcephaly, micropthalmia, a prominent occipit, malformed ears, and
an elongated skull (choice D).

Neurological deficits, along with cardiac, pulmonary, gastrointestinal, and genitourinary malformations
occur with high frequency. Typically, the umbilical cord contains just one artery, instead of two. As for the
endocrine system, hypoplasia is common in glands such as the adrenal, the thymus, and thyroid.

Deletions in chromosome 22 (choice A) are what leads to diGeorge syndrome. This manifests with several
abnormalities that may immune deficiency, hypoparathyroidism, and cardiac anomalies, such as tetrology
of Fallot, which consists of pulmonary stenosis, right ventricular hypertrophy, a ventricular septal defect,
and an aorta that overrides the right ventricle in part. An aorta overriding the right ventricle does not
mean that the aorta is completely over the right ventricle; generally, it overrides both ventricles, instead of
overriding only the left. This places the aorta over the ventricular septal defect so that circulation in the
heart is more like the circulation in the three-chambered heart of an amphibian. diGeorge syndrome is
associated with fetal alcohol syndrome, phenylketonuria (PKU) in the mother, and fetal hydantoin and
carbamazepine syndromes.

Tall stature (choice B) is associated with a variety of genetic conditions, including Klinefelter syndrome,
(one or more extra X chromosomes in a male), and Marfan syndrome, an autosomal dominant condition
connected with mutations on chromosome 15. Webbed neck (choice E) is a classic feature of Turner
syndrome, caused by the lack of one of the X chromosomes in a female.

6.

A 24-year-old primigravida is late in the second trimester of pregnancy. She experiences the sudden onset
of some cramping lower abdominal pain. This is immediately followed by passage of some clear fluid per
vagina along with a foul-smelling discharge. This is an acute chorioamnionitis, with likely premature
rupture of membranes. The fetus is stillborn two days later. Examination of the placenta demonstrates
extensive neutrophilic infiltrates in the chorion and amnion.

Which of the following organisms is most likely to be responsible for these findings?

6
Mycobacterium tuberculosis

Herpes simplex virus

Escherichia coli

Treponema pallidum

Toxoplasma gondii

Explanation

The correct answer is choice C

The most common cause of chorioamnionitis is bacterial in origin. It usually starts in the maternal
urogenital tract where E.coli, typically a gastrointestinal flora, can ascend (from the rectal area) into the
uterus, most commonly with prolonged labor. Risk of developing this infection increases with each vaginal
examination. Symptoms include:

 high temperature and fever


 tachycardia
 sweating
 uterine tenderness
 vaginal discharge with unusual smell

Blood samples and amniotic fluid cultures confirm the diagnosis.

Maternal complications associated with chorioamnionitis include:

 endometritis
 pelvic and lung blood clots
 infections in pelvic region and abdomen

Fetal complications include the following:

 sepsis
 meningitis
 respiratory problems
 possible death

There is general consensus that Mycobacterium tuberculosis (Choice A) does not worsen the pregnancy
outcome nor does pregnancy appear to increase progression of inactive to active TB. However, respiratory
risks are enough indication to treat patients when TB is found to be positive on screening.

Herpes simplex virus (Choice B) traditionally was associated with infections of the oral area with fever
blisters. Yet, due to increasing oral-genital contact, the simplex variety may now be found in both areas.
The problem with an outbreak and pregnancy is not to the chorion and amnion but a risk to the baby with
exposure of his eyes as the fetus descends through the infected area and then contracts the virus. This can
lead to blindness if not treated appropriately.

Treponema pallidum (Choice D) is the spirochete responsible for syphilis which is a sexually transmitted
disease. Because of the serious risk to the fetus, pregnant women are screened for exposure to

7
it. Symptoms of congenital syphilis include:

 hemorrhagic rhinitis
 perioral fissures
 hepatosplenomegaly
 lymphadenopathy
 hydrops
 thrombocytopenia
 microcephaly
 glomerulonephritis
 neurologic impairment
 ocular involvement

Toxoplasma gondii (Choice E) infection may be asymptomatic in healthy women. However, the parasite
can infect the placenta and cause miscarriage, stillbirth or death shortly after birth. Congenital
toxoplasmosis can cause mental or motor developmental delays, cerebral palsy and epilepsy along with
visual impairment and possibly blindness.

7.

A primigravida at 37th week of gestation reported to the labor room with a known central placenta praevia
with heavy bleeding per vaginum. The bleeding began 2 hours before and the blood flow has stopped
over the last hour. Her pregnancy was also complicated by gestational diabetes and pregnancy induced
hypertension. She is morbidly obese and has had poor prenatal care. The fetal heart rate was normal at
the time of examination. She is hemodynamically stable.

The best management option for this patient includes which of the following:

Expectant treatment

Caesarian Section

Induction and vaginal delivery

Induction and forceps delivery

Observe on bedrest in the lithotomy position

Explanation

The correct answer is choice B

Placenta praevia is an obstetrical complication whereby the placenta is attached to the uterine wall
covering the cervix in this case (central). It is a leading cause of antepartum haemorrhage (vaginal
bleeding). This condition complicated approximately 0.5% or pregnancies. With the heavy bleeding, the
placenta is already in jeopardy of separating and so delivery should be hastened (observing on bedrest is
not a viable option- Choice E). Because the placenta cannot precede the fetus in delivery, a cesarean
section must be performed now.

8
Vaginal delivery whether from instrumentation or natural with induction is not a viable alternative because
of the placental location obstructing the cervical os. (choices C, D, A)

The following have been identified as risk factors for placenta praevia:

 Previous placenta previa, caesarean delivery, or D&C


 Women who have had previous pregnancies, especially a large number of closely spaced
pregnancies, are at higher risk.
 Women who are younger than 20 are at higher risk and women older than 30 are at increasing
risk as they get older.
 Women with a large placentae from twins or erythroblastosis are at higher risk.
 Women who smoke or use cocaine may be at higher risk.

 Race is a controversial risk factor, with some studies finding that people from Asia and Africa are
at higher risk and others finding no difference.

8.

A 41-year-old woman complains of "bleeding too much" from her vagina over the last 3 menstrual cycles.
She reports a history of menstrual cycles lasting between 19 and 23 days, with a period length of 8 days.
Sometimes, she must change sanitary napkins every hour, and occasionally she needs to go home from
work to change her clothes. She also has some light, apparently random, bleeding between periods. A
urine pregnancy test is negative. Her last Pap smear was performed 6 months ago and was read as
negative.

What would be the first step in management?

9
Reassure her that she is probably perimenopausal, and that it is normal to have menstrual changes
at this time. Prescribe oral contraceptive pills to regulate her bleeding.

Perform pelvic ultrasound to look for fibroids and evaluate the endometrial stripe.

Check thyroid-stimulating hormone (TSH) level, complete blood cell (CBC) count, and follicle-
stimulating hormone (FSH) level to evaluate for thyroid dysfunction, anemia, and confirm
perimenopause.

Perform endometrial biopsy to rule out malignancy.

Perform colposcopy to evaluate for the presence of cervical lesions.

Explanation

The correct answer is Choice D.

In women over the age of 35 with abnormal uterine bleeding, after pregnancy has been excluded, an
endometrial biopsy should be performed to rule out endometrial cancer or a premalignant lesion
(endometrial hyperplasia) as the reason for abnormal uterine bleeding.

While a benign cause for the bleeding (such as a submucosal fibroid, anovulatory bleeding, polyp, etc.) is
most likely in this case, the first step is to rule out malignancy before proceeding to imaging or other
studies to determine the precise cause of bleeding.

Oral contraceptive pills may be prescribed for perimenopausal women to regulate dysfunctional bleeding,
but the cause of the bleeding should be established and the possibility of malignancy excluded before this
is done.

Although it must be determined that abnormal vaginal bleeding is of uterine and not cervical origin, in this
case the patient had a normal cervical cytology screen within the past year, making it less likely that a
premalignant or malignant cervical lesion is the source of bleeding.

9.

A 39 year old woman, gravida 3, para 2 mother presents at your clinic at 16 weeks gestation. The father
of the fetus is 47 years-old with no noted health problems. The other two children, both of the same
father, are ages 6 and 3 and have no health problems either, nor does either parent have a family history
positive for disease resulting from chromosomal abnormalities. No physical abnormalities are found on
maternal physical examination and the uterine size is consistent with the dates. The alpha-fetoprotein and
estriol measurements taken as part of the maternal serum screen is found to be low for this gestational
age, while the beta-hCG component is found to be normal.

Which of the following should be the next step in management of this patent?

this is a normal pregnancy; have the patient return in four weeks for routine prenatal
exam

ultrasound examination

chromosomal studies

measurement of serum acetyl-cholinesterase level

amniotic fluid AFP determination

Explanation

10
The correct answer is choice C.

The triple test (or triple screen) measures Alpha-fetoprotein (AFP), estriol, and beta human chorionic
gonadotropin (beta-hCG) in the maternal blood. AFP and estriol tend to be low in cases of Down syndrome
(Trisomy 21) and Edward syndrome (Trisomy 18). In the latter case, beta-hCG also tends to be low, while
in the former case it tends to be higher than normal for the gestational age.

Trisomy of a chromosome results from a nondisjunction of sister chromatids during the first anaphase of
meiosis. After DNA of a germ cell is replicated, each chromosome exists in duplicate in structures called
chromatids. As the cell prepares to divide, sister chromatids of each pair are pulled toward opposite poles
of the cell. Anaphase is the phase during which the chromatids actually separate from one another.
Because errors occur, however, sometimes a chromatid or a piece of a chromatid remains attached to its
counterpart, resulting in a gamete that has an extra chromosome.

Trisomies are associated with advanced maternal age. The risk of having a Down syndrome or Edward
syndrome child rises sharply after age 35 for a woman, and dramatically after age 40 compared to the risk
for younger mothers. Although the age of the father seems to have little effect on the risk when the
mother is young, when in combination with advanced maternal age, paternal age is associated with the
risk of trisomies. Thus, despite the health of this couple and their children, this is not a normal pregnancy
(choice A); because of their age, this patient and her husband now are at fairly high risk for chromosomal
abnormalities. While beta-hCG is normal, it is normal in the context of the other two components of the
triple test being low, such that Down syndrome and Edward syndrome both are possible. Thus, the next
step is amniocentesis.

Amniocentesis is a test in which amniotic fluid is drawn through a needle under ultrasound guidance. While
the AFP in the fluid could be measured (choice E) it is certain to be low, since it's low in the mother's
blood. Thus, it is not a useful test. More useful is the fact that amniotic fluid contains fetal cells whose
chromosomes can be viewed as a karyotype and counted. Considering this mother's triple test,
amniocentesis is vital, a much higher priority even than ultrasound (choice B) since trisomies need to be
ruled out.

Measurement of serum acetyl-cholinesterase level (choice D) is useful in cases when AFP level is high,
which could indicate omphalocele, gastroschisis, or neural tube defects such as spina bifida. The neural
tube is at the midline of the embryo and develops into the central nervous system. When it does not close
completely, parts of the spinal cord can be covered incompletely, usually in the lumbar and sacral levels.
Nonclosure also can occur at the level of the head, resulting in anencephaly, resulting in a lack of a
forebrain and parts of the skull and scalp that go over it.

10.

A healthy 26 year old primigravida presents in your office with vaginal spotting and mild cramping. She is
at gestational week 10. Examination shows that the bleeding is minimal, with no exudate and the patient
is afebrile. Forty-eight hours later, a spontaneous abortion occurs.

Which of the following conditions is the most likely cause?

Placental insufficiency

Perinatal infection

Chromosomal abnormality

Toxemia

Placenta previa

Explanation

11
The correct answer is choice C.

According to early pregnancy loss research, about 30% of pregnancies (both unrecognized and clinically
recognized / confirmed) end in natural or spontaneous termination. Of the clinically recognized / confirmed
pregnancies, approximately 15% are spontaneously lost. Sonograms are helpful in determining whether an
early pregnancy is viable. Abnormalities such as trisomy 16 and tetraploidy, not typically seen in livebirths,
are found in many first trimester abortuses. Many first trimester pregnancy losses relate to fetal, not
maternal, problems. Other causes of first trimester spontaneous abortions include abnormal cell division
after fertilization of the egg, problems with implantation of the fertilized egg, problems related to the
developing placenta, and structural abnormalities of the developing baby.

Placental insufficiency (choice A) as a problem is greatest in the third trimester. A good example of a cause
is maternal diabetes.

Perinatal infection (choice B) would most likely be seen with accompanying symptoms in the mother.
Further, infections usually have a greater impact upon fetal loss later in pregnancy.

Toxemia (choice D) is pregnancy induced hypertension which may develop into pre-eclampsia. Toxemia
has the greatest impact in the third trimester and during delivery.

Women with placenta previa (choice E) often present with painless, bright red vaginal bleeding. This
bleeding often starts mildly and may increase as the area of placental separation increases. Placenta previa
can sometimes occur in the later part of the first trimester, but usually during the second or third. Praevia
typically is suspected only when there is bleeding after 24 weeks of gestation.

11.

A 22 year old primagravida misses a menstrual period and finds that she is positive using a home
pregnancy test kit. Three weeks later she experiences some vaginal bleeding. A month later she retests
herself and finds that the pregnancy test is negative.

Which of the following explanations is most probable for the pathogenesis of these events:

A non-disjunctional event had occurred in meiosis

Maternal antibody was crossing the placenta

Premature rupture of membranes had occurred

Folic acid was lacking from her regular diet

She had contracted a Treponema pallidum infection

Explanation

The correct answer is choice A

The most common cause of spontaneous abortion in the first trimester is chromosomal abnormalities of which most occur during c
Nondisjunction in meiosis refers to the failure of either one of two homologous chromosomes from passing to separate cells in the
or of the two chromatids passing to separate cells during the second meiotic division.

Table 1: Summary of the types of Abortion

12
Maternal antibody crossing the placenta (Choice B) describes an event seen in Rh or blood type incompatibility causing erythrobla
and hydrops.

Premature rupture of membranes (Choice C) is an event that occurs in the late 2nd or 3rd trimester. If this goes undetected, can
chorioamnionitis with eventual fetal demise because the fetus has difficulty fighting off the infection.

Folic acid deficiency (Choice D) in pregnancy has been associated with an increased incidence of infant neural tube defects.

Treponema pallidum (Choice E) is a spirochete responsible for syphilis. Congenital syphilis is associated with microcephaly. menta
and neurological complications.

12.

A 33-year-old woman at 35 weeks' gestation presents to the emergency department with constant pelvic
pain. She says that she had a small amount of dark red vaginal bleeding earlier today after sexual
intercourse. Her blood pressure is 80/50 mmHg, and she has tachycardia. The uterus is firm and tender to
palpation.

Which of the following is the most likely diagnosis?

Abruptio placentae

Cervical laceration

Placenta previa

Preterm labor

Vasa previa

13
Explanation

The correct answer is choice A

Abruptio Placenta,( or placental bruption) is an obstetric catastrophe where the placental lining separates
from the uterus. It is the most common pathological cause of third trimester bleeding although it can
occur anytime after 20 weeks and is accompanied by pain. is a separation of the placenta from the wall of
the uterus which can be concealed (retroplacental), partial, or complete. Signs include mild to severe
acute abdominal/pelvic pain and bleeding. Depending on the severity, the patient may go into shock,
tachycardia and low or thready blood pressure.

Abruptio placenta occurs in 1% of pregnancies with a fetal mortality rate of 20-40% world wide depending
on the separation degree.

Symptoms include:

 contractions that don't stop ( or tetany)


 uterine pain
 pain in abdomen or pelvis
 vaginal bleeding
 pallor

Fetal effects:

 fetal distress or possible stillbirth


 prematurity leading to breathing and feeding difficulties
 low level of oxygen compromising neurological development
 mental retardation
 fetal anemia
 Death shortly after birth

Maternal effects:

 severe anemia leading to transfusion need


 uterine atony leading to further hemorrhage
 DIC or clotting problems
 shock leading to organ shutdown ( of liver, kidney and pituitary gland)

Cervical lacerations (choice B) can occur postcoital in a pregnant cervix as it is more fragile. However,
despite the signs of bleeding, one would not expect to see abdominal/pelvic pain.

Placenta Previa (Choice C) is the other condition seen as a third trimester bleeding problem, an obstetrical
complication at the end of the second trimester and third trimester. It distinguishes itself from abruptio
placenta by the lack of constant pain. Placenta previa is a condition whereby the placenta is preceding the
fetus in the birth canal and can be partial (where only part of the cervical os is obstructed) to a complete

14
obstruction of the cervical os. The presumptive diagnosis is made usually by ultrasound.

Vasa previa (Choice E) is an obstetrical condition where blood vessels within the placenta or umbilical cord
cross the cervical os or in close proximity so that vessels may become trapped between the cervix and
fetus. Fetal exsanguinaation is a significant risk if the vessels are disrupted, associated with a 50-75%
fetal mortality rate.

13.

A 21-year-old nulligravid woman visits her gynecologist for an annual examination. When questioned
regarding any unusual symptoms or concerns, she reports having an intermittent thin vaginal discharge
and continual vaginal irritation. She was recently treated for sinusitis with an antibiotic, although she does
not recall its name. She is sexually active and is taking oral contraceptives; she and her partner do not
always use condoms. She has been monogamous for the past 2 years, and recently they began practicing
mutual oral sex. Her medical and surgical history is otherwise noncontributory. She states that her
boyfriend has had no symptoms.

On examination, the patient’s vulva and vagina appear moderately inflamed, and she has a thin, gray,
odorless, purulent discharge. She has no cervical motion tenderness. Colpitis macularis ("strawberry-like"
spots on the cervix portio) are observed. Routine screening for chlamydia and gonorrhea are performed
with a Papanicolaou test. Microscopic examination of a wet preparation obtained during the test shows
small, flagellated organisms.

Which of the following is the patient’s most likely vaginal pH?

<2.5

2.5 to 3.8

3.8 to 4.2

4.2 to 4.5

> 4.5

Explanation

The correct answer is choice E

The vaginal infection described in this patient is trichomoniasis, a sexually transmitted disease caused by
an anaerobic,flagellated protozoan parasite.Infections between men and women are the same though
females are symptomatic.

Symptoms include a green or gray discharge, itching and dysuria. The infection occurs if the normal
acidity of the vagina is shifted from a healthy, semi-acidic pH of 3.8-4.2 to a much more alkaline one being
5-6, encouraging growth of the T.vaginalis. Symptoms include:

 inflammation with increasing number of organisms


 greenish yellow frothy vaginal exudate
 pruritus
 strawberry marks on the cervix or vagina on examination in 10% of cases

Classically, infected women have a transparent "halo" around their superficial cell nucleus when examiining
the pap smear. Trichomonas is traditionally diagnosed via a wet mount in which "corkscrew" motility is

15
seen.

With a pH less than 4.5, a fungal infection from Candida albicans should be suspected.

14.

There is research evidence to establish risk factors as being linked to breast cancer risk. Non-modifiable
risk factors are those which cannot be changed or controlled. Modifiable risk factors are those which can
potentially be changed or controlled.

All of the following are risk factors for breast cancer except:

Increased age

multiparity

early menarche

delayed childbirth

high socioeconomic status

Explanation

The correct answer is choice B

The American Cancer Society Breast Cancer Facts and Figures 2009-2010 estimate that the lifetime risk for a female getting breas
1 in 8.

Risk factors for breast cancer in addition to a family history of breast cancer include all those conditions which result in a prolonge

16
exposure. These conditions are:

 late menopause
 early menarche
 delayed childbearing
 obesity
 nulliparity

Women with one first-degree relative with breast cancer have a twofold increased risk of developing breast cancer and those with
degree relatives have a 14- fold increased risk of developing breast cancer.

Studies show that there is an increased risk of breast cancer in higher socioeconomic groups which was statistically significant for
groups.

Figure 1: Overview of breast cancer risk factors

Figure 2: Specific Risk factors for breast cancer

17
15.

A 35 year-old primigravida undergoes amniocentesis on account of an elevated alpha-fetoprotein on her


triple test, and advanced maternal age. Ultrasound testing throughout the remainder of the pregnancy
reveal no abnormalities, and testing of fetal cells reveals a normal male karyotye of 46,XY. In spite of this,
the infant is born with feminized external genitalia, with an internal male reproductive tract and testes
retained in the abdominal cavity.

Which of the following etiologies may explain this scenario?

5-alpha-reductase deficiency

partial testicular dysgenesis

17-alpha-hydroxylase deficiency

complete androgen resistance

Sertoli-only syndrome

Explanation

The correct answer is choice A.

While testosterote is required for differentiation of the Wolffian ducts in utero into and internal male
reproductive tract, dihydrotestosterone (DHT) is not required. However, for the external genital slit to
develop into the penis, scrotum, and prostate, DHT is needed. Since its synthesis depends on the enzyme
5-alpha-reductase, the lack of this enzyme results in feminized external genitalia. While the infants
affected with this condition will appear as normal females throughout childhood, if not treated the condition

18
will lead to virilization at puberty, since the child does have testicles, though hidden. This means that a
penis actually will grow, since it is present in a small, immature form from birth. Consequently, the
condition has been labeled the "penis-at-twelve" syndrome, and obviously can lead to a serious identity
crisis for the child. Thus, there are two treatment options. If discovered early in infancy, development of
external male genitalia can be stimulated by treatment with DHT. Otherwise, the undescended testicles can
be resected (orchiectomy) and the child given estrogen substitution therapy at puberty. In the latter case,
she will look at feel as a normal female, but of course will be infertile.

In cases of testicular dysgenesis (choice B), development of testes in utero means a decrease in
testosterone secretion as well as Müllerian regression factor (MRF). While this causes feminization of
external genitalia, the lack of MRF results in feminization of internal tracts as well. While the dsygensis
may be partial, in this case there was enough MRF to support the masculinization of the internal tract.

In cases of 17-alpha-hydroxylase deficiency (choice C), testosterone is not synthesized. This leads to
feminized external genitalia, as well as degeneration of the Wolffian ducts. Secretion of MRF causes
degeneration of the Müllerian ducts. Children with this deficiency may be hypertensive, due to elevated
secretion of deoxycorticosterone by the adrenal cortex.

In cases of complete androgen resistance (choice D), external genitalia are feminine, but internal tracts do
not develop, either male or female. Due to a lack of an androgen receptor, the Wolffian ducts degenerate,
as do the Müllerian ducts, since MRF is not deficient.

Sertoli-only syndrome (choice D) is a condition in which only the Sertoli cells of the seminiferous tubules
are present, which also is called germinal cell aplasia. In this case, spermatogenesis does not occur, while
FSH levels are elevated on account of decreased secretion of inhibin by Sertoli cells. Male and female
internal tracts may develop, since MRF is not produced, though since Leydig cells are normal, testosterone
is secreted, and thus external genitalia are masculine.

16.

A 36-year-old woman presents for her initial obstetrical evaluation at 19 weeks gestation. She has
enjoyed relative good health and has a negative past medical history. The patient does admit to smoking
3-5 cigarettes daily and denies alcoholic consumption.

Ultrasound assessment confirms a single intrauterine pregnancy at 19 weeks but also reveals:

shortened femur length and a nuchal cyst. The vertebrae and rib cage appear normal for the stated dates
but there seems to be an intracardiac echogenic foci.

What is the most likely presumptive diagnosis?

Achondroplasia

chronic fetal hypoxia from cigarettes

Down Syndrome

congenital dwarfism

Cri du Chat syndrome

Explanation

The correct answer is choice C

Ultrasonography cannot diagnose Down Syndrome although certain markers may suggest the occurrence

19
at which time an amniocentesis is done. Findings include

 increased nuchal translucency


 shortened femur length
 choroid plexus cysts
 intracardiac echogenic foci

Down syndrome is a trisomy 21 genetic disorder. It can be diagnosed by performing a karyotype on the
fetus with cells obtained during an amniocentesis- a procedure in which a small amount of amniotic fluid
(containing fetal cells) is extracted from the amnion surrounding a developing fetus and to measure the
alpha-fetoprotein (AFP) which is abnormally low in Down syndrome.

Although the procedure is routine, and almost 70% of women who undergo the test report little to no
discomfort, possible complications include infection of the amniotic sac from the needle, and failure of the
puncture to heal properly, which can result in leakage or infection. Serious complications can result in
miscarriage. Other possible complications include preterm labor and delivery, respiratory distress, postural
deformities, fetal trauma and alloimmunisation of the mother.

Studies from the 1970s originally estimated the risk of amniocentesis-related miscarriage at around 1 in
200 (0.5%). A more recent study (2006) has indicated this may actually be much lower, perhaps as low as
1 in 1,600 (0.06%). In contrast, the risk of miscarriage from chorionic villus sampling (CVS) is believed to
be approximately 1 in 100, although CVS may be done up to four weeks earlier, and may be preferable if
the possibility of genetic defects is thought to be higher.

Achondroplasia (Choice A) which is a type of congenital dwarfism (Choice D) is due to an autosomal


dominant mutation in the fibroblast growth factor receptor gene 3. This creates a cartilage formation
abnormality leading to severely shortened bones.

A sonogram can detect achondroplasia prenatally with a disparity between the femur length and biparietal
diameter by gestational age and trident hand configuration observed if fingers are extended, whereby a
skeletal survey is performed.

Results show:

 large skull with narrowed foramen magnum and small skull base
 vertebral bodies are short and flattened
 narrowed spinal canal
 short and thick tubular bones with metaphyseal cupping and flaring
 short metacarpals and phalanges with a trident configuration
 ribs are short

Tobacco usage in pregnancy (Choice B) is associated with premature delivery, low birth weight and even
stillbirth due to hypoxia caused from nicotine and carbon monoxide. Vessels are constricted including the
umbilical vessels. The effects on the fetus are symmetrical.

Cri du chat (Choice E) is also known as chromosome 5p deletion syndrome or Lejeune's syndrome. It is a
rare genetic disorder. Symptoms include:

 low birth weight


 severe cognitive,speech and motor delays
 behavioral problems
 excessive drooling
 constipation
 strabismus

20
Sonogram findings may include:

 microcephaly
 intestinal malrotation
 dislocated hips
 hypospadias
 renal malformations ( horseshoe kidneys , renal ectopia or hydronephrosis)
 clinodactyly of the fifth fingers
 low set ears
 patent ductus arteriosus

 atrial septal defect or ventricular septal defect

17.

A 39-year-old woman has been unable to conceive for the past 12 months. She undergoes follicle-
stimulating hormone (FSH) and estradiol testing on day 3 of her cycle.

Which one of the following combinations indicates favorable follicular reserve?

FSH level of less than 10 mIU per mL (10 IU per L) and estradiol level of less than 80 pg per mL
(294 pmol per L).

FSH level of 10 to less than 15 mIU per mL (15 IU per L) and estradiol level of 80 to less than 100
pg per mL (368 pmol per L).

FSH level of 15 to less than 20 mIU per mL (20 IU per L) and estradiol level of 100 to less than 120
pg per mL (440 pmol per L).

FSH level of 20 to less than 26 mIU per mL (26 IU per L) and estradiol level of 120 to less than 140
pg per mL (514 pmol per L).

FSH level of 26 to less than 30 mIU per mL (30 IU per L) and estradiol level of 140 to less than 160
pg per mL (588 pmol per L).

Explanation

The correct answer is choice A

FSH level of less than 10 mIU per mL (10IU per L)

Estradiol level of less than 80 pg per mL (294 pmol per L)

Poor ovarian reserve is also known as premature ovarian aging or impaired ovarian reserve which is a
state of low fertility characterized by lowered oocyte numbers or development. This is usually
accompanied by high FSH levels (as one would expect in menopause).

In the female fetus oocyte numbers peak by 16 weeks of gestation, reaching up to 7 million. They
decrease progressively and at the time of birth a female will have about 2 million eggs. As she approaches
menarche the woman will have about 500,000. This primordial follicular disappearance continues
throughout reproductive life and accelerates approximately 10 years prior to the menopause, by which

21
time the number of eggs have fallen to a few hundred.

At all stages of the menstrual cycle there are a number of small antral follicles present. This number
changes with age, being at its highest in younger women. As the level of FSH rises at the beginning of the
menstrual cycle more follicles are recruited and some are then selected for further development. Follicle
stimulating hormone (FSH) is crucial

for follicular development as it is the hormone that controls the process of recruitment, selection and finally
the full development of the maturing follicle.

As women age, the number of recruited follicles decrease. Consequently the suppression of the FSH level
decreases and, as a result, the level of FSH increases with age in an attempt to continue to recruit the
ever-decreasing number of eggs within the ovary. The level of FSH provides a biological marker for ovarian
reserve. The higher the level, the less the ovarian reserve.

There are many tests to assess ovarian reserve: the most common and the cheapest is the measurement
of day 3 FSH as well as the level of estradiol.

18.

A 24 year old G1 P0 is seen in the ER. Her LMP was 8 weeks ago. She is experiencing lower abdominal
cramping and heavy vaginal bleeding with clots. Examination reveals a soft abdomen with mild lower
abdominal tenderness. On pelvic exam, the vagina is filled with blood and clots. The cervical os is opened
and tissue is protruding. The uterus is enlarged to a 6 week size. She is hemodynamically stable and
appears non-toxic.

Which of the following is the most likely diagnosis:

complete abortion

inevitable abortion

threatened abortion

missed abortion

incomplete abortion

Explanation

The correct answer is choice E

Threatened miscarriage: Vaginal bleeding, abdominal/pelvic pain of any degree, or both during early pregnancy represents a threa
miscarriage. Approximately a fourth of all pregnant women have some degree of vaginal bleeding during the first 2 trimesters. Ab
these cases progress to an actual miscarriage. Bleeding and pain accompanying threatened miscarriage is usually not very intense
miscarriage rarely presents with severe vaginal bleeding. On vaginal examination, the internal cervical os is closed and no cervica
tenderness or tissue is found. Diffuse uterine tenderness, adnexal tenderness, or both may be present. Threatened miscarriage is
the absence of passing/passed tissue and the presence of a closed internal cervical os. These findings differentiate threatened mis
from later stages of a miscarriage.

Inevitable miscarriage: Vaginal bleeding is accompanied by dilatation of the cervical canal. Bleeding is usually more severe than w
threatened miscarriage and is often associated with abdominal pain and cramping.

Incomplete miscarriage: Vaginal bleeding may be intense and accompanied by abdominal pain. The cervical os may be open with
conception being passed, or the internal cervical os may be closed. Ultrasonography is used to reveal whether some products of c

22
are still present in the uterus.

Complete miscarriage: Patients may present with a history of bleeding, abdominal pain, and tissue passage. By the time the misca
complete, bleeding and pain usually have subsided. Ultrasonography reveals a vacant uterus. Diagnosis may be confirmed by obse
the aborted fetus with the complete placenta, although caution is recommended in making this diagnosis without ultrasonography
can be difficult to determine if the miscarriage is complete.

19.

A 21 year old female who is 26 weeks pregnant is admitted with 3 generalized tonic clonic seizures. She
has no previous history of epilepsy or neurological disorder. Her blood pressure is noted to be 210/90.
Temperature is 37.9 °C. Urinalysis demonstrates marked proteinuria.

What is the ultimate management of this patient?

Load with intravenous phenytoin

Treat symptomatically with benzodiazepines

Treat with intravenous magnesium

Deliver fetus and placenta

Observe for 6 hours, blood cultures and paracetamol

Explanation

23
The correct answer is choice D

Eclampsia (Greek, "shining forth"), an acute and life-threatening complication of pregnancy, is


characterized by the appearance of tonic-clonic seizures, usually in a patient who had developed
preeclampsia. Pre-eclampsia is a medical condition in which hypertension arises in pregnancy (pregnancy-
induced hypertension) in association with significant amounts of protein in the urine. The patient is
suffering from Eclampsia which is characterized by:

 hypertension
 proteinura
 edema
 seiures (ie. eclamptic convulsion)
 Other cerebral signs may precede the convulsion such as nausea, vomiting, headaches, and
cortical blindness
 Other organ symptoms may be present including abdominal pain, liver failure, signs of the HELLP
syndrome, pulmonary edema, and oliguria.
 HELLP is an abbreviation of the main findings: Hemolytic anemia, Elevated Liver enzymes and Low
Platelet count

The treatment of eclampsia requires prompt intervention and aims to prevent further convulsions, control
the elevated blood pressure and the immediate delivery of the baby and placenta. The condition will not
improve until the uterine contents are evacuated. The fetus may have been already compromised by
intrauterine growth retardation, and with the toxemic changes during eclampsia may suffer fetal distress.
Placental bleeding and placental abruption may occur.

Choices A,B and C are given to stabilize the mother after delivery as eclampsia symptoms may persist for
10 days to 2 weeks.

20.

A right adnexal mass is palpated during the routine pelvic examination of an asymptomatic 60-year-old
woman. Her last menstrual period occurred at age 49. Transvaginal ultrasonography reveals a 10 cm
complex ovarian cyst, with both solid and cystic components. Her past medical history is relevant for
hypertension but no known malignancies.

Which one of the following statements is correct concerning the management of this case?

If the cancer antigen (CA)-125 blood test is normal, the physician can be reassured that this is a
benign cyst.

Since the mass is partially cystic, it can be followed by transvaginal sonograms every 3 months.

A percutaneous biopsy of the solid component of the mass is indicated.

The mass requires surgical excision for definitive diagnosis and management.

Ultrasound examinations and serum CA-125 antigen measurements at 3 month intervals are
recommended for management.

Explanation

The correct answer is Choice D.

Postmenopausal women with a non-simple cyst as evidenced by ultrasound examination should undergo

24
surgical intervention by a gynecologic oncologist due to the higher risk of malignancy in postmenopausal
women with adnexal masses.

The common causes of adnexal masses in premenopausal women include:

 physiologic follicular cysts


 corpus luteum cysts
 ectopic pregnancy
 endometriomas
 PCOS
 tubo-ovarian abscess
 benign tumors

However, this patient is postmenopausal. In postmenopausal women with adnexal masses, both primary
and metastatic malignancies must be considered. The presence of solid components within the cyst as well
as the patient's age raise the suspicion for malignancy in this case. Surgical intervention with biopsy is
warranted to rule out malignancy.

Follow-up with serial ultrasound examinations and CA 125 measurements may be appropriate for women
who have a simple cyst (with no solid areas) and are asymptomatic with normal cervical cytology and
normal serum CA 125 concentration, provided there is no other suspicion of malignancy.

While serum glycoprotein CA 125 concentration (normal <35 U/mL) is elevated in over 80 percent of
women with advanced ovarian cancer, it is less sensitive in the early stages of disease.

21.

A 21 year old G1P0 presents at 39 weeks of gestation in active labor. Clinical pelvimetry shows her pelvic
dimensions as follows: Pelvic sidewalls are straight; ischial spines are not prominent; pubic arch is wide;
sacrum is hollow; and sacro-sciatic notch is well rounded.

Based on the general bone architecture, which is the type of pelvis this patient has:

Obsetric

Android

Anthropoid

Gynecoid

Platypoid

Explanation

The correct answer is choice D

The gynecoid pelvis describes the typical female structure where the posterior sagital diameter at the inlet
is slightly shorter than the anterior sagital diameter. The sides of the posterior segment are well rounded
and the forepelvis is also well rounded and wide. The inlet is slightly oval or round with a flatter iliac bone.
Side walls are straight and the spines are not prominent. The pubic arch is wide with a transverse
diameter at the spines of 10 cm. or more.

25
The Android pelvis (Choice B) is a type where the posterior sagital diameter at the inlet is much shorter
than the anterior sagital. The sides are not rounded but tend to form a wedge at their point of junction.
The forepelvis is narrow and the sidewalls are usually convergent. Ischial spines are prominent.

In the Platypelloid type (Choice E) is a flat gynecoid pelvis with a short anterior-posterior and a wide
transverse diameter.

Anthropoid pelvis (Choice C) is essentially where the A-P diameter of the inlet is greater than the
transverse. This forms an oval A-P.

22.

A 32-year-old nulliparous woman has a uterine size of approximately 13 weeks' gestation as a result of
fibroid tumors. These tumors are thought to contribute to her symptoms of menorrhagia, pelvic pain, and
delayed conception, and myomectomy is recommended.

Which one of the following statements about myomectomy is most accurate?

The procedure requires laparotomy

Perioperative morbidity with myomectomy is substantially lower than that with hysterectomy

Fibroid tumors recur in less than 10 percent of patients within five years after myomectomy

If the patient becomes pregnant after the procedure, cesarean delivery will be required

The outcome depends on the number and size of fibroid tumors that are treated

Explanation

The correct answer is Choice E.

Uterine fibroids are benign, arising from uterine smooth muscles but can have symptoms ranging from
menstrual abnormalities, pressure, pain and infertility. Depending on the location, size and number of

26
myomas to be removed, one may experience a greater blood loss during a myomectomy and an increase
in the incidence of postoperative adhesions. Myomas are designated by three types, depending on where
they are located.

 Subserosal- located under the outer serosal surface of the uterus


 Intramural- located within the muscular wall of the uterus
 Submucosal- under the endometrium

The surgical outcome depends on the number and size of fibroid tumors that are treated. It depends on
how close the tumors are to the main vessels, the number of tumors and the types of tumors i.e.
subserosal, submucosal or intramural. Morbidity will also depend on the mode by which the fibroids are
treated i.e. surgical myomectomy versus embolization of the uterine artery.

Laparotomy (Choice A) is no longer the only way that myomas can be treated. A viable option for
eradication of uterine fibroids are uterine artery embolization or embolization of the vessels feeding the
myomas. The procedure involves insertion of a catheter through the femoral artery and advanced to the
uterine arteries. Minute polyvinyl alcohol particles are then injected into the vessels through an invasive
radiological procedure. By blocking the uterine arteries, the reduced blood flow to the myomas causes
them to undergo necrosis.

Another option is medical therapy to induce hypoestrogenism causing a reduction in size of the myomas
and then destruction of the myomas through hot probes inserted during a laparoscopy.

Perioperative morbidity with myomectomy is not substantially lower than that with hysterectomy (Choice
B)

A majority of fibroids are found to grow back within five years after surgical intervention (Choice C)

If the patient becomes pregnant after the procedure, (Choice D) cesarean delivery will not necessarily be
required. The location of the myomas removed and the extent of the surgery will dictate the need for
cesarean as the delivery mode. Most physicians feel that surgical delivery will be necessary only if the
intrauterine cavity was entered at the time of the myomectomy. If a pedunculated myoma was clipped and
removed for example, it will have no bearing on the function of the pregnant uterus and subsequent mode
of delivery.

23.

A 32 year old woman in her third trimester presents with painless and profuse bright red vaginal bleeding.
Pelvic examination is deferred. Transvaginal ultrasonography reveals an abnormally positioned placenta.

Which of the following is the most likely diagnosis?

placenta accrete

placenta previa

abruptio placentae

bloody show

vasa previa

Explanation

27
The correct answer is choice B

Placenta previa is an obstetric complication that occurs in the second and third trimesters of pregnancy and
is characterized by painless vaginal bleeding. It is one of the leading causes of vaginal bleeding in the
third trimester and may cause serious morbidity and mortality to fetus and mother. It is a condition
whereby the placenta precedes the fetus in the uterus and is either partially obstructing the cervical canal
or totally whereby the entire internal os is covered by placenta. In the least extensive varieties (marginal
placenta previa and low-lying placenta), the placenta barely encroaches upon the internal os. The initial
presumptive diagnosis is made most often by ultrasound.

The etiology of placenta previa is unknown but may be related to multiple gestations, advanced maternal
age, prior cesarean sections, and smoking. It complicates nearly 5 of 1000 deliveries. Women over 30
years of age are three times more likely to have placenta previa than women less than 20.

Placenta accrete (choice A) is a condition where villi are in direct contact with myometrium without an
intervening decidual cushion. If this is not determined prior to delivery, there is a large risk of postpartum
hemorrhage.

Abruptio placentae (Choice C) is a common cause of serious third-trimester bleeding whereby the normally
situated placenta prematurely separates from the wall of the uterus after the twentieth week of gestation.
Abruption is classified as either severe or mild, depending upon the degree of separation.

The bleeding may be confined to the uterus as in a retroplacental hematoma with no external bleeding
(concealed hemorrhage) or where blood escapes into the vagina, with external hemorrhage resulting.
Maternal signs include vaginal bleeding, shock (if severe), uterine tenderness and rigidity.

Bloody show (Choice D) refers to the passage of a small amount of blood or blood-tinged mucus at the end
of pregnancy (passage of mucus plug) signaling the start of labor. The cervix changes shape, freeing
mucus and blood that occupied the cervical glands or cervix prior to labor. Passage does not denote a risk
to the mother or baby.

Vasa previa (Choice E) is an obstetrical condition where blood vessels within the placenta or umbilical cord
cross or un in close proximity to the inner cervical os, trapped between the cervix and the fetus. This may
occur due to a velamentous insertion of the cord or vessels running between the placenta and a
succenturiate lobe. This occurs in one per 3000 delieries and the cause is unknown. Color doppler
ultrasonography visualizes the vessel course and pulse doppler ultrasonography confirms the fetal origin.

The risk of fetal exsanguination is significant if the vessels are torn with membrane rupture, having a 50-
75% fetal mortality rate. If compressed during labor, the vessels can cause fetal heart decelerations.
Cesarean delivery is the delivery of choice for this after confirming fetal lung maturity.

24.

Match the most closely associated lettered pathologic finding involving the placenta with the description
below:

An ultrasound performed at 19 weeks reveals a fetus that is difficult to visualize because there are kidneys
present with multicystic renal dysplasia. No other anomalies are noted upon examination of the stillborn
two months later, but there are varus deformities of the feet, a flattened face, and prominent infraorbital
creases.

Abruption

Amnion nodosum

Amnionic band syndrome

Placenta accreta

28
Placenta previa

Explanation

The correct answer is choice B

Amnion nodosum, found more commonly in babies with cystic hyperplastic kidneys, renal agenesis or
oligohydramnios, are seen as areas of calcification within the placenta. It is a placental hallmark of severe
and prolonged oligohydramnios and shows nodules of amorphous granular material present on the surface
of the amnion. Additional symptoms include:

 abnormal ears
 hypoplastic lungs
 renal agenesis
 polycystic kidneys
 urethral obstruction
 stillbirth

Abruption (Choice A) refers to premature separation of the placenta which presents as vaginal bleeding
which can be heavy depending on the extent of the abruption and uterine rigidity presenting as pelvic and
abdominal pain.

Amniotic band syndrome (Choice C) is an obstetrical condition where constriction rings around the digits,
arms and legs of the baby occur with amniotic bands and extremities swelling distal to the constriction with
possible amputation. Many believe that this is due to partial rupture of the amniotic sac involving only the
amnion. Fibrous bands of the ruptured amnion float in the amniotic fluid and encircle and trap some parts
of the fetus. Later as the fetus grows and the bands do not, they become constricting, with ensuing blood
circulation reduction.

Placenta Accreta (Choice D) is an obstetrical complication whereby the placental implantation extends
beyond the endometrium into the myometrium which presents as a problem with hemorrhage intrapartum
and postpartum.

Placenta Previa (Choice E) refers to the condition where the placenta is obstructing the cervical os (totally
or partially) and precedes the baby in the birth canal.

25.

A hypertensive pregnant woman with gestational diabetes at 34 weeks presents to the ER with a history of
pain in abdomen, bleeding per vaginal and loss of fetal movements. Her pregnancy prior to this has been
uneventful except the recent elevation in blood pressure and gestational diabetes. Her prior 3 pregnancies
have been normal. She denies trauma or sexual intercourse and was feeling fetal movements the prior to
her presentation.

On examination, the uterus is contracted with increased uterine tone. Fetal heart sounds are absent.
There is obvious bleeding from her vagina but no fetal parts were obvious at this point.

The most likely diagnosis includes which of the following:

29
Placenta praevia

Hydramnios

Premature labor

Abrutio placenta

preeclampsia

Explanation

The correct answer is choice D

Abruptio Placenta- is an obstetrical complication, occurring after the 20th gestational week whereby the
normally situated placenta prematurely separates from the uterine wall and is associated with vaginal
bleeding, uterine rigidity, abdominal and pelvic pain and possible pallor. Abruption is classified as either
severe or mild, depending upon the degree of separation.

Bleeding may be confined to the uterus as in a retroplacental hematoma with no external bleeding i.e.
concealed haemorrhage or where blood escapes into the vagina with external haemorrhage. Shock may
ensure if severe. Direct causes include:

 abnormally short umbilical cord


 injury to the abdomen
 sudden loss in intrauterine volume (with rapid amniotic fluid loss on rupture or first twin delivery)

Risk factors include:

 advanced maternal age


 cocaine use
 diabetes
 cigarette smoking
 alcoholic drinks
 hypertension
 history of prior placenta abruptio
 Increased uterine distention (as with multiple pregnancies or polyhydramnios)

Placenta praevia (Choice A) is an obstetrical complication in the third trimester and is characterized by
painless vaginal bleeding. It is one of the leading causes of vaginal bleeding in the third trimester and is a
state whereby the placenta precedes the fetus in the uterus, either partially or totally obstructing the
cervical canal.

Hydramnios (Choice B) also known as polyhydramnios is condition where there is excessive amniotic fluid
surrounding the fetus. Symptoms include:

 rapid uterine growth


 risk of premature contractions

Premature labour (Choice C) or pre-term labor, begins more than three weeks before the due date.
Contractions cause the cervix to efface and or dilate.

Preeclampsia (Choice E) is an obstetrical complication consisting of:

 edema

30
 proteinuria

 hypertension

26.

A 25-year-old previously healthy primigravida is in the first trimester of pregnancy. During two successive
prenatal visits, she has fasting serum glucose levels of 127 and 131 mg/dL. Prior to this pregnancy, her
fasting serum glucose was 80 mg/dL. A hemoglobin A1C level is 8.1% at the last visit, at 18 weeks
gestation. She feels well and has no major health problems.

Which of the following problems is most likely to become apparent in the latter part of her pregnancy?

Intrauterine fetal growth retardation

Ketoacidosis

Hyperosmolar coma

Congenital anomalies

Placental insufficiency

Explanation

The correct answer is choice E

Placental insufficiency is a state whereby there is a diminished supply of nutrients and oxygen transported
to the developing fetus. Causes include the following conditions:

 Diabetes
 Hypertension
 Smoking

The condition described above is gestational diabetes which is diagnosed usually at 28 weeks, possibly
from insulin resistance. The hyperglycemia (due to an inability to metabolize the sugar) creates
hyperglycemic condition in the fetus as well, causing macrosomia which increases the risk of shoulder
dystocia at birth.

Intrauterine fetal growth retardation (Choice A) occurs when the placenta is insufficient to meet fetal
demands, leading to malnutrition of the developing fetus. Contributing factors include:

 tobacco smoking
 alcoholism
 drug addiction
 severe anaemia
 thrombosis
 pre-eclampsia
 chromosomal abnormalities
 connective tissue diseases (systemic lupus erythematosis)
 damaged placental tissue due to chronic renal failure, sicle cell anemia or PKU

It is the opposite of what is found in gestational diabetes with macrosomia and larger size than date
fetuses. In uncontrolled diabetes, marosomia occurs with the fetus weighing above 4000 grams. The

31
gestational diabetes increases maternal plasma glucose levels as well as insulin, stimulating fetal growth.

If the patient were to have had severely uncontrolled juvenile diabetes over time, intrauterine fetal growth
retardation may be associated with it due to poor vasculature. However, this is not the case from the
patient's history prior to conception.

Ketoacidosis (Choice B) is associated with Type 1 or 2 diabetes and can be life-threateneing. Fatty acids
are metabolized producing acidic ketone bodies due to a shortage of insulin. Symptoms include vomiting,
dehydration, confusion and dyspnea. This differs from gestational diabetes where there is no shortage of
insulin, just a resistance to it.

Hyperosmolar coma (Choice C) is a type 2 diabetic complication involving extreme hyperglycemic levels
without the presence of ketones (a byproduct of fat breakdown). The condition is seen where the blood has
a high concentration of sodium, and glucose, drawing water out of the body organs including the brain
thereby increasing dehydration and hyperglycemia.

Congenital anomalies (Choice D) would have been detected on her visit at 18 weeks or in the first
trimester.

27.

A 24 year old woman, gravida 3, para 2, presents with the chief complaint of some lower abdominal pain
accompanied by a small amount of vaginal bleeding. She is 16 weeks pregnant and has been healthy
throughout the pregnancy. She does not smoke cigarettes, drink alcohol or use illicit drugs. Abdominal
examination is normal. Pelvic examination reveals that the internal cervical os is closed.

Which of the following is the most likely diagnosis?

complete abortion

incomplete abortion

threatened abortion

inevitable abortion

missed abortion

Explanation

The correct answer is choice C

The definition of a Threatened abortion is bleeding within the first 20 weeks of pregnancy with an intact cervix having no effacem
dilatation. Only half of these will progress to pregnancy loss.

A complete abortion (Choice A) describes a total miscarriage or passing of the complete products of conception (including placenta
whereas passage of some of the conception products with retention of some tissue is considered an incomplete abortion (Choice B

In an inevitable abortion (Choice D), cramping and bleeding are accompanied by dilatation of the cervix. At this point there is no
for retention of the pregnancy and completing the abortion for the patient is the treatment of choice.

In a missed abortion (Choice E), the fetus dies but is retained in utero. Diagnosis is made on the basis of regression of pregnancy
of a fetal heart and ultrasound whereby there is no cardiac motion detected.

32
Table 1: Summary of the different types of abortion

28.

A 24 year old female G1P0 is admitted to hospital at 34 weeks gestation. The patient describes passing a
large volume of clear fluid per vaginal for 36 hours. Examination reveals the following: temperature 38.5
C, no uterine contractions, vague lower abdominal tenderness, fetal heart rate 185 bpm.

Lab investigations reveal: Hb 120, WBC 19.0, + nitrazine test of vaginal fluid, + ferning of vaginal fluid.

Which of the following is the most likely diagnosis?

premature rupture of membranes

chorioamnionitis

premature labour

acute pyelonephritis

acute vaginitis of pregnancy

A and B

A and B and C

All of the above are possibilities

33
Explanation

The correct answer is choice F

Rupture of membranes is evident and distinguished from involuntary urine loss by the positive ferning test
seen microscopically as well as the positive nitrazine test. Premature leakage of amniotic fluid may lead to
serious complications such as infection and preterm labor.The presence of amniotic fluid tends to elevate
the pH and detection of a pH increase may also aid in determining the amniotic fluid presence. Normal
Vaginal ph is 4.5-6.0 and ruptured vaginal pH is 7.1-7.3.

A fern test is an occurrence of amniotic fluid's protein and sodium chloride content displaying the presence
of fern-like patterns characteristic of crystals as the fluid dries on the microscope slide. One must ensure
not touching the cervical mucus when collecting the fluid which can alter the results.

Chorioamnionitis is seen by the abdominal tenderness and temperature elevation. The fetal tachycardia
reported is indicating fetal distress due to the difficulty a fetus has in coping with infections.

Chorioamnionitis is an inflammation of the fetal membranes (amnion and chorion) due to bacterial
infection, typically from ascending bacteria into the uterus from the vagina. The condition is diagnosed
clinically by maternal fever and uterine tenderness in the presence of confirmed premature rupture of
membranes. It can also be diagnosed histologically by examining the fetal membranes. Infiltration of the
chorionic plate by neutrophils is diagnostic. More severe chorioamnionitis may exhibit fetal membrane
necrosis and or abscess formation.

Premature rupture of membranes (Choice A) is evident as above but in and of itself would not cause the
fetal distress or temperature elevation.

Premature labour (Choice C) may occur subsequent to the membrane rupture but is not evident here by
the lack of contractions.

Acute pyelonephritis (Choice D) may cause ruptured membranes but the infection itself does not explain
the symptoms.

Acute vaginitis of pregnancy (Choice E) manifests in a discharge which may be thick or watery
accompanied by pruritus, swelling and discomfort. It does not cause fetal distress.

29.

A 19 year old primigravid woman is expecting her first child; she is 12 weeks pregnant by date. She has
vaginal bleeding and an enlarged for date uterus. In addition, no fetal heart sounds are heard. An
ultrasound is obtained.

The most likely diagnosis of this woman’s condition is which of the following:

sarcoma botryoides

tuberculous endometritis

adenocarcinoma of the uterus

hydatidiform mole

nomal pregnancy

Explanation

34
The correct answer is choice D

Molar pregnancy is an abnormal form of pregnancy, wherein a non-viable, fertilized egg implants in the
uterus, and thereby converts normal pregnancy processes into pathological ones. It is characterized by the
presence of a hydatidiform mole (or hydatid mole, mola hydatidosa). Molar pregnancies are categorized
into partial and complete moles. Complete moles have no identifiable embryonic or fetal tissues and arise
when an empty egg with no nucleus is fertilized by one (or occasionally two) normal sperm; the genotype
is typically 46,XX (due to subsequent mitosis of the fertilizing sperm) but can also be 46,XY or
23,X. Hydatidiform mole (trophoblastic disease) is a tumor that forms as a mass of cysts resembling a
bunch of grapes within the uterus. In its early stage, a hydatidiform mole make appear like a normal
pregnancy. However, it grows much faster than a pregnancy and so the uterus appears much larger than
gestational age would suggest the mole. The diagnosis consists of the following:

 lack of fetal movement


 pelvic examination
 ultrasound
 blood test for high levels of HCG

Molar pregnancies usually present with painless vaginal bleeding in the fourth to fifth month of pregnancy.
The uterus may be larger than expected, or the ovaries may be enlarged. There may also be more
vomiting than would be expected (hyperemesis). Sometimes there is an increase in blood pressure along
with protein in the urine. Blood tests will show very high levels of human chorionic gonadotropin (hCG).

Hydatidiform moles should be treated by evacuating the uterus by uterine suction or by surgical curettage
as soon as possible after diagnosis, in order to avoid the risks of choriocarcinoma. Patients are followed up
until their serum human chorionic gonadotrophin (hCG) level has fallen to an undetectable level. Invasive
or metastatic moles (cancer) may require chemotherapy and often respond well to methotrexate. The
response to treatment is nearly 100%. Patients are advised not to conceive for one year after a molar
pregnancy. The chances of having another molar pregnancy are approximately 1%.

Sarcoma botryoides (Choice A) also known as embryonal rhabdomyosarcoma, is a malignancy of the


vagina, cervix or bladder in infants and young children arising from embryonal rhabdomyoblasts. The
tumor resembles a bunch of grapes. Prognosis is usually good following conservative surgery and
chemotherapy.

With tuberculous endometritis (Choice B) a patient can present with abdominal pain, nausea, decreased
appetite and increased abdominal girth. However, because it is an inflammatory process, low-grade fever
and chills is not unusual. The white count may be found to be elevated.

Adenocarcinoma of the uterus (choice C) presents with abnormal vaginal discharge and abnormal bleeding
in 80% of the cases. Transvaginal ultrasound is useful in diagnosis.

With a normal pregnancy at 12 weeks or greater (choice E) one would have be able to hear a fetal heart.
If it was going to spontaneously abort, manifesting vaginal bleeding, it would not be a normal pregnancy.

30.

A 24-year-old female in seen for an annual examination. Her menstrual history is normal and all of her
previous Pap results have been normal. Her physical examination is normal but her Pap result returns with
low-grade squamous intraepithelial lesion (LSIL).

Which of the following is the most appropriate management option for this patient?

35
Repeat Pap in 6 months

HPV DNA typing

Endometrial biopsy

Colposcopy

Repeat Pap utilizing thin prep methodology

Explanation

The correct answer is choice A

A low-grade squamous intraepilthelial lesion is one in which the abnormal growth of squamous cells on the
surface of the cervix are not very abnormal or not much of the cervix is affected( probably from an HPV
infection). LSIL usually indicates mild dysplasia (CIN1)

CIN1 is the most common and benign type of cervical intraepithelial neoplasia. Since it usually resolves
spontaneously within two years, it can be managed with a "watch and wait" protocol.

In order to determine the extent or depth of the problem and because there is a 12-16% progression risk,
a colposcopy is performed to visualize the vagina and cervix with lighted magnification and biopsies are
performed during the procedure. The results of the colposcopy and biopsies will determine what if any
further action will be taken.

Although it is conceivable that the LSIL may go away without treatment, it may also progress and is
treatable so one would not simply ignore the condition for 6 months (choice A). Likewise one does not
need to repeat the smear for verification (choice E) since abnormal cells were already visualized on the
first cytological analysis.

This is in contrast to HSIL-high-grade squamous intraepithelial lesion which indicates moderate or severe
cervical intraepithelial neoplasia or carcinoma in situ, diagnosed witha pap smear. These lesions can lead
to invasive cervical cancer if not followed appropriately.

HSIL treatment involves the removal or destruction of the affected cells (usually by the LEEP method) or
by cryotherapy or laser ablation.

Endometrial biopsies are not needed (Choice C) at this time since this is a lesion of the cervical surface.

Table 1: Summary of Pap results and management recommendations

36
31.
A 29-year-old woman with a history of multiple sexual partners over the last 15 years has a routine
physical examination with no abnormal findings. On pelvic examination, the cervix shows no
abnormalities, but a Pap smear is taken and dysplastic cells (HSIL) are reported to be present. A
pregnancy test is negative. A cervical biopsy is performed and shows microscopic features of a minimal
dysplasia (CIN 1) involving the cervical squamous epithelium.

Which one of the following is an appropriate statement to make to the patient regarding these findings?

No further treatment is indicated

HPV testing and repeat testing in 3 months

Repeat Pap smear in 3-6 months

Complete excision of the lesion is recommended

Repeat Pap smear in one year

37
Explanation

The correct answer is Choice D.

Low grade cervical intraepithelial neoplasia (CIN 1) is caused by both oncogenic and nononcogenic human
papillomaviruses. CIN 1 is generally considered to be less worrisome when it is observed following low-
grade dysplastic changes on Pap smear than when it is observed following a Pap smear with HSIL.

Aggressive treatment is typically not warranted when CIN 1 is preceded by cervical cytology showing low
grade cytological lesions (ASC-US, ASC-H, LSIL). Many of these CIN 1 lesions spontaneously regress, and
progression to CIN 2,3 or invasive cancer is uncommon. On the other hand, when CIN 1 appears after
high-grade dysplasia (HSIL) has been confirmed on cervical cytology, there is 70% chance or greater that
these women will develop more serious lesions (CIN 2,3).

When HSIL is followed by a histologic diagnosis of CIN 1, as in this case, there remains the concern that
there may be an underlying high-grade lesion. An excisional diagnostic procedure has traditionally been
recommended for these situations. However, the American Society for Colposcopy and Cervical
Pathology (ASCCP) guidelines do allow for expectant management as an alternative approach, as long as
colposcopy was satisfactory and ECC was negative.

However, expectant management has its risks. Since women with CIN 1 preceded by HSIL are at higher
risk of progression to CIN 2,3 or having a missed underlying CIN 2,3 lesion than women who have CIN 1
preceded by low grade cytological abnormalities, monitoring of these women during expectant
management is more intensive. Both Pap smear and colposcopy are performed at 6 and 12 month intervals
if expectant management is chosen.

38
Table 2: Summary of Pap findings and Management options

39
32

A 24 year old primagravida has a normal pregnancy until 37 weeks gestation, at which time she notes
decreased fetal movement. She then has some cramping abdominal pain and she feels overall unwell.
When she sees you in her office, she presents with ruptured membranes with passage of foul-smelling
fluid. The fetus is stillborn a day later.

Which of the following organisms is most likely to be responsible for these findings:

Mycobacterium tuberculosis

Herpes simplex virus

Group B streptococcus

Treponema pallidum

Candida albicans

Explanation

The correct answer is choice C

Chorioamnionitis which is an infection of the chorion and amnion typically in the late 2nd trimester and
third trimester is due to an ascending bacterial infection in the birth canal, secondary to rupture of
membranes. Beta Streptococcus is found in the vagina of 15% of women. Other causes including E. Coli
and MRSA are becoming more and more prevalent. Chorioamnionitis is diagnosed clinically in the setting

40
of:

 Fever (an intrapartum temperature >100.4ºF or >37.8ºC)


 Significant maternal tachycardia (>120 beats per minute [bpm])
 Fetal tachycardia (>160-180 bpm)
 Purulent or foul-smelling amniotic fluid or vaginal discharge
 Uterine tenderness
 Maternal leukocytosis (total blood leukocyte count >15,000-18,000 cells/μL)

The use of penicillin rather than ampicillin as the chemotherapeutic agent to prevent GBS infections of the
newborn is encouraged. The effectiveness of erythromycin, clindamycin, and vancomycin for prevention of
neonatal GBS disease requires additional study. These antibiotics are often applied when a mother gives a
history of penicillin allergy. When acute chorioamnionitis is evident, delivery must be expedited.

Infants born to mothers with chorioamnionitis have unfavorable neurologic outcomes. Cerebral palsy (CP)
and cognitive impairment without CP have a relationship to the presence of maternal chorioamnionitis.

Mycobacterium tuberculosis (Choice A) causes TB with the and symptoms may go undetected during
pregnancy. This is unrelated to this case.

Herpes simplex virus (Choice B) causes painful blisters to the vagina and vulva. When the virus is positive
on the genitalia during labor, a Cesarean section is performed to prevent the neonatal eyes from direct
exposure which can lead to eventual blindness. Herpes is unrelated to chorioamnionitis.

Syphilis is caused by Treponema pallidum (Choice D). It is sexually transmitted and when contracted
during pregnancy causes congenital abnormalities, of which the most noted is microcephaly and neural
developmental problems.

Candida albicans (Choice E) is the most common fungus residing in the vaginal vault and is responsible for
the typical "yeast" vaginitis presenting with thick white cheesy discharge, inflammation of the vulva and
pruritis. It is harmless to the fetus. Bacterial vaginosis is associated with premature labor, although overt
infection of the neonate with microbes causing bacterial vaginosis is uncommon. Screening for and
treatment of bacterial vaginosis and other genital infections may prevent preterm birth, although recent
Cochrane reviews conflict regarding the effectiveness of therapy.

33.

A 23-year-old nulligravid woman visits her gynecologist for an annual examination. When questioned
regarding any unusual symptoms or concerns, she reports having an intermittent thin vaginal discharge
and continual vaginal irritation. She was recently treated for sinusitis with an antibiotic, although she does
not recall its name. She is sexually active and is taking oral contraceptives; she and her partner do not
always use condoms. She has been monogamous for the past 2 years, and recently they began practicing
mutual oral sex. Her medical and surgical history is otherwise noncontributory. She states that her
boyfriend has had no symptoms.

On examination, the patient’s vulva and vagina appear moderately inflamed, and she has a thin, gray,
odorless, purulent discharge. She has no cervical motion tenderness. Colpitis macularis ("strawberry-like"
spots on the cervix portion) are observed. Routine screening for chlamydia and gonorrhea are performed
with a Papanicolaou test. Microscopic examination of a wet preparation obtained during the test shows
small, flagellated organisms.

Which of the following treatments for the patient’s vaginal discharge is likely to be most effective and
acceptable to her?

41
Fluconazole 150 mg taken in a single oral dose

Metronidazole 2 g taken in a single oral dose

Metronidazole 500 mg taken orally twice daily for 7 days

Terconazole 0.8% cream (5 g) used intravaginally for 3 days

A and B

B and C

All of the above are true

Explanation

The correct answer is choice B

The infection described is Trichomonias, which is a sexually transmitted disease caused by a protozoan
found in the vagina and urethral tissues. Symptoms include:

 vaginal discharge-often malodorous, gray-green, frothy


 dysuria
 vaginal pruritus
 vaginal/vulvar inflammation with redness or excoriation

The diagnosis is made by wet mount. When cultures are performed on asymptomatic women, 10% are
found to be carriers of the vaginal trichomonads.

Metronidazole is the only approved drug for trichomoniasis therapy. A 2 gram single dose has been shown
to be as effective as 250 mg three times a day for 7 days. However, the treatment of choice is a single
dose of Metronidazole (Flagyl) which allows for more patient compliance than the pills taken three times a
day for 7 days (choice C) which is an alternative.

It is crucial that all sexual partners be treated as well as the patient, to prevent re-exposure.

Fluconazole 150 mg is is a triazole antifungal drug that works by slowing the growth of fungi and is used in
the treatment and prevention of both systemic and superficial fungal infections as those found not only in
the genital area, but also in the mouth, throat, esophagus, abdomen, lungs, blood and other organs. Also
known as diflucan, it is prescribed for Candida infections. Fluconazole is also used as prophylaxis of yeast
infections in those individuals who are likely to become infected due to immune compromise, like those being treated
with chemotherapy or radiation therapy before a bone marrow transplant.

Candida albicans causes nearly eighty percent of fungal vaginitis. Symptoms include itching and burning
with an increased discharge (usually white and curd-like). Diagnosis is made by identification of the fungus
by KOH preparations. Terconazole is also an antifungal agent and comes in both cream and suppository
forms to be administered in either 3 or 7 day therapies.

Chlamydia is usually asymptomatic in women but can lead to upper tract infections with tubal damage,
possibly leading to infertility. It can be cultured and once found, is treated with doxycycine 100 mg twice
daily for 7 days or azithromycin 1 g orally in a single dose.

Around 1 million new cases of gonorrhea occur annually. Most women are asymptomatic, causing delayed
diagnosis and treatment. Untreated gonorrhea can lead to pelvic inflammatory disease. Diagnosis is made
on the basis of cervical gram stain or endocervical culture. The sensitivity of the culture is
90%. Treatment regimens for gonorrhea include a single dose of:

42
 ceftriaxone 125 mg i.m. or
 cefixime 400 mg orally or

 ciprofloxacin 500 mg orally

34.

A 24-year-old primigravida presents in labour to the hospital. On examination the patient is found to have
an infection necessitating an emergency Cesarean section.

Which organism would be responsible for this operative intervention?

Mycobacterium tuberculosis

Herpes simplex virus

Escherichia coli

Treponema pallidum

Candida albicans

Explanation

The correct answer is choice B

Herpes simplex (Choice B) is a common sexually transmitted disease of the vagina and vulva. There are
two main kinds of HSVs:

 Type 1 which is associated with cold sores about the mouth and lips
 Type 2 which causes genital sores

Both types can infect the genital area. When found in the third trimester, a cesarean section should be
considered over a vaginal delivery so that the baby is not infected. If so, newborns develop skin or mouth
sores or eye infections and although most have no problem, serious permanent damage can occur to
nerves or the eyes.

E.coli can cause cystitis although it is also responsible for chorioamnionitis. Antibiotics are initiated and
then the patient can be allowed to progress into normal labor and is not a indication for an emergency C-
section. Symptoms include:

 fever
 uterine tenderness
 foul-smelling vaginal discharge
 maternal tachycardia
 fetal tachycardia or fetal compromise leading to death

Fetal complications if left untreatred include:

 preterm delivery
 infection (pneumonia, sepsis)
 cerebral palsy
 seizures

43
 death

Mycobacterium tuberculosis (Choice A) causes cough, lethargy, night sweats and fever, lymph node
enlargement of the neck, shortness of breath and chest pain in rare cases. Treatment should begin
immediately as the risk to the mother from TB is much greater than the possible risk of medication. The
organism can infect the neonate if the mother is untreated but it seldom affects the placenta or
chorion/amnion.

Treponema pallidum (Choice D) is responsible for syphilis and the effect in pregnancy depends when it was
contracted. Abortion early in pregnancy due to treponema is uncommon. Syphilis contracted prior to or at
conception usually causes midtrimester abortion or fetal death. When infection is during the pregnancy
and therapy is not given, the fetus can develop congenital symptoms including microcephaly, and
neurological impairment.

Candida albicans (Choice E) is a common fungus causing a yeast vaginitis or vulvitis and does not affect
the pregnancy.

35.

A 35 year old G3, P3 patient with Pap smear showing high grade squamous intraepithelial lesion (HSIL) of
the cervix (CIN III) has a negative colposcopy. A cone biopsy is performed that reveals a 6 mm focus of
squamous cells carcinoma that has invaded 1 mm beyond the epithelial basement membrane.
Histologically, no confluent tongues of tumor are seen, and there is no evidence of lymphatic or vascular
invasion. The margins of the cone biopsy specimen are free of disease.

Of the following answers, the most appropriate therapy of this suspicious lesion includes:

External beam radiation therapy

Radioactive cesium implantation

Total hysterectomy

Simple hysterectomy with pelvic lymphadenectomy

Total pelvic exenteration

Explanation

The correct answer is choice C.

The patient has cervical carcinoma, Stage T1a (FIGO Stage IA). Stage T1a tumors are recognizable only at
the microscopic level and have a horizontal spread of 7 mm or less and a depth of invasion that is 5 mm or
less. These early invasive cervical cancers are treated by total hysterectomy.

External beam radiation therapy (choice A) is typically used for more advanced-stage tumors (Stage II and
above).

Likewise, internal radiiation therapy or brachytherapy (choice B), is also used in more advanced-stage
tumors.

Hysterectomy with pelvic lymphadenectomy (choice C), is an appropriate treatment for Stage IB or IIA
tumors.

Total pelvic exenteration (choice E) is not an appropriate treatment for early invasive cervical cancer.

44
The FIGO (International Federation of Gynecology and Obstetrics) system and the American Joint
Committee on Cancer TNM staging system for the staging of cervical cancers are very similar. They both
classify cervical cancer on the basis of the extent of the tumor (T), whether the cancer has spread to
lymph nodes (N), and whether it has spread to distant sites (M):

 Stage 0 (Carcinoma in Situ): In stage 0, abnormal cells are found in the innermost lining of the
cervix. These abnormal cells may become cancer and spread into nearby normal tissue. Stage 0 is
also called carcinoma in situ.
 In stage I, cancer has formed and is found in the cervix only. Stage I is divided into stages IA and
IB, based on the amount of cancer that is found. In Stage IA, a very small amount of cancer that
can only be seen with a microscope is found in the tissues of the cervix. Stage IA is divided into
stages IA1 and IA2, based on the size of the tumor. In stage IA1, the cancer is not more than 3
millimeters deep and not more than 7 millimeters wide. In stage IA2, the cancer is more than 3
but not more than 5 millimeters deep, and not more than 7 millimeters wide. In stage IB, cancer
can only be seen with a microscope and is more than 5 millimeters deep or more than 7
millimeters wide, or can be seen without a microscope. Cancer that can be seen without a
microscope is divided into stages IB1 and IB2, based on the size of the tumor.

In stage IB1, the cancer can be seen without a microscope and is not larger than 4 centimeters.

In stage IB2, the cancer can be seen without a microscope and is larger than 4 centimeters.
 In stage II, cancer has spread beyond the cervix but not to the pelvic wall (the tissues that line
the part of the body between the hips) or to the lower third of the vagina. Stage II is divided into
stages IIA and IIB, based on how far the cancer has spread. In Stage IIA, cancer has spread
beyond the cervix to the upper two-thirds of the vagina but not to tissues around the uterus.

In Stage IIB, cancer has spread beyond the cervix to the upper two-thirds of the vagina and to
the tissues around the uterus.
 In stage III, cancer has spread to the lower third of the vagina, may have spread to the pelvic
wall, and/or has caused the kidney to stop working. Stage III is divided into stages IIIA and IIIB,
based on how far the cancer has spread. In Stage IIIA, cancer has spread to the lower third of the
vagina but not to the pelvic wall. In Stage IIIB, cancer has spread to the pelvic wall and/or the
tumor has become large enough to block the ureters (the tubes that connect the kidneys to the
bladder). This blockage can cause the kidneys to enlarge or stop working. Cancer cells may also
have spread to lymph nodes in the pelvis.

 In stage IV, cancer has spread to the bladder, rectum, or other parts of the body. Stage IV is
divided into stages IVA and IVB, based on where the cancer is found. In Stage IVA, cancer has
spread to the bladder or rectal wall and may have spread to lymph nodes in the pelvis. In Stage
IVB, cancer has spread beyond the pelvis and pelvic lymph nodes to other places in the body,
such as the abdomen, liver, intestinal tract, or lungs.

36.
A 28-year-old woman complains of several weeks' duration of pruritus and redness of the vaginal
introitus. She believes she has a "yeast infection." She is nulliparous, and medical history is significant for
type 2 diabetes mellitus of 4 years' duration, as well as severe sinusitis 8 weeks ago requiring a 2-week
course of amoxicillin/potassium clavulanate (Augmentin). Vaginal examination shows a thick, milky white
discharge with reddened, swollen labia. A wet mount reveals small oval yeast-like organisms.

Which of the following statements is true concerning this patient's condition?

45
The patient is suffering from a sexually-transmitted disease (STD).

The patient's diabetes is a risk factor for developing yeast vulvovaginitis.

The patient's condition occurs only in sexually active women.

Candida species are part of the normal vaginal flora in approximately 10% of healthy women.

Hormone therapy with estrogen decreases a woman's risk of developing a yeast infection.

Explanation

The correct answer is Choice B.

The patient is suffering from vulvovaginal candidiasis. Candida species are part of the lower genital tract
flora in 20 to 50 percent of healthy asymptomatic women. Colonization of the vagina with Candida is also
common in prepubescent girls. Several factors can upset the balance of flora in the vaginal area, leading to
overgrowth of Candida species.

Up to 75% of women report having had at least one episode of vaginal candidiasis. Vulvovaginal
candidiasis occurs most commonly when there are high estrogen levels, such as oral contraceptive use
(especially when estrogen dose is high), pregnancy, and hormone therapy with estrogen. Poory-controlled
diabetes mellitus also increases a woman's risk for developing vulvovaginal candidiasis.

Vulvovaginal candidiasis is not typically considered to be a sexually-transmitted disease since it occurs in


celibate women, and Candida is considered a part of the normal vaginal flora. However, sexual
transmission is likely possible.

Women with diabetes mellitus who have poor glycemic control are more prone to vulvovaginal candidiasis
than women with normal blood sugar levels. Recent broad-spectrum antibiotic use is also a risk factor for
vulvovaginal candidiasis.

Vulvovaginal candidiasis can generally be treated by topical antifungal medications, such as miconazole,
tioconazole, butoconazole, and clotrimazole.

37.

You are seeing a patient of yours in the follow-up gyneoncology clinic at the local university hospital. This
patient had undergone a laparotomy 2 weeks prior for a case of ovarian tumor and the histopathology
revealed a unilateral ovarian cancer with pelvic ascites fluid positive for malignant cells and in addition,
positive bilateral pelvic lymph nodes. All the other structures were free of disease.

What is the FIGO staging of the disease that this patient has:

stage IIc

stage IIIa

stage IIIb

stage IIIc

stage IV

Explanation

46
The correct answer is choice C

Stage IIIb is microscopic peritoneal implants outside of the pelvis with macroscopic peritoneal metastases
beyond the pelvis less than 2 cm in size.

The FIGO system is used to stage ovarian cancer and utilizes data obtained from surgery.

 Stage I: limited to one or both ovaries


 Stage II: pelvic extension or implants
 Stage IIa: extension or implants onto uterus or fallopian tubes with negative washings
 Stage IIb: extension or implants onto other pelvic structures
 Stage IIc (choice A): pelvic extension or implants with positive peritoneal washings
 Stage IIIa (choice B): microscopic peritoneal metastases beyond the pelvis
 Stage IIIc (choice D): peritoneal metastases beyond pelvis greater than 2 cm or lymph node
metastases

 Stage IV (choice E): distant metastases to the liver or outside the peritoneal cavity

38.

A 23 year old pregnant female is reasonably well and healthy with no chronic diseases is asking you about
any fetal risks after keeping a kitty at home. Those who have cats at home are more liable for certain cat
related infections than the general population.

All of the following diseases are associated with cat exposures during pregnancy except:

cat scratch disease

toxoplasmosis

tularaemia

hookworms

skin candidiasis

Explanation

The correct answer is choice E.

Cat-scratch disease (choice A), also called bartonellosis, is the most common zoonotic disease associated
with cats. Cat-scratch disease can occur when a person is bitten or scratched by an infected cat. Fleas may
also play a role in the transmission of infection. Within 3 to 10 days after a scratch, most patients develop
an erythematous, crusted papule (rarely, a pustule) at the scratch site. Regional lymphadenopathy
develops within 2 wk. The nodes are initially firm and tender, later becoming fluctuant, and may drain with
fistula formation. Fever, malaise, headache, and anorexia may accompany lymphadenopathy. However,
avoiding scratches and bites, controlling fleas, and keeping cats indoors all reduce the risk of cat-scratch
disease.

Toxoplasmosis (choice B) is caused by the parasitic protozoan Toxolasma gondii. People with weakened
immune systems, or infants whose mothers are infected during pregnancy, can develop severe illness.
People commonly become infected by eating undercooked or raw meat, or by inadvertently consuming
contaminated soil on unwashed or undercooked vegetables. In pregnancy, however, an active infection can

47
result on miscarriage and birth defects. According to the Council for Agricultural Science and Technology,
80% of children born with the disease who go untreated develop defects, ranging from mental impairment
to blindness.

Tularemia (choice C) is a rare infectious disease caused by the bacterium Francisella tularensis (a gram-
negative pleomorphic coccobacillus). The bacterium is found in cats that catch wild rabbits or rodents and
may also be spread to humans. Transmission occurs through contact with infected animals or there
habitats e.g. bites from infected insects or other animals, eating infected wild animals, contact with
contaminated water and soil. Symptoms can vary greatly depending on the method of infection. For
example infection through inhalation can cause symptoms similar to pneumonia, eating infected animals
can cause a sore throat and abdominal symptoms and transmission through the skin can cause result in a
painful skin ulcer.

Human hookworm infection (choice D) is a soil-transmitted helminth infection caused by the nematode
parasites Necator americanus and Ancylostoma duodenale. Puppies and kittens are especially likely to have
hookworm infections. Animals that are infected pass hookworm eggs in their stools. The eggs can hatch
into larvae, and both eggs and larvae may be found in dirt where animals have been. Eggs or larvae can
get into your body when you accidentally eat or have direct contact with contaminated dirt. Infective third-
stage larvae (L3) have the ability to penetrate through the skin, frequently entering the body through the
hands, feet, arms, or legs. Up to 44 million pregnant women are estimated to be infected with hookworm.
In pregnant women, anemia resulting from hookworm disease results in several adverse outcomes for both
the mother and her infant, including low birth weight, impaired milk production, and increased risk of death
for both the mother and the child.

39.

A 7 year old girl is seen by her pediatrician for left lower quadrant pain. After a thorough examination, you
decide to order a CT scan of the abdomen and pelvis. As a result of the finding on the CT, you consult
your oncology colleagues because an ovarian neoplasm is identified.

The most likely ovarian tumor in the patient includes which of the following:

germ cell

papillary serous epithelial

fibrosarcoma

Brenner

Sarcoma botryoides

Explanation

The correct answer is choice A

Germ cell tumors (also known as germinomas) come from the undifferentiated germ cell. It is histologically
identical with the seminoma of the tesis. The tumor is found in young adults and can be seen before the
age of 26. The dysgerminoma is bilateral in 10-20% of cases. Microscopically, it consists of large ovoid
cells separated by delicate septa of connective tissue with a sprinkling of lymphocytes. The tumor may
manufacture HCG.

Benign cystic teratomas (also known as dermoid cysts) make up 95% of germ cell tumors. Although
primarily ectoderm, the tissue may grow from any germ cell layers.

In females, germ cell tumors account for 30% of ovarian tumors, but only 1 to 3% of ovarian cancers in

48
North America. In younger women germ cell tumors are more common, thus in patients under the age of
21, 60% of ovarian tumors are of the germ cell type, and up to one-third are malignant.

Papillary serous epithelial tumors (Choice B) is a low grade cancer accounting for 10% of all ovarian cancer
cases. They can be unilocular or lobulated and multilocular withg papillary outgrowths from the wall and
small cavities filled with papillary growths.

Fibrosarcoma (Choice C) is either hard and nodular or ulcerative with interlacing bundles of elongated cells
with scant cytoplasm and extreme nuclear pleomorphism. It is seen as a malignancy of the vula (less
common than melanoma) or conversion from myomas of the uterus.

Brenner Tumor (Choice D) is usually unilateral accounting for 1-2% of all ovarian tumors. It is a solid
benign neoplasm with a very small likelihood of cancer and is usually found in women above the age of 40.
Histologically it consists of a fibrous stroma surrounding epitheloid cells and longitudinally grooved nuclei.

Sarcoma botyroides (Choice E) is a rare, highly malignant tumor involving the vagina and is ordinarily
found in infants. Grossly it appears as grape-like clusters, edematous red polyps filling and extruding from
the vagina. A bloody discharge is the most common presenting complaint. The tumor arises from the upper
vagina and the adjacent cervix from the growing tip of the mullerian duct.

40.

A 30-year-old G2 P1 woman presents to your office and has felt minimal fetal movement since 17 weeks
gestation. On examination the uterine size appears small for dates. An ultrasound scan shows growth
retardation. Amniocentesis is performed and chromosome analysis on fetal cells reveals a 47 XX, trisomy
13 karyotype.

Which of the following ultrasound findings is most likely to be present in this fetus with trisomy 13?

Oligohydramnios

Cystic hygroma of the neck

Postaxial polydactyly

Hepatomegaly

Neural tube defect

Explanation

The correct answer is choice C

Trisomy 13 syndrome, also known as Patau syndrome is associated with 50% infant death in the first
month. It is the third most common autosomal trisomy at birth (after 21 and 18).

Features include intrauterine growth restriction, facial, heart and limb anomalies. Typically the cranial
abnormality seen is holoprosencephaly with severe mental retardation for those infants surviving. Limb
deformities include postaxial polydactyly.

49
Cystic hygromas of the neck (Choice B) occurs with an abnormality of the lymphatic system in the fetus so
that a thin-walled cystic mass grows ( and can be larger than the actual head). There may be an absent or
inefficient connection between the lymphatic and venous system. Occasionally they resolve.

Oligohydramnios (Choice A) can occur with an intrauterine infection or renal agenesis of the fetus.

Fetal hepatosplenomegaly (Choice D) can be associated with Rh or ABO incompatibility (leading to


erythroblastosis fetalis).

41.

A woman comes to your office with her 12-year-old daughter seeking information regarding the human
papillomavirus (HPV) vaccination. Her daughter just began menstruating recently and has not been
sexually active. She is in good health and is no medications and has no known allergies.

Which one of the following statements is an American Cancer Society (ACS) recommendation regarding
the administration of this vaccine?

50
Routine HPV vaccination is recommended for girls within one year of menarche.

Women who have received the vaccine will not need to be screened with Pap smears as often as
unvaccinated women.

HPV vaccination is not currently recommended for women older than 26 years or for males.

HPV testing before initiating vaccination is recommended by the ACS.

HPV vaccination is recommended for both girls and boys.

Explanation

The correct answer is Choice C.

The Gardasil vaccine confers immunity against four common HPV types associated with the development
of cervical and anogenital carcinomas. Gardasil has received U.S. FDA approval for use in males and
females between 9 and 26 years of age and confers immunity against HPV types 6, 11, 16 and 18. Another
vaccine directed at HPV types 16 and 18, known as Cervarix, has been approved for use in females aged
10-15. These vaccines are used to prevent cancer that can result from an HPV infection before an
abnormal Pap test develops. They will not treat or protect against cancer from an existing HPV infection, so
vaccination at an age prior to the onset of sexual activity is recommended. Each vaccine requires a series
of 3 injections (shots) over a 6-month period.

HPV vaccines are estimated to protect against 70% of cervical cancers. Because the vaccine does not
protect against all HPV types, women who have been vaccinated should still undergo regular cervical
cancer (Pap) screening.

The American Cancer Society recommendations for the HPV vaccination are as follows:

 Routine HPV vaccination is recommended for girls 11 to 12 years old.


 Girls as young as age 9 can get HPV vaccination.
 HPV vaccination is also recommended for females 13 to 18 years old who have not started the
vaccines, or who have started but not completed the series.
 At this time there is not enough evidence to recommend for or against vaccinating every 19- to
26-year-old female. A decision about whether a woman aged 19 to 26 years should get the
vaccine should be based on an informed discussion between the woman and her health-care
provider. This discussion should take into account the woman's sexual history, including the
likelihood of previous HPV exposure and potential benefit from vaccination. The more sexual
partners a woman has had, the less likely the vaccine will be helpful.

 At this time vaccination is not recommended for women over age 26 or for males of any age.
(Gardasil has, however, been approved for use in males aged 9-26).

42.

An 17 year old patient consults you for evaluation of disabling pain with her menstrual periods, which has
been present since menarche at age 14. The pain is accompanied by nausea and headaches. Her medical
history is otherwise unremarkable, and pelvic examination is normal.

You diagnose primary dysmenorrhea and recommend intitial treatment with which of the following:

51
Ergot derivatives

Ibuprofen

GnRH analogs

Danazol

Codeine

Explanation

The correct answer is choice B.

Dysmenorrhea affects up to 50% of menstruating women and 90% of adolescent girls. In primary
dysmenorrhea, there is no underlying gynecologic problem causing the pain. Primary dysmenorrhea
can begin within six months to a year following menarche. In secondary dysmenorrhea, some underlying
abnormal gynecologic condition contributes to the menstrual pain. Secondary dysmenorrhea may be
evident at menarche but, more often, the condition develops later in life. The most common cause of
secondary dysmenorrhea is endometriosis, which accounts for the majority of cases.

Non-steroidal anti-inflammatory agents such as ibuprofen are the first-line treatment of primary
dysmenorrhea in young women.

Ergot derivatives (choice A) are most commonly used to treat severe migraine or cluster headaches.

GnRH analogs (choice C), have been successfully used in the management of endometriosis and other
conditions but are not an appropriate first-line treatment for primary dysmenorrhea.

Likewise, Danazol (choice D) is a modified testosterone that has been used to treat endometriosis and is
not appropriate as a treatment of primary dysmenorrhea.

Codeine (choice E) and other narcotic medications are not appropriate first-line treatments for
dysmenorrhea.

43.

A 31-year old female presents to the hospital in active labour with no prenatal care, but at term according
to dates supplied by her. She has a negative past medical history and this is her first pregnancy. The
patient also denies use of alcohol or tobacco.

A short time later, a male infant weighing 2200 gm is delivered. He is found to have microcephaly, with a
flat nasal bridge and palpebral fissures, epicanthal folds as well as tetralogy of Fallot and a high pitched
cry.

These characteristics are typical of which disorder?

Down Syndrome

Lejeune's syndrome

Edward''s Syndrome

Patau syndrome

Warkany syndrome 2

52
Explanation

The correct answer is choice B

The purpose of this questions wast to test you on the other well known genetic syndromes (choices A, C,
and D) that are a must know for the exams. Choice A and E are less common conditions but the other are
often tested.

Lejeune's syndrome, also known as Cri du chat syndrome or chromosome 5p deletion syndrome, is a rare
genetic disorder and gets its name from the typical cry of the affected infants due to larynx and nerve
problems. Common findings include:

 microcephaly
 growth retardation
 hypotonia
 hypertelorism
 epicanthal folds
 palpebral fissures
 strabismus
 flat nasal bridge
 micrognathia
 low-set ears
 short fingers
 single palmar creases
 cardiac defects (ventricular septal defect, atrial septal defect, patent ductus arteriosus, tetralogy
of Fallot)

Less often one may also see:

 cleft lip and palate


 thymic dysplasia
 intestinal malrottion
 inguinal hernia
 dislocated hips
 hypospadias
 cryptorchidism
 renal malformations (horseshoe kidneys, hydronephrosis, renal agenesis)
 syndactyly of the second and third fingers and toes

Down syndrome, trisomy 21, (choice A) is a genetic disorder that occurs one in every 800 live births and is
the leading cause of cognitive impairment. Characteristics include:

 flattened face and nose


 large protruding tongue
 small ears
 epicanthal folds
 Brushfield spots in iris

53
 short hands and short fingers with one pal crease
 widened iliac angle
 cardiac problems eg. tetralogy of Fallot

Edwards syndrome is a genetic disorder involving trisomy 18 with all or part of an extra 18th chromosome.
It is second only to Down syndrome as the most common autosomal trisomy. Symptoms include:

 microcephaly
 low-set,malformed ears
 micrognathia
 cleft lip/palate
 palpebral fissuers
 ocular hypertelorism
 ptosis
 choroid plexus cysts
 short sternum
 absent radius
 clubfoot or "rocker bottom feet
 undescended testicles in males.
 Occasionally accompanied by polyhydramnios

Patau syndrome is trisomy 13 and trisomy D whereby the additional 13th chromosome is due to a
nondisjunction of chromosomes during meiosis. The translocation disrupts the normal development causing
cardiac and renal defects. Additional symptoms include:

 microcephaly
 holoprosencephaly
 microphtalmia
 cataracts, retinal detachment
 polydactyly
 low-set ears
 rocker-bottom feet
 omphalocele
 cleft palate
 ventricular septal defect
 single umbilical artery

Warkany syndrome 2 is also known as trisomy 8. Complete trisomy 8 cases result in spontaneous
abortions. Symptoms of mosaic trisomy 8 include:

 moderate to severe mental retardation


 abnormally short or tall stature

 eye abnormalities

44.

A 26-year-old woman presents to your office complaining of a foul-smelling vaginal discharge, which
started 1 week ago. She has had multiple sexual partners and does not use condoms or any form of birth
control. Physical examination shows no signs of ulcers, lesions, or rash on her vulva. Speculum
examination reveals a gray, adherent, foul-smelling discharge on the walls of the vaginal vault.
Microscopy demonstrates multiple white blood cells (WBCs) as well as clue cells.

What is the most effective treatment for this infection?

54
Metronidazole (Flagyl)

Ceftriaxone (Rocephin)

Doxycycline (eg, Adoxa, Doryx, Vibramycin)

Trimethoprim-sulfamethoxazole ([TMP-SMX] Bactrim, Cotrim, Septra)

Ketoconazole (Nizoral)

Explanation

The correct answer is Choice A.

The patient is suffering from bacterial vaginosis. The presence of clue cells seen on a microscopic smear is
a diagnostic sign of bacterial vaginosis. Clue cells are vaginal cells covered with bacteria.

In the past, bacterial vaginosis was referred to as Gardnerella vaginitis, named after the bacteria that were
thought to cause the condition. However, the newer name, bacterial vaginosis, reflects the fact that there
are a number of species of bacteria that naturally live in the vaginal area and may proliferate under certain
conditions. The Gardnerella organism is not the only type of bacteria causing the symptoms.

Any woman with an unusual vaginal discharge should be evaluated so that more serious infections such as
chlamydia and gonorrhea can be excluded. Symptoms of bacterial vaginosis may also mimic those of yeast
infections of the vagina and trichomoniasis, and these conditions must also be excluded in women with
vaginal symptoms.

Bacterial vaginosis is a common condition. Studies have shown that approximately 29% of women in the
US are affected. Bacterial vaginosis is found in about 16% of pregnant women and approximately 60% of
women who have a sexually-transmitted disease (STD).

Antibiotic therapy is indicated for bacterial vaginosis. A few antibiotics are routinely used. Metronidazole,
administered either orally or vaginally, is an effective cure. Oral clindamycin or vaginal clindamycin cream
is also effective in treating bacterial vaginosis. Tinidazole has fewer side effects than metronidazole and is
also an appropriate antibiotic choice.

45.

A 38 year-old woman presenting with abdominal pain, nausea, vomiting, and muscle pain in the lower
extremities is diagnosed with an acute appendicitis. When pathology reveals a normal appendix and
symptoms do not resolve, she is given an pregnancy test, which comes out negative.

All of the following conditions may mimic an acute appendicitis except:

familial Mediterranean fever

lead poisoning

Crohn disease

ectopic pregnancy

all of the above are true

Explanation

55
The correct answer is choice E.

While the diagnosis of appendicitis often is straight forward, at times it can be tricky. For this reason,
medical literature is lush with reports of healthy appendices being removed, leading to a more careful
patient workup, elucidating a condition that can mimic an inflamed appendix.

Familial Mediterranean fever (FMF, choice A), as this condition often manifests as episodes of peritonitis,
often mimicking acute appendicitis. FMF also includes a component of arthralgia (joint paint), but this may
not be present in all attacks. For this reason, ironically, some experts suggest that elective appendectomy
should be performed on patients with FMF during periods between FMF flareups, so that acute appendicitis
can be eliminated from the differential diagnosis during all subsequent FMF attacks.

FMF is an autosomal recessive condition that appears in people who are homozygous for an MEFV gene, of
which several are known. Some MEFV genes are common in Arab people, occurring with in an estimated
1/50 Arabs, resulting in an FMF incidence of 1/2600 in many Arabic-speaking countries. FMF is even more
common in Turkey with an incidence of 1/1000, partly because the incidence is extremely high among
Armenians, with a gene frequency of 1 in 7 people and a disease incidence of 1/500. Almost as high is the
incidence among Sephardic Jews. Descended from Jews expelled from Spain and Portugal who migrated to
North Africa, but also to Holland and southeast Europe, including Turkey, Sephardic Jews have an
estimated FMF incidence of 1/250-1,000; thus, the gene occurs in 1 out of every 8-16 Sephardic Jews.
While Ashkenazic Jews (which make up most Jews in the US and Canada) previously were thought to have
a fairly low incidence of FMF genes, studies have revealed a very high incidence, 1/5, of a particular FMF
gene, but interestingly it has resulted in a disease incidence of only 1/73,000 suggesting a low penetrance
for this particular gene.

While lead poisoning (choice B) is a potential issue in pediatrics and while children are to be screened for
lead levels, the condition occurs in adults as well. Historians have found evidence of symptoms in people
dating back many centuries, one example being ancient Rome, which is known to have used lead in the
aqueducts carrying water.

Today, many different industrial products contain lead, including jet engines of aircraft. Others that you
may encounter in test questions may include battery workers, automobile mechanics, welders, glass
manufacturers, and plumbers. Those at risk are the people who work with the products. However, people
also can be exposed through such behaviors as eating from lead plates or drinking frequently from glasses
decorated with lead paint. Symptoms of lead poisoning include abdominal pain and lack of appetite, but
also irritability, sleep disorders, headaches, and constipation.

Usually, Crohn disease (choice C) causes chronic symptoms. However, one out of three patients with
ileocecal Crohn disease present the first time with abdominal pain so acute that they can be misdiagnosed
with appendicitis.

Ectopic pregnancy (choice D) is condition in which implantation of a fertilized ovum takes place outside of
the uterine endometrium. Often this occurs in a fallopian tube. While the highest risk group for this is in
the age range of 35-44 years, it can happen during adolescence as well. Classically, ectopic pregnancy
presents with pain, amenorrhea, and vaginal bleeding, but only half of patients present in this way. For this
reason, for any woman in her child-bearing years with amenorrhea and abdominal pain, ectopic pregnancy
must remain in the differential diagnosis until a negative pregnancy test takes it off the list. In the event of
an ectopic pregnancy, if it occurs in the right tube, the presentation can be very similar to that of
appendicitis.

Typically, patients with appendicitis present with an elevated white blood cell count; for this reason a
complete blood cell count (CBC) is warranted. A CBC contains several components only one of which is the
white cell count but normally white blood cell count is in the range of 4,500-10,000 cells/ul. Typically, in
appendicitis, the count is 11,000 cells/ul or higher. Because of the inflammatory response, C-reactive
protein also is elevated in acute appendicitis, but diagnosis still can be confused. Ultrasonography is
extremely useful, however, as it can reveal an inflamed appendix as well as a gestational sac in a fallopian
tube, or elsewhere.

46.

56
This is the gross appearance of placenta from a 31 year old female in the second trimester of pregnancy.
A malformed fetus with marked intrauterine growth retardation was present.

Cytogenetic analysis of this tissue is most likely to yield which of the following karyotype:

47, XXY

69, XXY

46, XX

47, XY, +18

23 X Click on image to Zoom

Explanation

The correct answer is choice B

The karyotype 69 XXY represents triploidy. Synonyms include incomplete molar gestation and partial
triploid mole. It is a focal hydropic chorionic villi swelling with trophoblastic hyperplasia and some fetal
tissue or embryonic tissue is seen. Associated anomalies include involvement of the heart, CNS, lungs,
skeleton and genitalia and all have intra-uterine growth retardation. Most die in utero during the first or
early second trimester.

47XXY (Choice A) is seen in Klinefelter's syndrome where a male fetus has an extra X sex chromosome.
Associated symptoms include hypothyroidism, infertility, testicular cancer and an increased risk for male
breast cancer. Symptoms include testosterone deficiency and gynecomastia with sparce facial and body
hair.

46XX (Choice C) is the normal chromosomal complement for females.

47 XY + 18 (Choice D) represents chromosomal mosaicism with trisomy 18. This is Edwards syndrome, a
genetic disorder caused by a meiotic nondisjunction event displaying kidney malformations, structural
heart defects (like septal defects) growth deficiency microcephaly, micrognathia, cleft lip and cleft palate
along with ocular hypertelorism (wide spread eyes), ptosis (drooping of the upper eyelids, clenched hands.

23 X (Choice E) is half of the chromosomal complement, and can also be referring to X-inactivation.

47.

A 19-year old woman in her second trimester, presents by ambulance stretcher to the emergency room.
She has a negative medical history other than a history of tobacco usage and has been depressed recently
over the breakup of her relationship with the baby's father. Examination reveals slurred speech,
hypotension and delirium. A fetal heart tracing on the monitor reveals a lack of variability.

Which drug do you suspect that this patient has taken?

57
Cocaine

Amphetamines

Marijuana

Heroin

Diazepam

Explanation

The correct answer is choice E

Diazepam is in the class of CNS sedatives (benzodiazepines) along with barbiturates, methqualone, chlordiazepoxide,
alprazolam, and flurazepam. Category X benzodiazepines include: Flurazepam, Estazolam, Temazepam, Quazepam and
Triazolam. Maternal symptoms include:

 fixed pupils
 hypotension
 respiratory depression
 hypoactive reflexes
 drowsiness
 coma
 lateral nystagmus
 delerium
 slurred speech
 convulsions

Fetal symptoms include:

 decreased heart variability


 sedation
 respiratory depression (mild if present)

Cocaine (Choice A) is a CNS stimulant along with amphetamines (Choice B). This causes shallow breaths in the mother along
with hypertension, hyperactive reflexes and pupils that are dilated but reactive. Fetal symptoms include:

 hyperactivity with increased movement


 spontaneous abortion
 premature labor
 fetal demise
 placental abruption
 IUGR
 SIDS
 neurobehavioral impairment
 limb reduction defects
 possible genitourinary malformations.

Marijuana (Choice C) does has been reported to cause miscarriages, Intrauterine growth restriction (IUGR), abruptio placentae,
preterm delivery, and neurobehavioural abnormalities. Maternal effect consists of normal pupils with inflamed conjunctivae.

Heroine (Choice D) is in a class of narcotics along with codeine, morphine and opium. Reported maternal affects include:

58
 constricted pupils
 decreased respirations
 hypotension
 hypoactive reflexes
 sensorium obtunded

Intrauterine fetal demise has been associated with this along with IUGR and increased fetal activity from withdrawal. Other
Perinatal complications following opiate misuse in pregnancy include:

 Miscarriage
 Placental abruption
 Premature rupture of membranes/labour and delivery
 Chorioamnionitis
 Intrauterine growth restriction (IUGR) Low birthweight
 Pre-eclampsia (PET)
 Intrauterine death (IUD)
 Postpartum haemorrhage (PPH)
 Respiratory distress syndrome (RDS)
 Neonatal abstinence syndrome (NAS)
 Neonatal mortality
 Neurocognitive abnormalities

Figure 1: Review of Pregnancy-Drug Categories

48.

A 67-year-old Caucasian woman who had been in good health until approximately 6 months ago presents

59
to her primary care physician with increased abdominal discomfort, persistent “fullness,” and early satiety.
Over the past 2 months, the patient has had gradual worsening of these complaints, and she has
developed progressive abdominal distention to the extent that her slacks no longer fit. The patient denies
significant past medical or surgical diseases. She underwent menopause at age 55 years, and she has no
past history of disorders of pregnancy or gynecologic diseases. The patient’s mother had ovarian cancer,
and one of the patient’s aunts also had a gynecologic malignancy. The patient is married, has 2
daughters, and is a smoker (1 pack/day).

Examination of the abdomen shows obvious abdominal distention. A fluid wave is present, with shifting
dullness. No masses are appreciated. Recto-vaginal examination demonstrates a large, firm mass in the
right adnexa and nodularity in the cul-de-sac. The limits of the mass cannot be fixed. A chest radiograph
shows a small, right-sided pleural effusion. A computed tomography (CT) scan reveals a 12-cm mass of
heterogeneous texture in the right ovary. A large number of ascites is present. The patient is informed
that she most likely has an ovarian malignancy.

What type of ovarian cancer is most common in this patient’s age-group?

Germ cell tumor

Endodermal sinus tumor

Epithelial tumor

Mature teratoma

Extraovarian peritoneal carcinoma

Explanation

The correct answer is choice C

The greatest risk factor for epithelial ovarian cancer, which is the leading cause of death from gynecological
malignancies, is advancing age. There is also a genetic predisposition associated with BRCA1 and BRCA2
in 10% of the cases.

Table 1: Important risk factors for Ovarian cancer

60
Dysgerminomas (Choice A) are types of germ cell tumors, usually malignant and extremely rare after the
age of 50. It usually occurs in adolescence and early adult life.

Endodermal sinus tumor (Choice B) also known as yolk sac tumor is a germ cell tumor. It is the most
common testicular tumor in children under 3 and is also known as infantile embryonal carcinoma.

Mature teratoma (Choice D) is in the germ cell tumor category and is the most common ovarian neoplasm
in patients younger than 20 years.

49.

A 63-year-old woman presents with history of postmenopausal vaginal bleeding for 2 weeks. She has not
been on hormone replacement therapy. She is a known diabetic with a Glycated hemoglobin of 7%. She is
also hypertensive maintained on a combination drug, ACE inhibitor and Calcium channel blocker. Her
family history is positive for colon cancer and she had had already two negative colonoscopies in the past
10 years. On physical examination she is found to have a weight of 170 lbs (77.2 kg) and 58 inches
(147.3 cm) tall. Her BMI is 35.6 kg/m2. Her blood pressure is 130/80. The pulse is 65 regular. She is
afebrile. The general physical examination is negative.

Which of the following courses of action is not appropriate at this time:

61
Obtain fractional D and C where 4 quadrant cervical biopsies are performed and curettage of the
endocervical canal.

Perform a total abdominal hysterectomy

Perform endometrial biopsy and fluid hysteroscopy

Hysteroscopy and directly visualized endometrial biopsies

Serum Thyroid function screen

Explanation

The correct answer is choice B.

One would not perform a hysterectomy on someone without a good cause. Currently a workup was not
performed and her bleeding may be of benign origin. A medical cause for abnormal bleeding for instance
is abnormal thyroid functioning.

A fractional curettage (choice A) is a method whereby each uterine section is examined and scraped to get
samples from all parts of the uterus. It is the original diagnostic technique to rule out emdometrial cancer
and is now often replaced by four quadrant endometrial biopsies. The sampling of cells from the
endocervical canal, is separate from the endometrial cavity. The purpose is to decipher where the origin of
any abnormal cells obtained, arose.

Endometrial biopsies (Choice C and D) are performed to get a good sampling of various areas of the
uterine canal to ensure that no abnormal areas are being missed. Hysteroscopy is performed by inserting a
filling medium into the uterus, held by a cervical cap and using direct visualization.

For diagnosing endometrial pathology, hysteroscopy is more accurate than ultrasound for diagnosing
endometrial hyperplasia, endometrial pollyps or submucous myomas. Abnormal areas are then biopsied
under direct visualization.

Serum thyroid Studies (Choice E) would reflect an abnormality in thyroid function which can manifest itself
in abnormal uterine bleeding.

50.

A 21 year old nulligravid woman presents for a routine gynecological exam with irregular menstrual
periods since menarche at age 13 years. She does not experience premenstrual symptoms, and has never
used contraception, despite being sexually active with males since age 16. She has noted increased hair
growth on her face and lower abdomen. On pelvic examination, there is copious cervical mucus and
slightly enlarged irregular ovaries. External genital exam is unremarkable. She is normotensive and has a
normal body mass index (BMI) of 24.

Which of the following is the most likely cause of these findings?

congenital adrenal hyperplasia

ovarian hyperthecosis

androgen producing neoplasm

cushing's syndrome

polycystic ovary syndrome

62
Explanation

The correct answer is Choice E.

Polycystic ovary syndrome (PCOS), with a reported prevalance in the US approaching 8%, may be the
most common reproductive endocrinopathy of women during childbearing years. Symptoms are due to
excess production of androgens by the ovaries, leading to chronic anovulation, characterized by
oligomenorrhea or amenorrhea. Periods tend to be characterized by lack of premenstrual symptoms, due
to this lack of ovulation. Hirsutism may be present involving the face and pubic hair in the classic diamond
shape pattern of growth, as well as potential acne and temporal balding. Virilization and clitormegaly are
not typically associated with PCOS. Insulin resistance is common with all body types and there is
associated obesity in about 50% of cases. Acanthosis nigricans usually accompanies cases with associated
obesity. Diagnosis is generally made based on clinical features, however testing may be indicated to rule
out other differentials. The choices below represent these differentials, all of which are relatively rare
diagnoses, but may resemble PCOS symptoms.

Choice A: Congenital Adrenal Hyperplasia (CAH) is an inherited disorder of adrenal steroidogenesis, the
nonclassic or adult-onset form may mimic certain features of PCOS, due to the associated
hyperandrogenism of CAH-21-hydroxylase deficiency. The severe hirsutism and virilization, shorter
stature, and associated hypertension can distinguish CAH from PCOS.

Choice B: Ovarian Hyperthecosis, is due to formation of luteinized theca cells in the ovarian stroma, which
cause high serum androgens. Like CAH, this condition is associated with a more severe hirsuitism,
including clitoromegaly, balding, deepining voice, and male body type. Like PCOS, there is usually insulin
resistance, and a large percentage of women with this condition are obese with acanthosis nigricans.

Choice C: Androgen producing neoplasms may be associated with the ovary or adrenal gland. Like ovarian
hyperthecosis and CAH, these lesions may induce severe hirsutism, virilization, and male body shape and
voice. Early on, the symptoms may be in a milder form, resembling the characteristics of PCOS,
however these neoplasms tend to cause rapid progression of androgen-induced changes, which distinguish
it from PCOS, a more gradual onset presentation.

Choice D: Cushings Syndrome characteristics result from excessive cortisol production usually from an
adrenal neoplasm or pituitary adenoma producing ACTH. It may also be induced by excess exogenous
cortisol consumption (ie Prednisone). The associated hirsuitism, acne, and oligomenorrhea resemble PCOS,
however other typical features of Cushing's syndrome include moon facies, buffalo hump, hypertension,
abdominal striae, and osteoporosis are the result of excessive cortisol.

51.

A 35-year-old woman has had dull pelvic pain for 4 months. A pelvic examination reveals a normal
appearing cervix, a normal sized uterus, and a large tender right adnexal mass. Ultrasound reveals an 8
cm cystic, fluid-filled mass involving the right adnexal region. At laparotomy, there are many filmy fibrous
adhesions in the pelvis. The mass is excised and on gross examination is found to have a thin wall and is
filled with purulent exudate. Microscopically, there are thin remnants of fallopian tube and ovary
comprising the wall of the mass, with numerous neutrophils filling the lumen.

All of the following statements concerning this condition are true EXCEPT:

63
Surgical management may be indicated.

The condition can occur as a complication of pelvic surgery.

Ultrasound appearance can generally establish the diagnosis.

A mixed bacterial infection most likely preceded the development of this condition.

Minimally-invasive drainage techniques are typically not effective in management of this


condition.

Explanation

The correct answer is Choice E.

The patient's condition and pathologic findings are consistent with a tubo-ovarian abscess (TOA), which
most often arises as a complication of pelvic inflammatory disease (PID), although it may on occasion
develop after pelvic surgery. As with this patient, abdominal and/or pelvic pain is a characteristic symptom
of TOA. Fever and leukocytosis may also be present but are not always observed.

A palpable adnexal mass is appreciable in most cases of TOA, although they may also be found
incidentally.

The microorganisms responsible for TOAs are those implicated in the pathogenesis of PID; as such, N.
gonorrheae is the most likely choice. As in PID, mixed bacterial infections are common, with a high
percentage of anaerobic bacteria.

Ultrasound examination is the preferred diagnostic modality to confirm presence of a TOA. The classical
sonographic appearance of a TOA is one or more relatively homogeneous, cystic, thin-walled, well-
demarcated mass(es), which are usually contiguous. Air-fluid levels and septations may also be
appreciated in some cases.

An absolute standard of care for TOAs has not been defined; both immediate surgical management as well
as inpatient conservative management with broad-spectrum antibiotic coverage with careful monitoring are
alternatives, depending upon the individual patient and exact clinical presentation. Minimally-invasive
drainage along with broad spectrum antibiotic coverage is also an acceptable option.

52.

A 26 year old prinigravida presents at 40 weeks in active labour with contractions every two minutes. She
is diagnosed as having transverse lie with the back up. Her pregnancy prior to her presentation has been
otherwise unremarkable.

Which of the following would be the most appropriate next step?

start vasodilan (isoxprine)

perform an external version

prepare for an immediate caesarean section

rupture membranes and then perform an internal version

none of the above

Explanation

64
The correct answer is choice C

When the patient is in labour and the transverse lie is with the back up, there is a risk of prolapsed cord.
Therefore, rather than manipulation, the patient should be prepared for a Cesarean Section right away.
However, if the patient is fully dilated and the membranes are intact, one can do a internal podalic version.
This should never be done if the cervix is not completely dilated. Some predisposing factors for a
transverse lie include:

 Grand multiparity - more than 5 term pregnancies


 Placenta previa
 Bony abnormalities of the pelvis
 Pelvic kidney
 Other pelvic mass

The definition of fetal presentation means the part of the fetus that is "presenting" at the cervix:

 Cephalic presentation means head first. This is the normal presentation.


 Breech presentation means the fetal buttock is coming out first.
 Transverse lie means the fetus is oriented from one side of the mother to the other and neither
the head nor the buttock is coming out first.
 Compound presentation means that a fetal hand is coming out with the fetal head.
 Shoulder presentation means that the fetal shoulder is trying to come out first.

Fetal "presentation" is different from fetal "position." Fetal position refers to the orientation of the fetus
within the birth canal (eg, looking toward the mother's pubic bone (OP), or look toward the mother's
coccyx (OA).

Vasodilan (Choice A) is a drug which relaxes blood vessels and dilates them to allow blood flow with less
resistance. Conditions for its use are arteriosclerosis, Raynaud's phenomenon and cerebral vascular
insufficiency.

One would not rupture the membranes (choice D) as the risk is high in this position for prolapsed cord.

53.

A 50-year old Caucasian woman presents to your office with complex endometrial hyperplasia.

Which of the following are you not likely to tell her regarding her prognosis and management?

Immediate hysterectomy is needed to ensure a favourable prognosis

Patient will need ongoing progestin therapy for several years

Her risk for endometrial cancer is greater than if she had atypical hyperplasia

A 3-month trial of progestin with monitoring for recurrence is safe

A fractional dilatation and curettage is advised

Explanation

65
The correct answer is choice D

A 3-month trial of progestin with strict monitoring for recurrence is not completely safe.

Endometrial hyperplasia is a state where there is excessive proliferation of endometrial cells, usually from
high estrogen levels combined with insufficient progesterone levels to counteract the proliferation on the
tissue. Endometrial hyperplasia is a significant risk factor for endometrial cancer.

There are two types of hyperplastic disorders.

1. Endometrial hyperplasia (simple or complex)- irregularity and cystic expansion of glands or


crowding and budding of glands (complex)
2. Atypical endometrial hyperplasia (simple or complex)-simple or complex architectural changes
with atypical changes in gland cells including cell sratification, tufting, loss of nuclear polarity,
enlarged nuclei and increase in mitotitc activity

The actual diagnosis is made by a fractional D&C. However, the risk for endometrial cancer is greater with
atypical hyperplasia rather than complex endometrial hyperplasia.

Whether women with endometrial hyperplasia need ongoing progestin therapy for several years is still
unknown.

Micrograph displaying endometrial hyperplasia with gland-to stroma ratio preerved but the glands have an
irregular shape and/or are dilated.

54.

A 35-year-old woman in her 18th week of gestation presents to your office with a blood pressure of
165/110 mm Hg. She has a known medical history of chronic hypertension and her obstetrician instructed
her to stop her antihypertensive treatment at her first few weeks of gestations as her blood pressure
dropped to a normal level. She feels otherwise healthy with no cramping, spotting, abdominal pain or
headaches.

Which of the following antihypertensive agents would be most suitable for initial therapy in this patient?

Fosinopril

Valsartan

Bumetanide

Methyldopa

Hydrochlorothiazide

66
Explanation

The correct answer is choice D.

Hypertensive disorders during pregnancy are classified into 4 categories, as recommended by the National
High Blood Pressure Education Program Working Group on High Blood Pressure in Pregnancy:

 Chronic hypertension (blood pressure exceeding 140/90 mm Hg before pregnancy or before 20


weeks' gestation), as in this case.
 Preeclampsia-eclampsia (new onset of elevated blood pressure readings after 20 weeks'
gestation).
 Preeclampsia superimposed on chronic hypertension
 Gestational hypertension (transient hypertension of pregnancy or chronic hypertension identified
in the latter half of pregnancy).

In normal pregnancy, women's mean arterial pressure drops 10-15 mm Hg over the first half of pregnancy.
Accordingly, most women with mild chronic hypertension may not require any medication during this
period. Conversely, DBP greater than 110 mm Hg has been associated with an increased risk of placental
abruption and intrauterine growth restriction, and SBP greater than 160 mm Hg increases the risk of
maternal intracerebral hemorrhage.

Therefore, pregnant patients should be started on antihypertensive therapy if the SBP is greater than 160
mm Hg or the DBP is greater than 100-105 mmHg (as in this case).

Methyldopa (choice D) is a centrally acting antihypertensive drug. It is converted intraneuronally to a-


methylnorepinephrine, an alpha-2 adrenergic agonist, which acts by decrease in sympathetic outflow from
CNS. It is selected by the American Association of Clinical Endocrinologists as the first-line agent for
treatment of hypertension during pregnancy as it not associated with the development of teratogenic or
other fetal abnormalities.

Angiotensin-converting enzyme (ACE) inhibitors, as fosinopril (choice A) should be avoided during


pregnancy, as they are associated with fetal renal dysgenesis or death when used in the second and third
trimesters and with increased risk of cardiovascular and central nervous system malformations when used
in the first trimester.

Angiotensin II receptor antagonists, such as valsartan (choice B) are not used during pregnancy because
they have a mechanism of action similar to that of ACE inhibitors.

Loop diuretics, as bumetanide (choice C) and thiazide diuretics, as hydrochlorothiazide (choice E) are
avoided in pregnancy, as they prevent the physiologic volume expansion seen in normal pregnancy and
may reduce uterine blood flow. They may be used in states of volume-dependent hypertension, such as
renal or cardiac disease.

55.

A 24 year old primigravid woman, woman is intent on breast feeding, decides upon a home delivery.
Immediately after the birth of a 4.1 kg, (9 lbs) infant, the patient bleeds massively from extensive vaginal
and cervical lacerations. She is brought to the nearest hospital in shock. Over 2 h, 9 units of blood are
transfused, and the patient’s blood pressure returns to a reasonable level. A haemoglobin value the next

67
day is 7.5 g/dL, and 3 units of packed red blood cells are given.

Development of the sequela could be evident as early as how long post-partum:

6 h postpartum

1 wk postpartum

2 weeks postpartum

6 mo postpartum

1 year postpartum

Explanation

The correct answer is choice B

The most common cause of this bleeding episode is Sheehan's Syndrome with pituitary necrosis. When
the patient tried to breast feed one week postpartum, she would discover that she failed to
lactate. Sheehan syndrome or necrosis of the pituitary gland, also known as postpartum hypopituitarism, is
a rare compication of postpartum hemorrhage. Women then develop hypopituitarism with faiure to lactate
or difficulties with lactaion. Many women also develop amenorrhea or oligomenorrhea after delivery. This
may not be diagnosed at the postpartum period and the diagnosis made years later when symptoms show
us, such as secondary hypothyroidism or secondary adrenal insufficiency, in a woman who had a
postpartum hemorrhage. In some cases, a woman with Sheehan syndrome might be relatively
asymptomatic, and the diagnosis is not made until years later, with features of hypopituitarism.
Uncommonly, Sheehan's syndrome may also appear acutely after delivery, mainly by hyponatremia.

The etiology of this condition is believed to a result of the pituitary gland physiologically being enlarged in
pregnancy and becomes sensitive to the decreased blood flow caused by massive hemorrhage and
hypovolemic shock. Women with Sheehan syndrome have varying degrees of hypopituitarism, ranging
from panhypopituitarism to only selective pituitary deficiencies. The anterior pituitary is more susceptible
to damage than the posterior pituitary. Treatment of young women with hypopituitarism usually includes
replacement of hydrocortisone first and then replacement of thyroid hormone and estrogen with or without
progesterone depending on whether she has a uterus. Both thyroxine replacement and gonadotropin
replacement are common, and doses are titrated to each individual. Replacement of growth hormone is
necessary in children with hypopituitarism but is controversial in adults.

Colostrum is secreted for the first two days postpartum and then lactation begins.

The mammary ducts fill with milk between the 2nd and 5th days postpartum. Suckling and oxytocin
release evoke the let down of milk.

56.

A 30 year old G0 previously healthy female has a routine Pap smear taken, and the report indicates
that high-grade dysplastic cells (HSIL) are present. This is her first abnormal Pap smear, and a pregnancy
test is negative.

Which of the following historical findings is most likely to be discovered by interview of this patient:

68
Family history of breast and ovarian carcinomas

Use of oral contraceptives

Recurrent vaginal Candida infections

Diethylstilbesterol exposure in utero

Multiple sexual partners

Explanation

The correct answer is Choice E.

Cervical dysplasia and cervical carcinoma are causally related to infection with human papillomaviruses
(HPVs). Of the more than 100 types of HPV, more than 30 types can be passed from one person to another
through sexual contact. HPV infections are very common, and the majority of infections do not produce
recognizable signs or symptoms. In many cases the infection may subside on its own over a period of
years. Some HPV infections, however, persist for several years and cause precancerous changes in the
cervical epithelium.

The types of HPVs that are associated wth cancers of the cervix and anogenital tract are known as high-
risk, oncogenic, or carcinogenic HPVs. Because they are transmitted by sexual contact, a history of multiple
sex partners increases a woman's risk of acquiring HPV infection. While the number of sex partners is the
greatest risk factor for infection, studies suggest that whether a woman develops cervical cancer depends
on additional factors acting together with high-risk HPVs. These factors that may increase the risk of
cervical cancer in women with HPV infection include smoking and having many children.

In 2006, the U.S. Food and Drug Administration (FDA) approved Gardasil®, a vaccine that is highly
effective in preventing infection with types 16 and 18, two “high-risk” HPVs that cause most (70 percent)
cervical cancers, and types 6 and 11, which cause most (90 percent) of genital warts.

Diethylstilbestrol exposure, recurrent vulvovaginal Candidiasis, oral contraceptive use, and a family history
of breast and ovarian carcinomas are not known risk factors for HPV infection.

Table 1: Pap Smears results and follow-up management

69
57.

A 32-year-old G3 P2 pregnant woman goes for a routine prenatal check at 12 weeks. On physical
examination she is found to be large for dates, and no fetal heart tones are audible. An ultrasound is
performed and revealed that no fetus was present, only many echogenic cystic areas within the uterus.

Which of the following is the most likely diagnosis?

Invasive mole

Partial hydatidiform mole

Placental site trophoblastic tumor

Choriocarcinoma

Complete hydatidiform mole

Explanation

The correct answer is choice E

Complete moles, which are androgenetic, result from the fertilization of an ovum missing the maternal
genes by a single sperm which duplicates to form the homozygote. The classical "grape-like"clusters are
seen starting the second trimester, due to chorionic villi swelling. Often it is diagnosed in the first
trimester when presenting as a threatened abortion or with abnormal vaginal bleeding and a uterus that is
larger than expectged for gestational age.

Invasive moles (Choice A) are diagnosed on the basis of histological invasion of the myometrium. The

70
uterus is larger than gestational age with vaginal bleeding, possible abdominal distension and pain. One
may also see a theca lutein cyst greater than 6 cm and rising levels of hCG after postevacuation of tissue.

invasive moles must be differentiated from vhoriocarcinoma (Choice D) and chorionic villi are present in
the latter. It has rapid growth and is metastatic, most commonly after a complete mole. Partial moles
rarely give rise to choriocarcinoma. The disease can metastasize to the lungs, brain, liver, heart and skin
leading to dyspnea, hemoptysis, pulmonary artery hypertension, convulsions, focal neurological signs or
abdominal pain with referred pain to the right shoulder tip.

Partial moles (Choice B) are genetically nearly all triploid having both maternal and paternal DNA. The
uterus is often not larger than expected for gestational age and vaginal bleeding tends to occur in the
second trimester or with a missed or incomplete abortion.

Placental Site Trophoblastic Tumors (Choice C) are rare and slow-growing from the intermediate
trophoblasts at the placental implantation site and has been detected after term delivery, nonmolar
abortion, complete or partial moles. Spread occurs late by local infiltration and via lymphatics. Distant
metastases can occur.

58.

A 23 year old woman G2P2 calls her physician 7 days postpartum because she is concerned that she is
still bleeding from the vagina. She denies any fever, chills or feeling unwell. She complains of abdominal
pain but relates it to gas pains. Her review of symptoms was unremarkable.

How many days is normal for bloody lochia to last?

2 days

5 days

8 days

11 days

14 days

Explanation

The correct answer is choice B

Puerperium is defined as the time from the delivery of the placenta through the first few weeks after the
delivery. This period is usually considered to be 6 weeks in duration. By 6 weeks after delivery, most of the
changes of pregnancy, labor, and delivery have resolved and the body has reverted to the nonpregnant
state.

Lochia which is a post-partum vaginal discharge containing blood and mucus and progresses through
three stages:

 Lochia rubra: the first discharge is red in color because of the large amount of blood it contains.
It typically lasts no longer than 3 to 5 days after birth
 Lochia serosa: thinned and turned brownish, containing exudate, erythrocytes, leukocytes and
cervical mucus which lasts until the tenth day after delivery
 Lochia alba: has turned whitish, lasting from the second through the fourth week after delivery. it
has fewer red blood cells and is mainly consistent with leukocytes, epithelial cells, fat and mucus

71
Lochia generally has an odor similar to that of normal menstrual fluid. Any offensive odor indicates
contamination by saprophytic organisms and should be reported to a healthcare provider. If it is retained
inside the uterus, it is known as lochiostasis.

The amount of flow and color of the lochia can vary considerably. Fifteen percent of women have continue
to have lochia 6 weeks or more postpartum. Often, women experience an increase in the amount of
bleeding at 7-14 days secondary to the sloughing of the eschar on the placental site. This is the classic
time for delayed postpartum hemorrhages to occur.

59.

A 17-year-old primigravida has noted no fetal movement during the pregnancy, and she is now at 18
weeks gestation. Oligohydramnios is noted on ultrasound scan, making it difficult to visualize fetal internal
organs, but the size of the baby appears consistent for 18 weeks gestational age. Following a counseling
session with their physician, the parents decide to proceed with prostin induction for a medically indicated
termination of pregnancy. Autopsy reveals no external anomalies upon examination of the stillborn male.

Which of the following findings is most likely to explain this deformation sequence?

Toxoplasmosis

Urethral atresia

Tracheal stenosis

Imperforate anus

Right renal agenesis

Explanation

The correct answer is B

Oligohydramnios is a condition of amniotic fluid deficiency. Other than rupture of membranes, the two
most serious causes of oligohydramnios are abnormalities of the fetal urinary system or
placenta. Diagnosis is made by ultrasound measurement of the amniotic fluid index (AFI) <5cm. It is
typically caused by the following:

 fetal urinary tract abnormalities such as bilateral renal agenesis ( Potter's syndrome )
 fetal polycystic kidneys
 genitourinary obstruction
 Uteroplacental insufficiency is another common cause
 It may also occur without a determinable cause (idiopathic).

Complications may include cord compression, musculoskeletal abnormalities such as facial distortion and
clubfoot, pulmonary hypoplasia and intrauterine growth restriction. Amnion nodosum is frequently also
present (nodules on the fetal surface of the amnion).

Urethral atresia (Choice B) is a rare condition causing neonatal uremia. It is due to a failure of urethral
development, both total or partial, leading to complete urine retention. The prognosis is uniformly fatal
unless accompanied by a rectovesical fistula that allows for bladder drainage.

In both Toxoplasmosis (Choice A) and Tracheal stenosis (Choice C) one would expect to see

72
polyhydramnios.

Imperforate anus (Choice D) is also associated with polyhydramnios

Polyhydramnios is a condition of excess amniotic fluid in the sac. Additional causes of polyhydramnios
include:

 gastrointestinal abnormalities. eg esophageal atresia, duodenal atresia


 facial cleft
 neck masses
 anencephaly (which impairs swallowing reflex)
 twin-twin transfusion syndrome
 chorioangioma of the placenta
 diabetes

Bilateral renal agenesis is associated with oligohydramnios. However, when seen unilaterally as in Right
renal agenesis (Choice E) as long as the other kidney is healthy is not of any major health consequence.
This is associated with mullerian duct abnormalities and can be a cause of infertility in females.

60.

A 30-year-old infertile woman has had episodic lower abdominal and pelvic pain for 2 years. A physical
examination, including pelvic exam, reveals no abnormalities. A Pap smear shows only a few trichomonads
and no dysplastic cells. A laparoscopy is performed, and the gynecologist notes the presence of several
blue to red 0.2 to 0.4 cm slighted raised lesions scattered on the pelvic peritoneum in the cul-de-sac and
broad ligaments.

Which of the following is the most likely diagnosis?

Metastatic adenocarcinoma

Neisseria gonorrheae infection

Endometriosis

Ovarian Cysts

Leiomyomata

Explanation

The correct answer is Choice C

Endometriosis is a malady where endometrial glands and stroma are located outside the uterine cavity and
is found in up to 30% of infertile women. Definitive diagnosis is made by biopsies of the lesions found on
laparoscopic examination.

The gross appearance of endometriosis seedlings may be new blood-filled red or blue-black lesions
seemingly raised or cystic. Cysts range in size between 1mm to 10 cm. Patients with endometriosis present
with a variety of symptoms including dysmenorrhea (most common), heavy or irregular bleeding, pelvic
pain, lower abdominal or back pain, and dyspareunia.

Typical physical signs of endometriosis include a fixed retroverted uterus, nodularity of uterosacral
ligaments and posterior cul-de-sac, possible adnexal masses and tenderness on examination. The stromal

73
implants can cause a rigidity or fixing of the fallopian tubes as well as blocking of the tubes,preventing the
sperm and ova from meeting for fertilization.

In terms of Metastatic adenocarcinoma (Choice A), ninety percent of women with endometrial malignancies
have abnormal vaginal discharge with 80% having abnormal bleeding. A fractional dilatation and curettage
allows for a histological examination to confirm the neoplasia. Endometrial biopsies can also confirm the
diagnosis. Endometrial cells found on a screening pap smear for a menopausal woman, calls for further
action. Transvaginal ultrasound may also be helpful in the diagnosis.

Many gonorrheae infections (Choice B) are asymptomatic, delaying diagnosis and treatment. Untreated,
the disease can progress to pelvic inflammatory disease or disseminated gonorrhea infections. A diagnosis
is made on the basis of cervical Gram stain, and endocervical cultures (for the majority of cases and is 80-
90% accurate)

Ovarian Cysts (Choice D) are fluid-filled sacs which although harmless, may lead to rupture, bleeding or
pain, depending on its size. On a sonogram, it appears as a bubble. In an ovulatory cycle, functional cysts
formed. If an egg is not released, the ovary can fill with fluid but will usually resolve after a few menstrual
cycles.

There are several non-functional cysts, the most common of which is the dermoid cyst which contains
several types of primordial tissue including muscle, bone and hair mixed with mucinous material. Another
is a polycystic-appearing ovary with small cysts present around the ovarian exterior.

Leiomyomata (Choice E) (also known as myomas or fibroids) are benign tumors stemming from smooth
muscle cells and vary with the amount of fibrous tissue contained within. Locations can be subserosal,
intramural and submucosal. Diagnosis of leiomyomata can be made by physical examination or ultrasound.

61.

A 32-year-old woman presents to your office at 32 weeks' gestation for a routine prenatal visit. Her
pregnancy has been uncomplicated. Her only complaint is an occasional headache that is relieved with
acetaminophen. Fundal height is 31 cm, and fetal heart rate is 144 bpm. Her BP is 152/92 mm Hg, and
she has 1+ protein on urine dipstick testing.

What is the next appropriate step in management?

start on Empiric Antibiotics for presumed Urinary tract infection

repeat BP measurement and urine dipstick testing at her next routine prenatal visit

order laboratory evaluation of complete blood cell (CBC) count; liver function tests; uric acid, BUN,
and creatinine levels; 24-hour urine collection for determination of total protein level; and perform
antenatal fetal testing as an inpatient

order laboratory evaluation of complete blood cell (CBC) count; liver function tests; uric acid, BUN,
and creatinine levels; 24-hour urine collection for determination of total protein level; and perform
antenatal fetal testing as an outpatient

schedule a follow-up visit in 3 to 4 days for reevaluation

Explanation

The correct answer is choice C

There is a real possibility that the patient has preeclampsia, although it can also be a renal condition. A
good clue that the patient is starting to become symptomatic is her headaches in addition to the elevation

74
in blood pressure and proteinuria. Although eclampsia is potentially fatal, pre-eclampsia is often
asymptomatic, and so its detection depends on signs or investigations. It is prudent to check and see what
the degree of her problem is. Symptoms of preeclampsia include:

 hypertension
 edema
 proteinuria

If ignored, preeclampsia can lead to eclampsia (which is the above symptoms plus seizures) with threat of
death to mother and infant. This happens only very rarely with proper treatment. Therefore scheduling a
follow-up visit (Choice E) (and Choice B) would be an injustice to the patient. As her BP is high, she may
quickly evolve into severe preeclampsia. This patient requires immediate inpatient management! This
Choice D in incorrect.

The only known treatments for eclampsia or advancing pre-eclampsia are abortion or delivery, either by
labor induction or Caesarean section. Antihypertensives may reduce maternal and fetal mortality among
pregnancy patients with hypertension. In some cases, women with preeclampsia or eclampsia can be
stabilized temporarily with magnesium sulfate intravenously to forestall seizures while steroid injections
are administered to promote fetal lung maturation.

Fetal tachycardia is beginning to manifest (with a rate greater than 140) and fetal testing (in the form of
monitoring or nonstress test) would be prudent. While the patient is being observed, a 24 hour urine
collection can be done to measure total protein level to ensure that the problem that the patient is
experiencing is not from renal etiology.

62.

A 24 year old primigravida who received no prenatal care has marked vaginal bleeding after the onset of
labor at 38 weeks gestation. Cesarean section is performed and a lacerated low-lying placenta is removed.
She remains hypotensive for 6 hours and requires transfusion of 12 packed RBC units. Postpartum, she
becomes unable to breast-feed the infant and does not have a normal resumption of her menstrual cycle.
She becomes more sluggish and tired. Laboratory findings include hyponatremia, hyperkalemia, and
hypoglycemia.

Which of the following pathologic lesions is she most likely to have had following delivery?

Pituitary necrosis

Metastatic carcinoma

Subacute thyroiditis

Bilateral adrenal hemorrhage

Insulitis

Explanation

The correct answer is choice A.

The patient has Sheehan syndrome, also known as postpartum hypopituitarism or postpartum pituitary
necrosis, is hypopituitarism (decreased functioning of the pituitary gland), caused by necrosis due to blood
loss during and after childbirth. Most common initial symptoms of Sheehan's syndrome are agalactorrhea
(absence of lactation) and/or difficulties with lactation. Many women also report amenorrhea after delivery.
In some cases, a woman with Sheehan syndrome might be relatively asymptomatic, and the diagnosis is

75
not made until years later, with features of hypopituitarism.

The pathophysiology of Sheehan syndrome includes hypertrophy and hyperplasia of lactotrophs during
pregnancy results in the enlargement of the anterior pituitary, without a corresponding increase in blood
supply. Secondly, the anterior pituitary is supplied by a low pressure portal venous system. These
vulnerabilities, when affected by major hemorrhage or hypotension during the peripartum period, can
result in ischaemia of the affected pituitary regions leading to necrosis. The posterior pituitary is usually
not affected due to its direct arterial supply.

A choriocarcinoma could possibly occur following a normal pregnancy, but metastases are typically to the
lungs. Metastases involving the pituitary are quite rare. Thus, metastatic carcinoma (choice B) is not an
appropriate answer.

Subacute thyroiditis (choice C) could lead to hypothyroidism, but this does not explain her adrenal
insufficiency.

Adrenal hemorrhage (choice D) could explain acute adrenal insufficiency with the laboratory findings given,
but not the amenorrhea or failure of lactation or features of hypothyroidism.

Insulitis (choice E) is a pre-clinical finding for type I diabetes mellitus. This does not explain
panhypopituitarism. The maternal pancreas is not affected by pregnancy.

63.

A 15-year-old girl presents to her primary care physician because of amenorrhea. She states that her
menses were initially irregular, but had been regular for about the last year until 3 months ago, when she
had her most recent period. Review of systems is otherwise negative. She is 5 ft 4 in tall and weighs 110
lb, and the remainder of her examination is unremarkable.

Which of the following is the most appropriate next step in this patients management?

Follicle-stimulating hormone and luteinizing hormone measurement

Urine bHCG

Progesterone challenge

Reassurance

Vaginal smear

Explanation

The correct answer is choice B

Since this patient has had regular menses over the course of the last year, there is good evidence that she
has been ovulating. If it suddenly stopped, the first step is to rule out pregnancy in this teen.

If the pregnancy test is negative, you can do a progesterone challenge (choice C) because she may have
stopped ovulating secondary to stress or diet changes and therefore the progesterone will be low. If this is
the case, a five day oral course of progesterone and then stopping the medication will initiate withdrawal
bleeding.

Blood for prolactin levels and thyroid function may then be drawn.

76
Follicle-stimulating hormone and luteinizing hormone measurement (Choice A) is not indicated because this
is not primary amenorrhea, and she has no symptoms of ovarian failure. However estrogen and
testosterone levels may be drawn here to determine elevation which may be indicative of polycystic
ovarian syndrome. Insulin resistance is also associated with this condition.

Reassurance (choice D) should only be given when all the tests and examinations have been found to be
normal.

A vaginal smear may be performed as a routine test in the office (Choice E) but this does not address the
problem of amenorrhea. Also if the hymen is intact, a speculum examination may be contra-indicated.

64.

A 19-year-old woman gave birth to a healthy male infant at term following an uncomplicated pregnancy.
She has now been breast feeding the baby for a month, but notes that her left breast has gradually
become swollen and painful to touch over the past week. On physical examination her temperature is 38.2
C.

Which of the following is the most likely diagnosis?

Acute mastitis

Fibrocystic disease

Fat necrosis

Intraductal papilloma

Galactocele

Explanation

The correct answer is choice A

Acute mastitis is an inflammation of the mammary gland. Clinical signs include pain, heat and swelling of
the affected gland and abnormality of the milk, either as flakes or clots. It is diagnosed on the basis of
bacteriological exam, principally based on the cell count of the milk. It commonly occurs from a blocked
milk duct (with lactation).

Acute mastitis is characterized by a sudden onset, moderate to severe inflammation of udder, decreased
production, and serous milk/fibrin clots. Systemic signs are similar but less severe than the peracute
form. Treatment of acute mastitis includes: Administration of antibiotics (systemic or intramammary),
administration of fluids if needed, and administration of anti-inflammatory drugs, analgesics, and/or
antipyretics.

Fibrocystic disease (Choice B) is a benign condition of the breasts which may predispose to chronic
mastitis. Lumps are often detected by the patient with symptoms of pain or temporary discomfort made
worse during menses.

Fat necrosis (Choice C) which is damage to some fat tissue within the breast can present as a mass. This
cannot be distinguished from a malignancy without a biopsy.

Intraductal papilloma (Choice D) is a small, benign tumor which grows in a milk duct of the breast
presenting with nipple discharge, pain in the breast, breast lump and breast enlargement. It is the most
common cause of spontaneous nipple discharge from a single duct.

77
Galactocele (Choice E) is a benign cystic tumor of the mammary glands containing milk or milky
substances and is caused by a protein plug that blocks the outlet. Once lactation has ended, the cyst
usually resolves spontaneously without further treatment.

65.

A 24-year-old G1 P1 woman gives birth to a male infant at 36 weeks gestation. Microscopic examination
of the placenta following delivery reveals that numerous polymorphonuclear leukocytes are present in the
fetal membranes (both chorion and amnion).

Which of the following is most likely to be associated with this placental finding?

Herpes simplex infection

A multiple gestation

Meconium staining

Abruptio placenta

Premature rupture of membranes

Explanation

The correct answer is choice E

Premature rupture of membranes is associated with a higher incidence of chorioamnionitis which is an


infection of the chorion and amnion (as witnessed by the infiltration of increased leukocytes). It occurs
from a bacterial infiltration up the birth canal and is increased with multiple vaginal examinations.

Infiltration of the chorionic plate by neutrophils is diagnostic of mild chorioamnionitis with subamniotic
tissue and fetal membrane necrosis in more severe chorioamnionitis cases. Abscess formation may be
found. Associated with severe chorioamniotis is vasculitis of the umbilical vessels due to the fetal
inflammatory cells and if very severe, funisitis occurs (inflammation of the umbilical cord's connective
tissue). Chorioamnionitis is associated with neonatal sepsis. Infants may have the following symptoms:

 dyspnea
 diarrhea
 hypoglycemia
 decreased movements, or sucking
 seizures
 bradycardia
 vomiting
 jaundice

Blood tests to diagnose neonatal sepsis and identify the bacteria include:

 C-reactive protein
 blood culture
 complete blood count

Neonatal sepsis is a leading cause of infant death. The quicker the infant receives treatment, the better

78
the outcome. Babies younger than 4 weeks old may be started on antibiotics before lab results are back
as it may take 24-72 hours.

Herpes simplex (Choice A) causes an increase in eosinophiles and typically causes an infection of the
vaginal adn vulvar regions.

Multiple gestation (Choice B) does not pertain to this case since there was only one baby delivered with no
additional gestational tissue.

Meconium is a thick, greenish, tarry substance lining the fetal intestines which is released in utero when
the anal sphinter relaxes during fetal distress. Evidence of this is greenish particulate matter in the
amniotic fluid and coating the umbilical cord, referred to as meconium staining. (Choice C)

Abruptio placenta (Choice D) is an obstetrical complication of the third trimester due to a premature
separation of the placenta from the wall of the uterus. Symptoms include vaginal bleeding (which can be
heavy depending on the extent of separation), pelvic and abdominal pain and uterine rigidity.

66.

A 26 year old lady comes to you with a history of increase in her facial hair, and deepening of her voice.
On exam, a mass is present over the left ileac fossa along with an enlarged clitoris.

What is the most likely diagnosis:

Granulosa-theca cell tumor

Brenner tumor

Sartoli-leydig tumor

Dysgerminoma

Krukenberg tumor

Explanation

The correct answer is choice C.

Sartoli-leydig tumor (closely related to arrhenoblastoma and androblastoma) is a group of tumors made up
of Sertoli cells, Leydig cells and intermediate and poorly differentiated neoplasms along with primitive
gonadal stroma. It is a part of the sex cord-stromal tumor group of ovarian and testicular cancers and is
almost always unilateral. It can be solid, semicystic or papillary.

The tumor secretes excess testosterone creating masculinization in one-third of female patients. This is
preceded by anovulation, oligomenorrhea, amenorrhea and defeminization. Additional symptoms include:

 hirsutism
 deepening voice
 acne
 clitoromegaly
 temporal hair loss

Blood test reveals very high serum testosterone level.

Granulosa-theca cell tumors (Choice A) can occur at any age after puberty but is more common in

79
postmenopausal women and is most common clinically estrogenic ovarian tumor. It can be associated with
endometrial hyperplasia and carcinoma. Characteristic features include Call-Exner bodies (small cavities
filled with eosinophilic matter) and nuclei with longitudinal grooves (coffee bean appearance)

Brenner tumors (Choice B) are rare, usually benign tumors of the ovary characterized by groups of
epithelial cells lying in a fibrous connective tissue stroma.

Dysgerminoma (Choice D) is a type of malignant germ cell tumor of the ovary occuring in adolescence and
in early adult life. It is very rare after age 50. Grossly, the tumor has a smooth, bosselated (knobby)
surface with a soft, fleshy, gray or pink consistency when cut open. Microscopically, dysgerminomas
typically reveal uniform cells that resemble primordial germ cells and lymphocytes are in the stroma.

Krukenberg tumors (Choice E) belong to a class of metastatic ovarian malignancies whose primary site
rises in the gastrointestinal tract or breast. They are often found bilaterally. Microscopically, they are
characterized by mucin-secreting signet-ring cells in the ovarian tissue.

67.

A 25 year-old gravida 4, para 1 mother presents at your clinic at 15 weeks gestation. Her history includes
two spontaneous abortions during the 5th and 7th week of gestation, but one successful pregnancy
resulting in a healthy child who is now 2 years of age. A triple test shows alpha-fetoprotein levels higher
than normal for this point in the pregnancy.

This may be an indication of any the following, except:

omphalocele

Down syndrome

spina bifida

twins or triplets

gastroschisis

Explanation

The correct answer is choice B.

The prenatal screen known as a triple test, or triple screen, is a measure of the levels of alpha-fetoprotein
(AFP), estriol, and beta human chorionic gonadotropin (beta-hCG) in the maternal blood.

A higher than normal level of AFP suggests that the fetus is at risk for conditions that develop when parts
of the embryo do not close when they should. Such conditions include omphalocele (choice A),
gastroschisis (choice E), and neural tube defects such as spina bifida (choice C).

The neural tube is at the midline of the embryo and develops into the central nervous system. When it
does not close completely, parts of the spinal cord can be covered incompletely, usually in the lumbar and
sacral levels. Nonclosure also can occur at the level of the head, resulting in anencephaly, resulting in a
lack of a forebrain and parts of the skull and scalp that go over it.

Gastroschisis is a defect in the abdominal wall, resulting in intestines developing outside of the abdomen of
the fetus. Other organs also may develop outside of the abdomen. Omphalocele is similar to gastroschisis,
except that in omphalocele intestines, and organs such as the liver, though outside the abdomen are
contained within a sac.

80
Alternatively, a high AFP may mean that the mother is carrying twins or triplets (choice D).

Down syndrome, on the other hand, is associated with a low level of AFP, as is Trisomy 18.

68.

A 28-year-old G3 P2 woman has had an uncomplicated pregnancy. A screening ultrasound is performed at


16 weeks gestation, and the findings prompt performance of maternal serum alpha-fetoprotein test, which
is elevated.

Which of the following abnormalities of the CNS is most likely to be present in this fetus?

Holoprosencephaly

Metachromatic leukodystrophy

Anencephaly

Germinal matrix hemorrhage

Spina bifida occulta

Explanation

The correct answer is choice C

Anencephaly is a cephalic disorder that results from a neural tube defect that occurs when the cephalic (head) end of the
neural tube fails to close, usually between the 23rd and 26th day of pregnancy, resulting in the absence of a major portion of
the brain, skull, and scalp[1]. Children with this disorder are born without a forebrain, the largest part of the brain consisting
mainly of the cerebral hemispheres. Recent studies have shown that the addition of folic acid to the diet of women of child-
bearing age may significantly reduce, although not eliminate, the incidence of neural tube defects. Therefore, it is
recommended that all women of child-bearing age consume 0.4-1 mg of folic acid daily.

Neural tube defects are associated with an elevation of maternal serum alpha-fetoprotein as opposed to Down syndrome
which is associated with an abnormally low AFP.

81
Elevated AFP can also be found due to other causes including:

 Multiple gestation
 Fetal demise
 Premature delivery
 Growth retardation
 Abdominal wall defect
 Congenital nephrosis
 Maternal liver disease

Holoprosencephaly (Choice A) is a condition whereby the forebrain of the embryo fails to develop into two hemispheres.
Because of this, structures that are to be paired to the left or the right side of the midline, fail to occur with defects in the
facial development (eg.cyclops).

Metachromatic leukodystrophy (Choice B) is a genetic disorder affecting nerves, muscles and behavior. Symptoms include:

 decreased mental function


 decreased muscle tone
 difficulty on ambulation
 feeding difficulties
 loss of muscle control
 seizures
 dysphagia

Germinal matrix hemorrhage (Choice D) is the most common neurologic injury in preterm neonates where bleeding occurs
into the subependymal germinal matrix with or without rupture into the lateral ventrical of the developoing brain.

Spina bifida occulta (Choice E) is a condition in which one or more vertebrae are malformed. Because there is no open neural
defect, the AFP is not elevated.

82
Table 1: Maternal Serum Screen Results Summary

69.

A 21-year old presents to her obstetrician at 30 weeks with a fundal height of 50 cm. She is otherwise
healthy and her pregnancy is otherwise normal. She has been seeing you regularly for the past few
months and you note no abnormalities in your chart.

Which of the following statements concerning polyhydramnios is true?

Acute polyhdramnios always leads to premature labor

The occurrence of associated malformations is approximately 3%

Maternal edema in the lower extremities is rare

complications include uterine dysfunction and placental abruption

In 10% of cases, esophageal atresia is associated with polydramnios

Explanation

The correct answer is choice D

Polyhdramnios is an excessive quantity of amniotic fluid around the fetus. The condition may occur as a
result of gastrointestinal, neurological, lung or other disorders. The most frequent maternal complications
associated with polyhdramnios are placental abruption, uterine dysfunction and postpartum hemorrhage.

The incidence of asociated malformations is approximately 20% with gastrointestinal and central nervous
system abnormalities especially common. Polyhydramnios accompanies half of anencephaly cases and
almost all cases of esophageal atresia.

Edema of the lower extremities, vulva and abdominal wall (choice C) occurs due to compression of major
venous systems.

Causes of polyhdramnios include the following:

 neural tube defects


 multiple pregnancy

83
 prune belly syndrome
 Bamforth-Lazarus syndrome
 congenital myotonic dystrophy
 gestational diabetes
 congenital sodium diarrhea
 langer-saldino achondrogenesis
 cystic adenomatoid malformation of lunk
 milker-dieker syndrome
 spina bifida

 anencephaly

70.

An 88 year old female patient comes to your office complaining about urinary incontinence that has
developed over the past few months. She is otherwise relatively healthy and shows no signs of cognitive
impairment. She is on multiple medications for hypertension, elevated lipids and hypothyroidism but her
medications have been stable for years. She is an active smoker and had no plans to change her smoking
habits.

In which of the following conditions is the urine storage phase characterized by involuntary detrusor
contractions?

Genuine stress urinary incontinence

Overactive bladder (OAB)

Mixed incontinence

Overflow incontinence

Transient incontinence

Explanation

The correct answer is choice B.

Urinary incontinence can often be classified into different clinical types that can be useful in diagnosis and
planning treatment. However, in some patients, the types of symptoms overlap among the different
diagnostic categories and do not correlate exactly with the underlying pathophysiological defect.

Overactive bladder (OAB) is a condition that results from sudden, involuntary contraction of the bladder
muscles, leading to sudden urinary urgency. The detrusor is the main muscle of the bladder wall. This
condition is sometimes referred to as urge incontinence. The involuntary detrusor contractions may be
spontaneous or provoked by events such as postural changes (from lying to standing), running water, or
hand washing. In women with neurologic disease, uninhibited detrusor contractions are known as
neurogenic detrusor overactivity.

Stress incontinence (choice A) results from anatomical weaknesses or defects in the supporting structures
of the bladder (often caused by pregnancy or other changes). In stress incontinence, leakage occurs with
an increase in abdominal pressure, such as coughing, sneezing, or a Valsalva maneuver, without a rise in
true detrusor pressure.

Overflow incontinence (choice D) results from improper emptying of the bladder, resulting in urine leakage.

84
A combination of stress and overflow incontinence is typically referred to as mixed incontinence.

Transient incontinence (choice E) occurs because of a temporary condition such as medication or infection.

71.

A 27 year-old lady attends early pregnancy assessment clinic with her partner and is distressed to learn
that she has had a miscarriage, with a 7 week non-viable pregnancy. This is her third in 4 years. On
reviewing her notes you note a raised anti-cardiolipin antibody titre in her laboratory results section.

All of the following clinical problems would fit with the diagnosis of anti-phospholipid syndrome except?

Headaches

Thrombocytosis

Systemic lupus erythematosus

Deep venous thrombosis

Livedo reticularis

Explanation

The correct answer is choice B

Anti-phospholipid Syndrome (APS) is one of the most important causes of hypercoagulability which stem
from the presence of autoantibodies directed against anionic macromolecules. Many have other connective
tissue disorders like SLE. The autoantibodies in APS can result in a prolonger prothrombin time.

The syndrome occurs due to the autoimmune production of antibodies against phospholipid (aPL), a cell
membrane substance. In particular, the disease is characterised by antibodies against cardiolipin (anti-
cardiolipin antibodies) and β2 glycoprotein I.

APS is commonly seen in conjunction with other autoimmune diseases; the term "secondary
antiphospholipid syndrome" is used when APS coexists with other diseases such as systemic lupus
erythematosus (SLE). In rare cases, APS leads to rapid organ failure due to generalised thrombosis and a
high risk of death; this is termed "catastrophic antiphospholipid syndrome" (CAPS). Symptoms include the
following:

 venous thrombosis
 pregnancy loss
 arterial thrombosis
 thrombocytopenia
 livedo reticularis (a skin condition)
 thrombocytopenia (not thrombocytosis - choice B)
 Headaches and migraines

Since the patient has had 3 pregnancy losses- she is considered to be a habitual aborter which always
merits further investigation.

Antiphospholipid syndrome is tested for in the laboratory using both liquid phase coagulation assays (lupus
anticoagulant) and solid phase ELISA assays (anti-cardiolipin antibodies). Genetic thrombophilia is part of
the differential diagnosis of APS and can coexist in some APS patients. Thus genetic thrombophilia

85
screening can consist of:

 Further studies for Factor V Leiden variant and the prothrombin mutation, Factor VIII levels,
MTHFR mutation.

 Levels of protein C, free and total protein S, Factor VIII, antithrombin, plasminogen, tissue
plasminogen activator (TPA) and plasminogen activator inhibitor-1 (PAI-1)

72.

A 27 year old nulligrvida woman and her husband have been unable to conceive for 12 months. She has
never used hormonal contraception. Menses occur at 28 day intervals, and her last menstrual period was
2 weeks ago. She had a single episode of pelvic inflammatory disease 4 years ago and was treated with
the appropriate antibiotics. Since then she has had no abnormal vaginal discharge, dyspareunia, or pelvic
pain. Vaginal examination shows no abnormalities. Endocervical cultures are negative for Gonorrhea and
Chlamydia. On bimanual exam, the uterine size, shape, and position are within normal limits and the
ovaries are non enlarged.

Which of the following is the most appropriate next step in diagnosis?

reevalution in 6 months

hysterosalpingography

sperm penetration assay

pelvic ultrasound

hysteroscopy

Explanation

The correct answer is choice B.

Infertility is defined as a couple not being able to conceive after one year of regular, unprotected
intercourse. If a woman is over 35, diagnostic work up can be initiated after 6 months, rather than one
year, in order to maximize the time left during which she may be able to conceive. (As age increases,
ability to conceive decreases). In a patient with an unremarkable gynecologic history, both she and her
partner should be evaluated for infertility, including a comprehensive exposure history. In this case, there
is a pertinent history of pelvic inflammatory disease, which can result in tubal occlusion or scarring,
especially if it was due to Chlamydia trachomatis infection.

In this case a hysterosalpingogram (Choice B) would be the most appropriate way to identify an occlusion.
Hysterosalpingography may also itself be therapeutic, as instillation of contrast material can sometimes
open the blocked tube. Laparoscopy is also an option for diagnosis, however it is more invasive and
generally not the first step in a younger woman (under 35).

Waiting 6 months (Choice A) would be plausible if there were no history of pelvic inflammatory disease (a
known risk factor) and a positive finding on exposure history for either the patient or her partner (ie the
partner wearing tight underwear) in order to allow for correction of lifestyle risk factors.

Sperm penetration assay (Choice C) could be considered further down the diagnostic work up if the
patient's tubes are determines to be free from pathology. A semen analysis is a less expensive exam to
perform to determine sperm counts and motility.

Pelvic ultrasound (Choice D) cannot image the fallopian tubes well and cannot assess patency.

86
Hysterosalpingo-contrast-ultrasonography may be utilized to better assess, however this is not a readily
accessible study.

Hysteroscopy (Choice E) can evaluate uterine abnormalities, however this test on its own is invasive and
surgical correction of uterine abnormalities have not been demonstrated to improve pregnancy rates. Her
history of pelvic inflammatory disease also makes tubal abnormalities more likely than a uterine cause.

73.

A 33 year-old Jewish woman is admitted to hospital with nausea, fever, and severe pain in the right lower
abdominal quadrant. She is nulliparous with no major illnesses or surgery in her history, though she is a
former smoker who quit at age 27. On physical examination, her blood pressure, pulse rate, and
respiratory rates are normal, at 110/72 mmHg, 68 beats/min, and 16 breaths/min, respectively. Her
temperature is elevated at 38.2°C (normal 37.0). You note rebound tenderness in the abdomen which is
most severe around the right iliac fossa.

Which of the following conditions should be high in the differential diagnosis?:

acute appendicitis

ectopic pregnancy

endometriosis

familial Mediterranean fever

all of the above are true

Explanation

The correct answer is choice E.

Clearly, this presentation allows for several possible diagnoses. While the incidence of appendicitis (choice
A) peaks in late adolescence, it does not decline until the geriatric years. Thus, when anybody from early
adulthood through late middle age presents with pain in the right lower quadrant, appendicitis figures
prominently among possible diagnosis. The presence of rebound tenderness, together with a fever makes
this condition more probable. Additional information that would support this, if available, is a white blood
cell count (WBC) above 11,000 cells/ul (normal range is 4,500-10,000) and elevated levels of C-reactive
protein.

The incidence of ectopic pregnancy rises with age in women in their child-bearing years. While the highest
risk group is ages 35-44 years, smokers have been shown to have 1.6-3.5 times the risk of non-smokers.
The location of the ectopic pregnancy can vary, but one common location is in the fallopian tube. If this
occurs in the right tube, the presentation can be very similar to that of appendicitis. An important
component of the etiology of tubal pregnancy is a history of pelvic inflammatory disease (PID) due to
Chlamydia trachomatis which is transmitted sexually. Important relevant information might be obtained by
taking a careful history and includes knowing whether or not this woman has had a high number of sexual
partners and whether she has attempted to get pregnant and had difficulty. The reason for this is that ova
traveling from the ovaries during ovulation tend to get slowed or trapped in fallopian tubes that have
narrowed due to fibrosis resulting from the past PID. This causes infertility while at the same time can lead
to a tubal pregnancy if a fertilized ovum does is not able to traverse the tube and thus implants there.
Thus, any woman of child-bearing age presenting with abdominal pain should be given a pregnancy test
and if positive she must be worked up for an ectopic pregnancy.

In endometriosis (choice C) , endometrium-like tissue is found outside of the uterus. This can cause a
chronic inflammatory response in areas of the pelvis such as on the vagina, cervix, ovaries, fallopian tubes,
uterosacral ligaments and the rectovaginal septum. Typically, endometriosis limited to the pelvis develops
in women ages 25-30, while outside of the pelvis it develops with a still older peak incidence of 35-40

87
years. It is another cause of infertility, but whether or not this woman has a fertility problem, given her
age, endometriosis must be considered.

Although it has other components, such as arthralgia (joint pain), Familial Mediterranean fever (FMF,
choice D) tends to manifest with episodes of peritonitis, often mimicking acute appendicitis, sometimes
resulting in a healthy appendix being removed.

FMF is an autosomal recessive condition that appears in people who are homozygous for an MEFV gene, of
which several are known. Some MEFV genes are common in Arab people, occuring with in an estimated
1/50 Arabs, resulting in an FMF incidence of 1/2600 in many Arabic-speaking countries. FMF is even more
common in Turkey with an incidence of 1/1000, partly because the incidence is extremely high among
Armenians, with a gene frequency of 1 in 7 people and a disease incidence of 1/500.

Almost as high is the incidence among Sephardic Jews. Descended from Jews expelled from Spain and
Portugal who migrated to North Africa, but also to Holland and southeast Europe, including Turkey,
Sephardic Jews have an estimated FMF incidence of 1/250-1,000; thus, the gene occurs in 1 out of every
8-16 Sephardic Jews. While Ashkenazic Jews (which make up most Jews in the US and Canada) previously
were thought to have a fairly low incidence of FMF genes, studies have revealed a very high incidence, 1/5,
of a particular FMF gene, but interestingly it has resulted in a disease incidence of only 1/73,000
suggesting a low penetrance for this particular gene.

74.

You have just started your obstetrics rotation and are called to the emergency room to see a 23 year old
female with right lower quadrant pain. Her bHCG is positive and she shows classic signs of an ectopic
pregnancy. She is otherwise healthy and on no current medications. She is sexually active and normal
uses a form of contraception but was traveling in europe recently and had unprotected intercourse. In the
ER, she is stable and has no immediate concerns.

Which of the following statements about the treatment of ectopic pregnancy is true?

Treatment failure with methotrexate (MTX, Trexall) is most likely in women with high serum
beta-hCG concentrations (greater than 5000 mIU/mL).

Rates of tubal patency are much higher after medical management than after surgical
salpingostomy.

Women with signs of impending rupture of ectopic pregnancy should be treated with mutliple-
dose MTX therapy.

Methotrexate treatment results in loss of fertility due to loss of ovarian reserve.

Future intrauterine pregnancy rates are higher with laparoscopic salpingostomy than with MTX
therapy.

Explanation

The correct answer is Choice A.

Methotrexate (MTX, Trexall) is a commonly used treatment for women with unruptured ectopic
pregnancies. The optimal candidates for MTX treatment are patients who have a human chorionic
gonadotropin beta-subunit (hCG) concentration ≤5000 mIU/mL and no fetal cardiac activity. Ectopic mass
size less than 3 to 4 cm is generally used as a patient selection criterion for MTX therapy although this
criterion has not been confirmed as a predictor of success.

Women with a high baseline hCG concentration (greater than 5000 mIU/mL) are more likely to require

88
multiple courses of MTX or experience treatment failure.

Treatment with MTX has not been shown to compromise ovarian reserve or function. In women who are
eligible for medical management of ectopiuc pregnancy, treatment with MTX is as effective as surgery and
leads to similar success rates for tubal patency and future intrauterine pregnancy.

Women who have signs of rupture or impending rupture of ectopic pregnancy such as severe or persistent
abdominal pain or significant peritoneal fluid outside the pelvic cavity should not be treated with MTX but
managed surgically. Most ectopic pregnancies can be treated by a laparoscopic procedure.

75.

A 2 week old female infant is noted to have a thin membrane adhering together the upper portion of labia
minora. The pregnancy and birth history are normal for this neonate.

The most appropriate course of action for these labial adhesions includes which of the following:

apply estrogen cream daily

refer for surgical repair

apply traction to the opposing labia until the adhesion breaks

evaluate the patient for congenital adrenal hyperplasia

do nothing as the lesions are of no consequence

Explanation

The correct answer is choice A

The condition described in this example is agglutination of the labia. This may be confused with an
imperforate hymen. The simplest effective treatment is very gentle digital separation of the adhesions and
the use of estrogen cream applied daily to prevent adhesion formation form reoccurring. The disorder is
usually asymptomatic and is often first noticed by parents or during a routine physical examination. A host
of other pediatric vaginal or urethral disorders, including an imperforate hymen or a septate vagina, must
be excluded. Treatment of labial adhesions is typically conservative. Labial adhesions are fibrous adhesions
between the labia majora. Labial adhesions are due to low estrogen levels in prepubertal girls and contact
with irritants.

Labial adhesions can often be managed with periodic observation, and spontaneous resolution has been
reported in as many as 80% within 1 year. If treatment is necessary based on symptoms or parental
request, estrogen cream is indicated. It is directly applied to the labia minora can be used twice daily for 2-
4 weeks. A literature review performed in 2007 reported that the success rate of topical estrogen
intervention in girls with labial adhesions is typically about 90%, with published success in case series
reports ranging from 46.7-100%.

Surgical repair (Choice B) is not necessary in that gentle digital separation will suffice. If medical care does
not result in separation of the labia minora, manual or surgical separation may be considered. Other
reasons to consider intervention include severe fibrous dense adhesions or rare cases with urinary
retention.

Applying traction to the opposing labia until the adhesion breaks (Choice C) may cause tearing and trauma
which if it occurs could lead to possible scarring.

Evaluating the patient for congenital adrenal hyperplasia (Choice D) is a step that one would take if the

89
cervix, uterus and vagina were found to be malformed in development.

Doing nothing as the lesions are of no consequence (Choice E) is not valid as the adhesions, once filmy can
become thickened over time.

76.

A 57 year old woman presents to your office because of vaginal bleeding of four months' duration. She
reports that her last spontaneous menstrual period was at age 50. She does not take hormone therapy or
any other prescription medications. Her last periodic health examination was 1 year ago. Both physical
and pelvic examinations are unremarkable.

Which of the following is the most likely diagnosis?

Fibroids

Blood coagulation disorder

Endometrial carcinoma

Cervical carcinoma

Ovarian cancer

Explanation

The correct answer is choice C

Uterine bleeding in a postmenopausal woman raises the suspicion of endometrial cancer or endometrial
hyperplasia and must be evaluated to rule out these conditions.

In women who are still ovulating, abnormal uterine bleeding is usually due to one of the following:

 abnormal bleeding of pregnancy


 fibroids (leiomyomata)
 polyps
 adenomyosis
 blood clotting disorders
 systemic lupus erythematosus
 hypothyroidism
 medications that interfere with blood clotting

Women who are postmenopausal (those who have not had a menstrual period for 12 consecutive months
or more) should not experience vaginal bleeding. Any vaginal bleeding is considered abnormal in
postmenopausal women and must be investigated to rule out the possibility of endomtetrial hyperplasia or
cancer.

Fibroids or leiomoymata (choice A), are a more common source of bleeding in premenopausal women than
in postmenopausal women.

Although a blood coagulation disorder (choice B) can present with vaginal bleeding, it is an unlikely choice
because of the women's age and the fact that a bleeding disorder would be apparent earlier in life.

While cervical cancer (choice D) can present with vaginal bleeding, it is a less likely cause of bleeding in a

90
postmenopausal woman than endometrial cancer. Endometrial carcinoma is the most common gynecologic
cancer in women over 45 years of age.

Vaginal bleeding is not a typical symptom for ovarian cancer (choice E).

77.

A 24-year-old primigravida woman arrives at the hospital at term at the onset of labor, following what she
describes as an uneventful pregnancy. On taking a history, however, you learn that occasionally she has
experienced abnormally heavy bleeding during menstrual periods, one of which recalls lasted as long as 9
days. You order a complete blood count (CBC) which reveals a red blood cell count and hematocrit that
are normal for pregnancy, but you find that her bleeding time is prolonged at 14 minutes (normal
bleeding time is up to 9.5 minutes).

Which of the following tests is most important to perform next?:

von Willebrand antigen test

Factor VIII concentration

G6PDH level

Coomb’s test

A and B

A and B and C

A and B and C and D

Explanation

The correct answer is choice A.

A history of a prolonged menstrual period, or menorrhagia, reported for this woman together with the
prolonged bleeding time suggests that she may have von Willebrand disease. Affecting an estimated one
percent of the population, von Willebrand disease is the most common hereditary inherited bleeding
disorder. It results from a deficiency or abnormality of von Willebrand factor (vWF), a glycoprotein
secreted by capillary endothelial cells that acts as a carrier for clotting factor VIII (FVIII). This protects
FVIII from degradation; thus, if vWF is abnormal, the amount of FVIII is low and the patient's coagulation
ability is reduced.

Usually symptoms are fairly mild; in many cases menorrhagia may be the only symptom a woman
experiences. Since the woman in this question is about to give birth, it is very important to know the
reason for her prolonged bleeding time so that it can be treated. While levels of FVIII and vWF often
increase during the second and third trimesters, casuing symptoms of von Willebrand disease to subside,
the fact that this patient has a prolonged bleeding time puts her at risk of the condition is not managed.

Assays for vWF antigen (choice A) test for the presence and concentration of vWF in the blood. vWF is the
antigen that is detected by antibodies using various methods. While a low or absent vWF indicates von
Willebrand disease (specifically vWD type 1, a quantitative deficiency) a normal vWF antigen test cannot be
used to rule out the disease. This is because abnormally functioning vWF may bind the antibodies used in
the assay just as well normal vWF binds them. For this reason, a von Willebrand factor activity test is
performed. Also known as a ristocetin cofactor test, this assay indicates how well a patient's vWF actually
functions. If a patient has a normal concentration vWF antigen, but an abnormal vWF activity test, this
indicates von Willebrand disease type 2, a functional deficit.

91
Evaluating FVIII itself (choice B) is appropriate when the suspicion is high for hemophilia A which is an
FVIII deficiency. Not only is this condition X-linked, making the incidence very low in women, but is much
more severe than most cases of von Willebrand disease. Thus, for a woman with hemophilia, bleeding
problems would not be limited to her menstrual periods.

The level of G6PDH (choice C) is tested to determine whether the patient suffers from G6PDH deficiency.
Also known as G6PD deficiency, G6PDH deficiency is an hereditary disorder in which the body does not
produce enough of the enzyme glucose-6-phosphate dehydrogenase (G6PD or G6PDH). Since RBCs depend
on this enzyme for protection against oxidative stress, RBC hemolysis can be triggered by certain drugs
such as trimethoprim-sulfamethoxazole, nitrofurantoin, and antimalarial drugs primaquine, chloroquine,
pamaquine, and pentaquine, or by other agents, such as fava beans.

Present in approximately 400 million people globally, G6PDH deficiency is the most common disease-
producing enzyme disorder. However, since G6PDH deficiency is a recessive disease carried on the X-
chromosome, females more often are carriers and not affected themselves by the disease. Since the
condition manifests with symptoms of hemolysis such as jaundice and anemia, it is not a condition that
ought to be suspected in this woman.

The purpose of a Coomb's test (choice D), also known as an anti-globulin test (AGT), is to check for the
presence of autoantibodies against red blood cells. A direct Coomb's test determines whether IgG
immunoglobins (antibodies) are bound to the membranes of a patient's own RBCs. It is used as a test for
auto-immune hemolytic anemia. False positives are possible, if the patient has received a transfusion
within 3 months. An indirect Coomb's test determines whether unbound immunoglobins capable of binding
RBCs are present in a patient's blood. This is useful to know in cases when the blood of two different
people will be interacting, as in transfusions. It also is useful during pregnancy in a mother who is rh
negative who may be carrying a fetus who is rh positive as she may produce antibodies against the red
blood cells of the fetus, resulting in hemolysis. This is not an issue, however, during a first pregnancy.

78.

A 23 year old woman presents to your office for a prenatal visit. She has not received and previous
prenatal care and does not know the date of her last menstrual period. On physical examination, the
fundal height is palpated to at the level of the umbilicus.

Which of the following is the estimated number of weeks of gestation?

10wk

15wk

20wk

25wk

30wk

Explanation

The correct answer is choice C.

The fundus is the name for the portion of your uterus where the baby is carried for the pregnancy. Fundal
height (McDonald’s rule) is a measurement that caregivers use as an indicator of how well a pregnancy is
progressing. Prior to the invention of ultrasound technology, fundal height measurement was relied on
heavily to determine how well the baby was growing. Fundal height is measured once the baby bulge
becomes apparent, typically sometime in the second trimester. The fundal height is determined by
measuring the distance from the pubic symphysis to the highest part of the uterus using a tape measure.
The top of the fundus is felt by gently manipulating the abdomen. The measurement, in centimeters, is

92
recorded on each prenatal visit. As the baby grows larger, the fundal height measurement increases, and
shows that the baby is growing as expected.

A general rule of thumb while measuring the fundal height is that it should measure one centimeter per
week of pregnancy +/- 1 to 3 centimeters. The symphysis-fundal height approximates to the number of
weeks of pregnancy up to 36 weeks.

The height of the uterus is midway between the symphysis pubis and umbilicus at 16th week; at the level
of umbilicus around 24th week (choice D) and at the junction of the lower third and the upper two-third of
the distance between the umbilicus and ensiform cartilage at 28th week. To determine the fundal height in
the last trimester, the distance between the umbilicus and the ensiform cartilage is divided into 3 equal
parts. The fundal height corresponds to the junction of the upper and middle third at 32 weeks; up to the
level of ensiform cartilage at 36 weeks; and it comes down to 32 week level at 40th week because of
engagement of the presenting part. To know whether the height of the uterus corresponds to 32 weeks or
40 weeks, engagement of the head should be tested. If the head is floating, it is 32 weeks pregnancy and
if it is engaged, the pregnancy is of 40 weeks.

Abnormal fundal height may be due to no particular reason or it may be an indication of certain conditions
like:

 A baby who has dropped down into the pelvis when it is healthy but physically small
 Transverse lie
 Breech presentation
 Multiple pregnancy, twins, triplets or more
 Oligohydramnios
 Polyhydramnios

The above conditions are ruled out by an ultrasound. The latter also reveals if the baby is underdeveloped
or is large for gestational age.

79.

A 21-year-old G2P1 woman feels that she is large for dates. She has felt fetal movement for the past
week. She has a screening ultrasound scan performed at 16 weeks gestation. This ultrasound reveals that
there is markedly increased amniotic fluid (polyhydramnios).

93
Which of the following fetal abnormalities is most likely to cause this finding?

Renal agenesis

Ventricular septal defect

Cytomegalovirus infection

Esophageal and intestinal atresias

Anencephaly

Explanation

The correct answer is choice D

Polyhydramnios (polyhydramnion, hydramnios) is a medical condition describing an excess of amniotic fluid


in the amniotic sac. It is seen in 0.2 to 1.6% of pregnancies. It is typically diagnosed when the amniotic
fluid index (AFI) is greater than 20cm ( ≥ 20cm).

The causes of polyhydramnios is complex and includes many causes such as gestational diabetes, twin-
twin transfusion syndrome, fetal anomalies and in 65% of cases, for unknown reasons. Fetal anomalies
related to polyhydramnios include:

 gastrointestinal abnormalities such as esophageal atresia, duodenal atresia, facial cleft, neck
masses, and tracheoesophageal fistula
 fetal renal disorders that results in increased urine production during pregnancy, such as in
antenatal Bartter syndrome
 chromosomal abnormalities such as Down's syndrome and Edwards syndrome
 neurological abnormalities such as anencephaly, which impair the swallowing reflex

Nearly all cases of esophageal atresia and intestinal atresias cause polyhydramnios. On ultrasound, one
faiis to see a fetal stomach bubble. A transient anechoic tubular area may be observed in the midline of the
fetal neck, distending during fetal swallowing. Prenatal sonogram diagnoses esophageal atresia by the
presence of polyhydramnios and fetal stomach that is absent or demonstrates a decrease in filling. Fetuses
with polyhydramnios are at risk for a number of other problems including cord prolapse, placental
abruption premature birth and perinatal death.

Oligohydramnios or a low amniotic fluid volume can be associated with uteroplacental insufficiency,
congenital anomalies, viral diseases, fetal hypoxia and or postmaturity sydrome. Choices A,B,C,E) It is
responsible for malpresentations, umbilical cord compression and difficult or failed external cephalic
version.

80.

An asymptomatic 32-year-old woman is found to have a 4-cm intramural fibroid tumor. This was
incidentally found during an ultrasound procedure she was undergoing for her bladder incontinence. She is
otherwise healthy on no medications.

Which one of the following statements about the likely implications of this finding is best supported by
evidence?

94
The tumor is likely to cause delay in conception

If the patient becomes pregnant, she is at increased risk of miscarriage

Treatment is not indicated

The patient is likely to develop menorrhagia

The patient should stop taking oral contraceptives

Explanation

The correct answer is choice C

Fibroids, medically known as 'leiomyomas' or simply 'myomas' are tumorous growths composed of smooth
muscle cells and admixed with fibrous cells. Smooth muscle tumors (leiomyomas or myomas) are almost
always benign tumors, with a very low chance of progress to a malignant variant (about 2-3%). Myomas
are the most frequent type of tumorous growth in the pelvic area. Most often, they occur in the walls of the
uterus (intramural myoma); in addition, they can occur underneath the peritoneal layer of the uterus
(subserous myoma), where they grow in a mushroom like shape (pedunculated) at times and can become
necrotic if twisted. Other myomas grow under the mucosal lining of the endometrial cavity within the
uterus (submucous myoma) and can cause heavy menstrual irregularities and even infertility. Less
frequently observed locations are in the walls of the cervix and subserous myomas which grow into the
ligaments along the fallopian tube.

In this case, no treatment is necessary. The intramural myoma is asymptomatic and will not interfere with
the pregnancy. Nearly all myomas are benign and can be reevaluated once the patient has finished with
her postpartum period.

The tumor is not likely to cause delay in conception (Choice A). An intramural myoma is one that is located
in the muscular wall of the uterus. As such it would not interfere with the fallopian tubes or the upward
mobility of the sperm from the cervical canal, up the uterine cavity and into the fallopian tubes, allowing
fertilization to occur.

The myomas that are likely to cause miscarriages (Choice B) are submucosal in location. As these enlarge
with estrogen stimulation, they decrease the space in the uterine cavity in which the developing fetus will
grow. The impingement on the space will cause premature labor in many pregnancies that do not abort
early on.

Menorrhagia (choice D) is associated with submucosal myomas by two mechanisms. A thin endometrium
over the fibroid surface may not respond normally to hormonal fluctuations. It may also ulcerate or
necrose and bleed directly into the endometrial cavity.

Oral contraceptive use (Choice E) is controversial. Many believe that it has no effect although others
believe that it could stimulate the growth of the myomas directly.

81.

The second pregnancy for a 30 year old female is unremarkable. An ultrasound performed at 18 weeks
gestation revealed a placenta on the posterior fundus of the uterus, a fetus with no anomalies, and normal
amniotic fluid volume. However, at 37 weeks, the mother has the sudden onset of severe lower abdominal
pain and there is decreased fetal movement. She passes blood per vagina.

Which of the following complications has occurred:

95
Placenta previa

Villitis

Placental infarction

Abruptio placenta

Placenta accrete

Explanation

The correct answer is choice D

Abruptio placenta is a complication of pregnancy, typically seen in the third trimester with vaginal bleeding
(which may be hemorrhage depending on the extent of abruption), uterine rigidity (increased uterine tone)
pelvic and abdominal pain. This can occur anytime after the twentieth week of gestation.

The bleeding may be confined to the uterus as in a retroplacental hematoma with no external bleeding
(concealed hemorrhage with increasing fundal height) or where blood escapes into the vagina with external
haemorrhage resulting. Maternal signs include vaginal bleeding, shock (if severe), uterine tenderness and
rigidity.

The cause is a separation of the placenta from the wall of the uterus. While the cause is unknown, there is
a much higher incidence with chronic hypertension, short cord, poor nutrition, and trauma.

Placenta previa (Choice A) is an obstetrical complication whereby the placenta is abnormally attached to
the wall of the uterus causing an obstruction to the cervical os and preceding the fetus in the birth canal.
It can be marginal, total or partial in relation to the extent of obstruction. This is one of the most common
causes of third trimester bleeding and is usually painless.

The etiology is unknown but may be associated with:

 advanced maternal age


 history of previous cesarean section
 smoking

Villitis (Choice B) is a pathological finding of placental injury affecting 5-15% of placentas. It is a cause of
intrauterine growth restriction and recurrent reproductive loss. One can see large fetal vessels in the
placenta (obliterative fetal vasculopathy) and is caused by maternal T lymphocytes inappropriately gaining
access to the villous stroma. Fetal antigen presenting cells (Hofbauer cells) expand.

Placental infarction (Choice C) is a degeneration and tissue death with replacement of fibrin scar tissue
caused by interference with maternal circulation as the pregnancy progresses. The fetus usually is not
affected by infarction of the placenta unless the process is extensive.

Placenta accrete (Choice E) is an obstetrical complication whereby implantation of the placenta penetrates
beyond the endometrium into the myometrium. This does not affect the fetus but can cause hemorrhage
and catastrophe intrapartum and postpartum. Incidence increases when the placenta implants at the
lower uterine segment and also with previous cesarean sections.

82.

A 19 year old primigravid woman is expecting her first child; she is 12 weeks pregnant by dates. She
presents with vaginal bleeding and an enlarged uterus when her dates are considered. In addition, no fetal
heart sounds are heard. An ultrasound is obtained.

What is the next most appropriate intervention:

96
weekly hCG titers

hysterectomy

single agent chemotherapy

combination chemotherapy

radiation therapy

Explanation

The correct answer is choice A

In a patient with vaginal bleeding and a uterus that is larger for dates, one must suspect and rule out
trophoblastic disease (hydatidiform mole or molar pregnancy). The first step after the examination is a
sonogram to visualize the uterus. Differential includes a complete abortion with inaccurate dates, and
multiple pregnancy with demise. If a mole is seen on sonogram it has a very characteristic picture of
"grape-like" cyst clusters. Should this be the case, a dilatation and curettage would be done to evacuate
the uterus. The diagnosis of trophoblastic disease consists of the following:

 lack of fetal movement


 pelvic examination
 ultrasound
 blood test for high levels of HCG

Presumably, the uterus was seen to be empty in this case. Weekly HCG titers should be done to see if the
molar tissue (which is no longer present) has microscopically reappeared or grown in addition to screening
for trophoblastic cells spread to other areas of the body (i.e. lungs).

Molar pregnancies usually present with painless vaginal bleeding in the fourth to fifth month of pregnancy.
The uterus may be larger than expected, or the ovaries may be enlarged. There may also be more
vomiting than would be expected (hyperemesis). Sometimes there is an increase in blood pressure along
with protein in the urine. Blood tests will show very high levels of human chorionic gonadotropin (hCG).

Hydatidiform moles should be treated by evacuating the uterus by uterine suction or by surgical curettage
as soon as possible after diagnosis, in order to avoid the risks of choriocarcinoma. Patients are followed up
until their serum human chorionic gonadotrophin (hCG) level has fallen to an undetectable level. Invasive
or metastatic moles (cancer) may require chemotherapy and often respond well to methotrexate. The
response to treatment is nearly 100%. Patients are advised not to conceive for one year after a molar
pregnancy. The chances of having another molar pregnancy are approximately 1%.

No further radical treatment is necessary at this time.

83.

A 56-year-old G0 P0 otherwise healthy woman reports intermittent vaginal bleeding over the past 2
months. Her last menstrual period was 6 years ago, and her last Pap smear, one year ago, was normal.
On physical examination there are no abnormal findings. She is taking alprazolam for anxiety but no other
medications.

All of the following statements are true about endometrial cancer EXCEPT:

97
The five-year survival rate for endometrial cancer that is localized to the uterus is approximately
66%.

Women with Lynch syndrome (hereditary nonpolyposis colorectal cancer) have an increased risk
of developing endometrial cancer.

Endometrial cancer is the most common gynecologic cancer in postmenopausal women.

Endometrial cancer is found in up to 20% of postmenopausal women with uterine bleeding.

An endometrial biopsy is warranted in all postmenopausal women with uterine bleeding.

Explanation

The correct answer is Choice A.

Uterine bleeding in a postmenopausal woman is never normal and always warrants further diagnostic
evaluation. Vaginal bleeding in a postmenopausal woman raises the suspicion of endometrial cancer or
endometrial hyperplasia and must be evaluated to rule out these conditions. Up to 20 percent of
postmenopausal women with uterine bleeding will have endometrial cancer. An endometrial biopsy is
indicated and can generally establish the diagnosis.

In women who are still ovulating, abnormal uterine bleeding is usually due to one of the following:

 abnormal bleeding of pregnancy


 fibroids (leiomyomata)
 polyps
 adenomyosis
 blood clotting disorders
 systemic lupus erythematosus
 hypothyroidism
 medications that interfere with blood clotting

It is important to establish the source of vaginal bleeding. While cervical lesions and cervical cancer can
present with vaginal bleeding, it is a less likely cause of bleeding in a postmenopausal woman than
endometrial cancer. Endometrial carcinoma is the most common gynecologic cancer in women over 45
years of age. The history of a normal Pap smear in the last year is also a factor that suggestes that the
bleeding is likely not of cervical origin.

Endometrial cancer is a common gynecologic malignancy that accounts for 6% of all cancers in women.
Women have a 2.5 percent lifetime risk of developing endometrial cancer. Most cases are diagnosed at an
early stage when surgery alone may be curative. The five-year survival rate for localized disease is 96%,
with regional disease having a 66% five-year survival rate. Metastatic disease at the time of presentation
is associated with a 24% survival rate. The mean age at diagnosis is 60 years.

The strongest known risk factor for the development of endometrial cancer is excess estrogen, either from
exogenous or endogenous sources, in the absence of adequate exposure to progestins. Excess estrogen
from exogenous sources includes both estrogen therapy and tamoxifen, while endogenous excess estogen
may result from obesity, anovulatory cycles, or estrogen-secreting tumors. Oral contraceptive pill (OCP)
use has been shown to decrease the risk of endometrial cancer by 50 to 80 percent. This protective effect
lasted for at least 15 years after cessation of OCPs.

Women with Lynch syndrome (hereditary nonpolyposis colorectal cancer) have a higher-than-normal risk
of developing endometrial cancer. In these women, the lifetime risk of developing endometrial cancer is 27
to 71 percent.

Total hysterectomy with bilateral salpingo-oophorectomy is recommended for staging and initial

98
management of endometrial cancer.

84.

A 26-year-old woman comes to the clinic for a routine pelvic examination and a Pap smear. She has had
regular yearly pelvic exams and Pap smears since age 22, and they have all been normal. She has one
current male sexual partner and has had 5 partners overall in her lifetime. She has no known history of
STDs. The current Pap smear shows atypical squamous cells of undetermined significance (ASC-US).

What is the appropriate next step in management of this patient?

Reassure the patient that, since all her previous Pap smears have been normal, she can have
another Pap test in 1 year.

Refer her for conization of the cervix because her history of multiple partners places her at
increased risk for cervical cancer or precancerous changes.

Perform colposcopy with four quadrant cervical biopsy and endocervical curettage if no lesion is
seen.

Test for high-risk subtypes of human papillomavirus (HPV) and, if positive, perform colposcopy.

Recommend that Pap smears be repeated 2 times per year for the ensuing 3 years.

Explanation

The correct answer is Choice D.

Atypical squamous cells (ASC) are classified as either ASC-US, which means that the atypical cells are "of
undetermined significance," or ASC-H, in which a high grade squamous intraepithelial lesion (HSIL) cannot
be excluded. ASC of either type always requires further evaluation to exclude the presence of a
precancerous or cancerous lesion.

An appropriate next step in this case is testing for "high-risk" HPV types. In reflex HPV testing, a specimen
for HPV testing is collected when the cytology sample is collected, but the HPV test is only performed after
the Pap smear results are obtained. This obviates the need for the patient to return to the clinic for HPV
testing yet avoids unnecessary testing if the Pap smear results are negative. Colposcopy is then performed
on women with ASC-US who test positive for high risk HPV types.

Because the cell changes of ASC-US can ofte be caused by low hormone levels, applying an estrogen
cream to the cervix for a few weeks can usually help to clarify the cause of the cell changes.

If the Pap test shows a finding of ASC–H, LSIL, or HSIL, coloposcopy is generally recommended.

Table 1: Summary of Pap results and follow-up testing that is required

99
85.

A genetic counselor elicits the history that three adult males and one adult female in a family of 10 over 3
generations are mentally retarded, and the males more severely so. Physical examination of these
affected males reveals no major morphologic anomalies, though their testes appear to be slightly
enlarged, without mass lesions present. These males have been healthy, without a history of major
illnesses or relevant drug exposure.

Which of the following genetic abnormalities is the most likely etiology for these findings?

Klinefelter syndrome

Gaucher disease

Fragile X syndrome

Phenylketonuria

Down syndrome

Explanation

The correct answer is choice C

Fragile X syndrome is a group of genetic conditions related by gene changes in the FMR1 located on the X-
chromosome. The pattern of inheritance has been found to be X-linked dominant with males being more
predominantly affected. It is the most common cause of inherited mental impairment ranging from mental
retardation to learning disabilities. It is the most common cause of autism and delays in speech with poor
language development. Fragile X can be passed down by individuals in a family who have no signs of this

100
genetic condition.

Aside from intellectual disability, prominent characteristics of the syndrome include an elongated face,
large or protruding ears, flat feet, larger testes (macroorchidism), and low muscle tone. Speech may
include cluttered speech or nervous speech. Behavioral characteristics may include stereotypic movements
(e.g., hand-flapping) and atypical social development, particularly shyness, limited eye contact, memory
problems, and difficulty with face encoding. Physical characteristics include the following:

 Prominent ears (one or both)


 Long face (vertical maxillary excess)
 High-arched palate (related to the above)
 Hyperextensible finger joints
 Double-jointed thumbs
 Flat feet
 Soft skin
 Larger testes in men (macroorchidism)
 Low muscle tone

In Klinefelter Syndrome (Choice A) males have an extra X chromosome(XXY males). The main symptoms
are small testicular development and infertility. While there may be some language learning disability,
many show no signs of mental impairment. Associated symptoms include gynecomastia and rounded body
type.

Gaucher disease (Choice B), known as a storage disease, is an inherited metabolic disorder that leads to
lipid accumulation throughout the body due to a deficiency of the enzyme acid beta-glucocerebrosidase
which is needed to metabolize glucosyl ceramide. The lipid can gather in the spleen, bone marrow, brain,
liver and lungs. In type 1, the bones are affected and in Type 2 where the brain is involved, newborns die
at birth or within 18 months.

Phenylketonuria (Choice D) is an autosomal recessive metabolic disorder where a deficiency in


phenylalanine hyroxylase causes the body to improperly break down the amino acid
phenylalanine.Although rare, babies are screened for this from birth to avoid health risk. Asymptomatic at
birth, if undetected and no treatment is given, leads to:

 behavioral/social problems
 seizuers
 mental retardation
 stunted growth
 skin rashes
 microcephaly
 musty breath

In Down's Syndrome, (Choice E) caused by trisomy 21, have both physical and mental symptoms which
range from mild to severe. Problems include:

 cardiac disease
 dementia
 hearing problems
 skeletal problems
 thyroid dysfunction

 intestinal and eye problems

86.

101
A 4-year-old girl is brought to her primary care physician because of perineal itching. Her mother states
that she has been scratching mainly at night, and that the itching is interfering with sleep. On
examination, the patient is noted to have erythema and excoriations in her perianal area as well as mild
vulvovaginal erythema with a scant white discharge.

Which of the following is the most likely diagnosis?

Candidiasis

Enterobiasis

Pediculosis pubis

Physiologic leukorrhea

Trichomoniasis

Explanation

The correct answer is choice B

Enterobiasis, commonly known as Pinworm infection, is common in children. Nocturnal perineal itching is a
common complaint and excoriation from scratching may be seen and is caused by the female pinworms
migrating to lay eggs around the anus. One third of individuals with pinworm infection are totally
asymptomatic. Apply tape to the anal region, and then apply to a glass slide and examine microscopically
for Oxyuris. Advise mother that the child should wash her hands and nails after each defecation.
Underpants must be boiled.

Although hygiene plays a role, medication is the chief treatment. Treatment consists of the benzimidazole
compounds albendazole (brand names e.g., Albenza, Eskazole, Zentel and Andazol) and mebendazole
(brand names e.g., Ovex, Vermox, Antiox and Pripsen) which are the most effective. Other medications
including piperazine citrate, 1 gm orally twice daily for 7 days. Ammoniated mercury ointment 1% can also
be applied to the perianal region BID for itching relief. Regardless of the medication used, reinfection is
frequent.[3] Asymptomatic infections, often in small children, can serve as reservoirs of infection, and
therefore the entire household should be treated regardless of whether or not symptoms are present

Candidiasis (Choice A) could be considered initially but would manifest with inflammation or reddening of
the vulva and vaginal area. Diagnosis can be made by performing a wet preparation of the discharge and
10% KOH.

Pediculosis Pubis (Choice C) is caused by the crab louse and attaches to hirsuite skin of the pubis, axilla or
scalp. This is not applicable to this patient due to her age and lack of pubic hair.

Physiologic leukorrhea(Choice D) is a clear mucoid vaginal exudate and is asymptomatic.

Trichomoiasis (Choice E) is one of the most frequent sexually acquired diseases. Diagnosis is often made
by viewing profuse, foul smelling vaginal discharge which tends to be malodorous, frothy and greenish in
color. Itching is also prevalent with inflammation of the external genitalia.

87.

A 38-year-old healthy woman has had a white, curd-like vaginal discharge for the past week. There is no
bleeding. She denies having fever and abdominal pain. A Pap smear demonstrates normal appearing
squamous epithelial cells along with scattered neutrophils and budding cells with pseudohyphae.

All of the following statements concerning the organism responsible for this infection are true EXCEPT:

102
The organism typically produces an odorless discharge.

Overgrowth of the organism may occur when there is immunosuppression or disruption of the
normal vaginal flora.

Topical treatments are generally effective against this organism.

Up to 10% of healthy women carry the organism in their vaginal flora.

Vaginal douching may increase a woman's chance of developing an infection with this organism.

Explanation

The correct answer is Choice D.

The presence of budding cells with pseudohyphae seen microscopically in this case is diagnostic of vaginal
Candidiasis. Candida albicans is the type of yeast most commonly responsible for vaginal yeast infections.
Yeast vaginitis is the most common type of vaginitis and accounts for about one-third of cases of vaginitis.

Up to 50% of healthy women have been shown to carry Candida species as part of the normal vaginal
flora. Vaginal yeast infections occur when new yeast is introduced into the vaginal area or when there is an
increase in the quantity of yeast already present in the vagina relative to the quantity of normal bacteria,
such as when antibiotics are taken for an unrelated condition.

Candidiasis can also occur as a result of injury to the vagina, such as after chemotherapy, or in cases of
immune suppression. Other conditions that may predispose women to developing vaginal yeast infections
include diabetes mellitus, pregnancy, and the use of oral contraceptives. The use of douches or perfumed
vaginal hygiene sprays may also increase a woman's risk of developing vaginal Candidiasis.

Although vaginal discharge is not always present, when it occurs, it is typically is odorless with a whitish,
thick appearance and texture, as observed in this case. Itching, burning, soreness, dyspareunia, and pain
in the vaginal area are other potential symptoms of Candidiasis.

Vulvovaginal Candidiasis can generally be treated with topical antifungal medications, such as miconazole,
tioconazole, butoconazole, and clotrimazole.

88.

A 48-year-old woman has noted a small amount of irregular vaginal bleeding for the past 2 months. She
has a pelvic examination that reveals no cervical lesions, and a Pap smear that shows no abnormal cells.
Next, an endometrial biopsy is performed, and there is microscopic evidence for endometrial hyperplasia.
An abdominal ultrasound reveals a solid right ovarian mass.

Which of the following estrogen-producing neoplasms is this woman is most likely to have?

Mature cystic teratoma

Choriocarcinoma

Sertoli-Leydig cell tumor

Granulosa cell tumor

Cystadenocarcinoma

103
Explanation

The correct answer is Choice D.

Granulosa cell tumors belong to the group of sex cord-stromal tumors of the ovary, a heterogeneous group
of benign or malignant tumors that develop from the cells surrounding the oocytes. Sex cord-stromal
tumors make up only about 5-8% of primary ovarian neoplasms.

Many ovarian sex cord-stromal tumors produce steroid hormones. The diagnosis of such a tumor must be
considered in women who have signs of estrogen excess (precocious puberty, abnormal uterine bleeding,
endometrial hyperplasia or carcinoma) or androgen excess (virilization), especially if the patient has an
adnexal mass. In this case, the patient presents with both evidence of estrogen excess (endometrial
hyperplasia) and an adnexal mass.

As with other ovarian neoplasms, surgery is necessary to definitively establish the diagnosis and to confirm
whether the lesion is benign or malignant. As with epitheilal ovarian neoplasms, sex cord stromal tumors
must be surgically staged.

Granulosa-stromal cell tumors include granulosa cell tumors, thecomas, and fibromas. These tumor
types account for 70 percent of ovarian sex cord-stromal tumors. Among granulosa-stromal cell tumors,
fibromas are the most common and occur equally in pre- and postmenopausal women. Granulosa cell
tumors, thecomas, and mixed tumors usually produce hormones, whereas fibromas typically do not.

Among the malignant ovarian sex-cord stromal tumors, granulosa cell tumors are the most frequent
histologic type.

89.

A 33-year-old G3 P2 woman gives birth at 38 weeks gestation to a male infant who weighs 2270 gm.
Apgar scores are 7 and 9 at 1 and 5 minutes after birth. This neonate has no external anomalies present
on physical examination except that he appears somewhat disproportionate, with a head size more
appropriate for gestational age than body size. The pregnancy was uncomplicated.

Which of the following conditions is most likely to account for these findings?

Trisomy 18

Niemann-Pick disease

Gestational diabetes

Congenital rubella

Maternal tobacco use

Explanation

The correct answer is choice E

Tobacco use in the pregnant patient is associated with small for gestational age infants (SGA). Smoking
during pregnancy has actually been associated with several complications:

 low birth weight


 premature rupture of membranes
 placenta previa
 placental abruption

104
 preterm birth

Carbon monoxide has an affinity for hemoglobin, 200 times greater than oxygen and even higher affinity
than fetal hemoglobin causing a decreased in oxygen-carrying capacity.

Trisomy 18 also known as Edward's Syndrome (Choice A) is a rare genetic chromosomal syndrome
associated with cardiac malformations.Most fetuses abort before term. It causes mental retardation if born
at term and numerous physical defects that cause early infant death. Symtoms of Edward's Syndrome
include:

 prominent occiput
 mental retardation
 micrognathia
 low set ears
 rocker bottom feet
 congenital heart disease
 low birth weight
 poor feeding and week cry
 failure to thrive
 prominent back of skull
 exgtreme rigidity
 hypoplastic nails
 small pelvis
 short sternum
 single umbilical artery
 small mouth
 pulmonary stenosis
 patent ductus areteriosus
 cleft palate
 choanal atresia
 hypospadias
 limb defects

Niemann-Pick disease (Choice B) is a rare condition that causes fat to accumulate in organs. Symptoms
include:

 liver enlargement
 spleen enlargement
 difficulty walking and swallowing
 progressive vision and hearing loss
 growth delay
 delayed puberty
 mental retardation
 spasticity
 jaundice
 ataxia
 hepatic failure
 recurrent vomiting
 myoclonic jerks

Babies associated with Gestational Diabetes (Choice C) are larger than gestational age when the diabetes
has been uncontrolled.

Babies born with Congential rubella (Choice D) have microcephaly (a smaller than normal-sized

105
head). Additional symptoms include:

 cloudy corneas
 deafness
 developmental delay
 excessive sleepiness
 irritability
 low birth weight
 mental retardation
 seizures

 skin rash at birth

90.

A 35-year-old gravida 1, para 0 woman who is otherwise healthy undergoes an amniocentesis after the
detection of an abnormality by ultrasound. The ultrasound results showed the classic “lemon sign” of the
cranium seen with open neural tube defects.

What will be the most likely karyotype be for this fetus?

45,XO

46,XX + 21

46,XX t(q21;14)

46,XX

46, XYY

Explanation

The correct answer is choice D.

The "lemon sign" refers to shape of the fetal skull at ultrasonography (US) when the frontal bones lose
their normal convex contour and appear flattened or inwardly scalloped, giving the skull a shape similar to
that of a lemon. It is strongly associated with open spina bifida and is very useful in the detection of spina
bifida in a high-risk population before 24 weeks of gestation. However, it is not absolutely specific for spina
bifida and had been observed with other fetal malformations.

While karyotypic abnormalities can be associated with open neural tube defects, the majority of affected
with spina bifida have a normal karyotype. Still, because there is a high prevalence of karyotypic
abnormalities among fetuses with neural tube defects, the use of fetal karyotyping as an aid in diagnostic
evaluation and recurrence risk counseling is warranted.

The American College of Medical Genetics recommends use of maternal serum alpha-fetoprotein and/or
ultrasound for detection of neural tube defects between 15 and 20 weeks of gestation.

91.

A 16-year-old girl presents with right shoulder pain and nausea for the past 2 days. She thinks that she
injured her shoulder when she was lifting heavy boxes last week. She has been taking ibuprofen, without
much relief. On further questioning, she reports mild, intermittent, crampy lower abdominal pain for the
past few days. She also says that gets cyclical pains in her right abdomen with every period for the last 1

106
year. She denies fever, dysuria, vaginal bleeding, or discharge. She is sexually active and thinks that her
last menstrual period was 4 or 5 weeks ago. She is not taking any medications.

Vital signs are: BP, 105/70 mm Hg; heart rate, 97 beats/min; respiratory rate, 20 breaths/min. Physical
examination findings are unremarkable, except for mild tenderness in the right lower quadrant, but no
rebound or guarding. Her right shoulder examination is normal. During preparations for a pelvic
examination, the patient becomes hypotensive and tachycardic.

What is the most likely diagnosis for this patient?

Right ovarian follicular cyst rupture

Ruptured ovarian cyst

Appendicitis

Ruptured ectopic pregnancy

Renal colic

Explanation

The correct answer is Choice D.

Although all the conditions listed are possible causes of many of this patient's symptoms, her presentation
is most characteristic of ruptured ectopic pregnancy. Lower abdominal pain, shoulder pain, and potentially
late menstrual period are all suggestive of ectopic pregnancy; the abrupt-onset hypotension and
tachycardia are consistent with recent rupture.

Symptoms of ectopic pregnancy include abnormal vaginal bleeding, missed menstrual period, low back
pain, breast tenderness, lower abdominal pain and/or cramping, nausea, and pelvic pain. If rupture occurs,
serious and potentially life-threatening symptoms can develop. These may include fainting, pressure/pain
in the pelvis or rectum, shoulder pain, and severe pain and cramping in the lower abdomen.

The greatest risk factor for an ectopic pregnancy is a prior history of an ectopic pregnancy. The recurrence
rate is 15% after the first ectopic pregnancy, and 30% after the second. Other risk factors include
disruption of the normal architecture of the Fallopian tubes, including previous surgery or reconstructive
procedures. Infection, congenital abnormalities, or tumors of the Fallopian tubes can all disrupt Fallopian
tube archtitecture and increase a woman's risk of having an ectopic pregnancy.

Infection in the pelvis is another risk factor for ectopic pregnancy as well as infection-related scarring and
partial blockage of the Fallopian tubes. Like pelvic infections, conditions such as endometriosis, fibroid
tumors, or pelvic adhesions, can increase the chances of an ectopic pregnancy.

Approximately 50% of pregnancies in women using intrauterine devices (IUDs) will be located outside of
the uterus. However, the total number of women becoming pregnant while using IUDs is extremely low.
Cigarette smoking around the time of conception has also been associated with an increased risk of ectopic
pregnancy.

92.

A 42 year old G2P2 woman presents with the chief complaint of severe bilateral breast pain that seems
worse around the time of her menstrual periods. Physical examination reveals bilateral breast
tenderness upon palpation as well as multiple lumps in both breasts. A mammogram is performed and
reveals dense bilateral breast tissue.

Which of the following is the most likely diagnosis in this patient?

107
Fibroadenoma

Fibrocystic breast condition

Paget’s disease

Mastitis

Mammary duct ectasia

Explanation

The correct answer is choice B.

Fibrocystic breast condition, also known as fibrocystic breast disease or fibrocystic changes, is a common
and benign condition that predominantly affects premenopausal women between the ages of 30 and
50. Fibrocystic breast condition is the most common cause of breast lumps in women, and it affects more
than 60% of women. The condition is less pronounced after menopause.

Fibrocystic breast condition varies significantly in severity, which can complicate the diagnostic process.
Some women may have fibrocystic breast condition with very mild symptoms of breast tenderness or pain.
In some women, the ysmptoms may occur constantly, while in others, they may occur only during the
premenstrual phase. Other women with the condition may experience severe and constant pain and
tenderness throughout both breasts.

Palpation of the breasts is useful in the establishment of a diagnosis, although the differentiation from
malignant breast masses is essential. This lumps of fibrocystic breast condition are most commonly noticed
in the upper outer quadrant of the breast. The nodules felt in fibrocystic breast condition are typically
mobile, rounded, have smooth borders, and may feel rubbery or somewhat changeable in shape.
Sometimes, the fibrocystic areas may feel irregular, ridge-like, or like tiny beads. These

It can be difficult to interpret mammogram results in women with extremely fibrocystic breasts. In these
cases, breast ultrasound exams can be helpful. A biopsy may be necessary to rule out the presence of
malignancy in uncertain cases.

A fibroadenoma (choice A) is a solitary lesion and would not present with multiple lumps in both breasts.

Paget's disease of the nipple (choice C) is characterized by redness, scaling, and eventually destruction of
the skin of the nipple. It is associated with an underlying breast carcinoma in the majority of cases.

Mastitis (choice D) occurs most frequently in lactating women and presents as a firm, tender area.

Likewise, mammary duct ectasia (choice E) typically results in a localized, firm, tender area.

93.

A woman who is positive for hepatitis B surface antigen (HBsAg), but negative for hepatitis B antigen
(HBeAg), delivers at term. The delivery was uncomplicated although mother was found to be GBS positive
and treatment was started accordingly. She also suffered from pregnancy induced hypertension which has
been appropriately controlled. She recalls contracting the hepatitis as a young adult via IV illicit drug use.
She has cleaned up recently and she denied drug use during pregnancy.

What would be the best management for this woman’s infant?

108
administer gamma globulin intramuscularly immediately and at 1 month of age

administer hepatitis B (HB) vaccine immediately and at 1 month and 6 months


of age

administer hepatitis B immune globulin (HBIG) if cord blood is positive for


HbsAg

HBIG and HB vaccine immediately, and HB vaccine again at 1 and 6 months

advise mother that breastfeeding is contraindicated

Explanation

The correct answer is choice D

The hepatitis B surface antigen determines whether the infection is acute or chronic. Since this was
positive with a negative HBeAG, the infection is not chronic but rather due to recent exposure. Therefore,
one would administer HBIG and HB vaccine immediately, followed by vaccine again at 1 and 6 months.

HBIG is a blood plasma product that can prevent hepatitis B if given within 14 days of exposure. The
protocol for hepatitis vaccination would then be enacted because the protective effect of HBIG is transient.

Vertical transmission from the infected mother to the newborn baby is a common mode of transmission in
the developing world. The infant should be given hepatitis B immunoglobulin (HBIG) 0.06 mL/kg IM—
equivalent to antibody against hepatitis B—and hepatitis B vaccine immediately after delivery and the
vaccine series should be completed (3 total doses- 0, 1 month, 6 months). Both of these should be
administered within 12 hours of birth. Testing for HBsAg and Anti-HBs should be done at age of 9 to 15
months of age to assess the carrier state or immunity—presence of HBsAg indicates carrier state and that
of Anti HBs confers success of vaccination.

10-20% of women seropositive for HBsAg transmit the virus to their neonates in the absence of
immunoprophylaxis. In women who are seropositive for both HBsAg and HBeAg, vertical transmission is
approximately 90%. In patients with acute hepatitis B vertical transmission occurs in up to 10% of
neonates when infection occurs in the first trimester and in 80 -90% of neonates when acute infection
occurs in the third trimester. Chronic infection occurs in about 90% of infected infants, 30% of infected
children aged <5 years , and 2%--6% of adults. Among persons with chronic HBV infection, the risk of
death from cirrhosis or hepatocellular carcinoma is 15%--25%.

HBV infection does not appear to be cause birth defects, but there appears to be a higher incidence of low
birth weight among infants born to mothers with acute infection during pregnancy. In one small study
acute maternal hepatitis (type B or nontype B) had no effect on the incidence of congenital malformations,
stillbirths, abortions, or intrauterine malnutrition. However, acute hepatitis did increase the incidence of
prematurity.

Although cesarean delivery has been proposed as a means of reducing mother to child transmission (MCT)
of HBV, the mode of delivery does not appear to have a significant effect on the interruption of HBV
maternal-baby transmission by immunoprophylaxis. Delivery by cesarean section for the purpose of
reducing MCT of HBV is note presently recommended by either the CDC or the ACOG. With appropriate
hepatitis B immunoprophylaxis, breast-feeding poses no additional risk for transmission from infected
hepatitis B virus carriers.

94.

A 37 year old nulligravid patient complains of bleeding between her periods and increasingly heavy
menses along with severe menstrual cramping, worsening over the past two years. Her menstrual periods
began at age 13 and have been regular. Her last Pap test was performed six months ago and was
negative. Therapy with oral contraceptives and ibuprofen 600 mg per day has failed to resolve her
symptoms. A urine pregnancy test performed in your office is negative.

109
Of the following, which is the most appropriate next step?

Performing a hysterectomy

Insertion of a progesterone-releasing IUD (intrauterine device)

Endometrial ablation

Endometrial biopsy

Starting the patient on a high-dose progestational agent

Explanation

The correct answer is choice D.

Abnormal uterine bleeding accounts for up to one-third of gynecologic office visits and occur due to a wide
range of causes. In ovulating premenopausal women, abnormal uterine bleeding is often due to pregnancy
or to an anatomic or structural abnormality of the uterus such as an endometrial polyp, adenomyosis, or
leiomyoma.

In nonpregnant women over 35 years of age or in women with risk factors for endometrial cancer, an
endometrial biopsy should be performed to rule out endometrial hyperplasia or carcinoma as the cause of
abnormal uterine bleeding.

A hysterectomy (choice A), may be perfomed as a definitive cure for abbormal uterine bleeding if more
conservative therapies are ineffective in women who do not desire to preserve fertility.

Choice B, insertion of a progestin-releasing IUD, is an appropriate treatment for abnormal uterine bleeding
in many women. However, a premalignant or malignant lesion of the endometrium must forst be ruled out
by endometrial biopsy.

Likewise, endometrial ablation (choice C) may be an effective management step after an endometrial
biopsy has ruled out the presence of malignancy.

Therapy with high dose progestational agents (choice E) may be successful, but it is important to rule out
malignancy as the source of bleeding in this patient prior to initiation of therapy.

95.

A 20 year old woman presents to the labour floor at 34 weeks gestation complaining of painless vaginal
bleeding. Her pregnancy has been uneventful prior to her presentation. She has no prior vaginal bleeding
and prior pregnancies were all carried to term. She is hemodynamically stable and vital signs are
appropriate and within normal range.

All of the following are appropriate steps except:

110
vaginal examination

complete blood count

crossmatch blood

ultrasound

fetal heart rate monitor

Explanation

The correct answer is choice A

Painless vaginal bleeding in the third trimester is most often due to placenta previa.
Performing a vaginal examination with the placenta obstructing the cervical os which
is dilating and preceding the fetus, runs the risk of inadvertently inserting a finger
through a portion of the placenta.

There are various causes of antepartum haemorrhage and occurs in 2-5% of all
pregnancies. These include the following:

 abruptio placenta- 40%


 unclassified- 35%
 placenta previa- 20%
 lower genital tract lesion- 5%
 other- <1%

Etiology of APH may be divided into the following:

 Cervical: contact bleeding (e.g. intercourse, pap, neoplasia, examination),


inflammation (e.g. infection), effacement and dilatation (e.g. labour, cervical
incompetence)
 Placental – abruptio, previa, marginal sinus rupture
 Vasa previa
 Other - abnormal coagulation

Diagnostic Procedures should include the following:

 History and physical - No digital pelvic exam


 Ultrasound – definitive test for previa – less useful in abruptio
 Electronic Fetal Monitoring – for fetal compromise and uterine tone•
Speculum – do ultrasound first if possible – No digital pelvic exam

Laboratory tests that you should consider:

 CBC, blood type, Rh, Coombs


 coagulation status – INR, PTT, fibrinogen or TCT
 2-4 units of PRBC cross matched as appropriate
 bedside clot test
 Kleihauer-Betke or Neirhaus test – vaginal and/or maternal blood
 fetal lung maturity indices if appropriate

Performing a sonogram (Choice D) will assist in locating the placenta and give a good

111
estimate as to the extent of the previa obstructing the os (i.e. total vs partial vs.
marginal previa)

A complete blood count (Choice B), crossmatch (Choice C) and fetal monitoring
(Choice E) should be done for the patient as soon as she is evaluated.

96.

A 22 year-old woman comes to your office complaining of lower abdominal pain, a slight vaginal
discharge, and fever since the previous day. She reports that she is sexually active and has not been
using birth control. Last year she was successfully treated for Trichomonas infection which cleared after
treatment. Pelvic examination reveals bilateral tenderness to palpation but no discrete masses. A urine
pregnancy test is negative. She is diagnosed with pelvic inflammatory disease (PID) based on her signs
and symptoms.

Which one of the following is most appropriate?

The woman's partner(s), even if asymptomatic, should be given specific or empirical treatment for
chlamydia and gonorrhea.

Laparoscopy should be performed, as it essential to establish the diagnosis of PID.

She should be admitted to the hospital and given antibiotics immediately in an effort to prevent
further infection.

If the woman's sex partner(s) have symptoms, they should be treated with antibiotics.

She should be offered infertility counseling.

Explanation

The correct answer is A.

Prompt and appropriate treatment of pelvic inflammatory disease can help prevent serious complications
including permanent damage to the female reproductive organs, infertility, an increased risk of ectopic
pregnancy, and chronic pelvic pain. Because it is difficult to identify the precise bacteria infecting the
internal reproductive organs and since more than one organism may be responsible for an episode of PID,
the usual treatment for PID involves at least two antibiotics that are effective against a wide range of
infectious agents, including Chlamydia and N. gonorrheae.

Women at greater risk for PID are those who are at greater risk for sexually transmitted infections (STIs)
and those with a prior episode of PID. Sexually active women under age 25 are at risk as well because the
cervix (opening to the uterus) of teens and young women has greater susceptibility to STIs. Douching is
another risk factor, use of intrauterine devices (IUDs) for contraception, and less commonly, gynecological
procedures or surgeries.

Although sex partners may have no symptoms, they may still be infected with the organisms that can
cause PID. Her sex partner(s) should be treated to decrease the risk of spreading the disease and possible
reinfection, even if the partner(s) has no symptoms.

The signs and symptoms of PID are often sufficient to establish the diagnosis. Typical symptoms
include fever, vaginal discharge that may have an unpleasanht odor, painful urination, painful
intercourse, irregular menstrual periods, and pain in the right upper abdomen. Ultrasound or other
imaging studies may be helpful in determining whether and abscess is present. Laparoscopy may also be
necessary to confirm the diagnosis in some cases. Infection with certain organisms, notably Chlamydia,
may be present in asymptomatic women or in women who only have mild symptoms.

112
Hospitalization is sometimes necessary to treat PID but is not always necessary. For example,
hospitalization may be recommended if the woman is severely ill, is pregnant, requires intravenous
antibiotics, or has a tubo-ovarian abscess.

97.

A 25-year-old primigravida experiences a spontaneous abortion at 13 weeks gestation. She had an


uncomplicated pregnancy up to that time. There is no family history of congenital abnormalities. Her
nutritional status is normal.

Which of the following is the most likely cause for her fetal loss?

Placental abruption

Umbilical cord torsion

Congenital syphilis

Trisomy 16

Bicornuate uterus

Explanation

The correct answer is D

First trimester pregnancy loss is most commonly caused by chromosomal abnormalities and trisomy 16 is
the most common trisomy associated with spontaneous abortion(being incompatible with life). The
karyotype is usually 47 XX+16.

Mosaic trisomy 16 is an extremely rare chromosomal event and the effects vary. However some of the
more common traits include intrauterine growth retardation and congenital heart defects.

Placental abruption (Choice A) is an obstetrical complication occurring in the third trimester with the
placenta prematurely separating from the wall of the uterus. Symptoms include vaginal bleeding and a
rigid uterus with pelvic and abdominal pain.

Umbilical cord torsion (Choice B) leading to fetal death, is an occurrence of late 2nd trimester or third
trimester in pregnancy.

Congenital syphilis (Choice C) contracted during the pregnancy leads to abnormal fetal development like
microcephaly and neurological deficits as well as a higher risk of prematurity.

A Bicornuate Uterus (Choice E) or "heart- shaped" uterus is one where a septum causes a uterine
malformation with two horns forming at the upper uterine sections. This is formed dyring embryogenesis
where the fusion process of the upper part of the Mullerian ducts is changed, resulting in a bifurcation. This
is associated with a premature delivery or pregnancy loss or malpresentation of fetus in labor.

98.

You have just received a Papanicolaou (Pap) smear report on a healthy 25-year-old woman that reads
“epithelial cell abnormality - low-grade squamous intraepithelial lesion (LSIL).” She has been your patient
for the past four years and has had yearly negative Pap smears. She has no evidence of sexually-
transmitted diseases, and a urine pregnancy test is negative.

What is the next appropriate step in management of this patient?

113
Human papilloma virus (HPV) DNA testing

Refer for colposcopy

Repeat Pap smear at 6 and 12 months

Repeat Pap smear and proceed with colposcopy if the repeat is abnormal

Perform colposcopy and endometrial biopsy

Explanation

The correct answer is Choice B.

Further evaluation of this patient is indicated because of the cellular changes observed on the Pap smear.
Colposcopy, with biopsy of suspicious lesions, is the recommended method for evaluation of LSIL, since
around 15% of women with LSIL are known to have moderate to severe dysplasia (cervical intraepithelial
neoplasia or CIN 2,3) on biopsy.

HPV testing for high-risk types is generally recommended for women with atypical squamous cells of
uncertain significance (ASC-US) on Pap smear. Women with ACS-US who also are positive for high-risk
HPV types have a risk of CIN 2,3 similar to that of women with LSIL. Colposcopy is recommended for
women with ASC-US who test positive for high risk HPV types.

Endometrial biopsy is not indicated in the management of LSIL.

Table 1: Flow Chart of Pap Smear results and treatment recommendations

114
Table 2: Summary of Pap results and follow-up testing and treatment

115
99.

The nurse calls you to evaluate a 23-year-old woman, G3P2 who delivered an 8 lb 2 oz infant 3 hours ago.
She is having heavy vaginal bleeding and passing large clots. She required oxytocin (Pitocin, Syntocinon)
for augmentation of labor and had an epidural for labor. A second-degree laceration was repaired, and she
was given an oxytocin infusion after delivery. The patient's BP is 110/74 mm Hg, and her pulse is 88 bpm.

What is the next appropriate step in management?

have the nurse continue to monitor the amount of vaginal bleeding

intramuscular injection of methylergonovine maleate (Methergine), 0.2 mg, and repeat in 5


minutes

IV hydration lines, type and crossmatch, administration of 2 units of packed red blood cells

notify the operating room team to be prepared to perform dilation and curettage

Start iron supplementation to help restore patient's blood count

Explanation

The correct answer is choice C

Postpartum hemorrhage, Affects 5-15% of women giving birth and is divided into two categories:

 Early (primary) hemorrhage: occurs within the first 24 hours postpartum

116
 Late (secondary) hemorrhage: occurs after 24 hours postpartum

Etiology of the bleed has often been referred to as the 4 T's:

 Tone: uterine atony, ~ 1 in 20 deliveries (75-85%)


 Tissue: retained placental tissue
 Trauma: uterine, cervical or vaginal lacerations
 Thrombin: dilutional coagulopathy, consumptive coagulopathy and coagulation disorders

PRIMARY MANAGEMENT –“ABC’s”

 Notify attending physician and other staff


 Monitor vital signs, urine output, possible foley
 1 large bore IV
 Type of cross-match 2-4 units of PRBC’s
 Fluid resuscitation with crystalloids
 Baseline blood work for Hgb, hematocrit, platelets
 and coagulation profile
 Then proceed with directed treatment

Although the patient's vital signs are currently stable, she is still actively bleeding heavily. There is a real
possibility that she has retained placental fragments or an atonic uterus. In addition, she may have a
cervical laceration. Until you can personally evaluate the patient, it is a good idea to have the extra
intravenous lines. The patient's condition can deteriorate quickly at any time with a sudden drop in her
blood pressure and heart rate. If this were to occur, you would want to have the extra line for a transfusion
of blood to make up the loss and stabilize her as well as having a line for possible anesthetics.

The nurse should continue to monitor the amount of vaginal bleeding (Choice A) as well as her vital signs.
However at this point, she needs further evaluation as to the cause of her heavy bleeding.

Administration of methergine (oxytocic properties) (Choice B) should be done. However, the first thing
needed is the IV line and cross match of blood.

A dilatation and curettage (Choice D) may become necessary to evacuate the uterus. However, if it is
ascertained that the patient has retained placental fragments, this may be removed manually at the
bedside without further instrumentation needed. Again, if the problem is a cervical laceration, then it is the
repair of the cervix that needs to be done, not a D&C.

Iron supplementation will not build up the current blood loss situation.(Choice E)

100.

A 32 year old woman with history of infertility conceived after treatment with Clomiphene citrate is seen in
the clinic. Ultrasound at 16 weeks reveled twin gestation with a membrane dividing the uterine cavity into
2 sacs. Further, 2 placentas were visualized with one anterior and one posterior.

This woman has the highest risk of developing which of the following complications:

117
Urinary Tract Infection

Pregnancy induced hypertension

Gestational diabetes

Low maternal serum alpha- fetoprotein

Iron deficiency

A and B

A and B and C

All of the above are correct

Explanation

The correct answer is choice B

Gestational hypertension or pregnancy-induced hypertension is defined as the development of new arterial


hypertension in a pregnant woman after 20 weeks gestation. Hypertension is found in 20% of multiple
gestations as opposed to 5% in single pregnancies. Preeclampsia (related to pregnancy induced
hypertension) is seen to be a recognized complication due to the extra demands on the circulation and
possible decrease in circulating prostaglandin levels.

Complications of the Fetus and Newborn with Multiple Gestation

 Preterm birth occurs in over 50% of twin pregnancies, 90% of triplet pregnancies, and virtually all
quadruplet pregnancies.
 Compared to singleton pregnancies, a twin is seven times more likely and a triplet is over 20
times more likely to die in the first month of life.
 Prematurity is associated with an increased risk of respiratory distress syndrome (RDS), intra-
cranial hemorrhage, cerebral palsy, blindness, low birth weight, and neonatal morbidity and
mortality. RDS accounts for 50% of all neonatal deaths associated with premature birth.
 Intrauterine growth restriction, intrauterine death of one or more fetuses, miscarriage, and
congenital anomalies are all more common.
 Lifelong disability is over 25% for babies weighing less than 1,000 grams (2 lbs., 3 oz.).

Maternal Complications Associated with Multiple Gestation

 Preeclampsia, also called pregnancy-induced hypertension, occurs three to five times more
frequently
 Premature labor requiring prolonged bed rest or hospitalization is common
 Placental abnormalities associated with maternal hemorrhage are more likely to occur
 Gestational diabetes, anemia, and polyhydramnios occur more frequently
 Cesarean section is often needed for twin pregnancies and almost always required for triplets.

There is no reason for Urinary Tract Infections (Choice A) to be increased in twin gestations.

Gestational Diabetes (Choice C) is secondary to insulin resistance and is associated with multiple
gestations but not as frequently as hypertension.

Low maternal serum alpha-fetoprotein (Choice D) is a reflection of chromosomal abnormalities like


trisomies (trisomy 21 in Down Syndrome, trisomy 18 in Edwards syndrome). Abnormal levels may also be
a result of the following multiple pregnancies and inaccurate dates.

118
101.

A 51 year old woman complains of recurrent vaginal pruritus and dyspareunia at the introitus. She denies
pelvic pain, but notices occasional mild dysuria, urgency, and frequency with no hematuria, nausea, or
back pain. She has been in a monogamous sexual relationship with a female partner for 20 years. On
examination, the vulva is erythematous and moist with labial swelling. There are erythematous satellite
lesions on her inner thighs at the groin. There is no tenderness to palpation of the suprapubic, adnexal or
lower abdominal regions. She denies any recent antibiotic use or change in detergents, soaps, or lotions.

Which one of the following conditions is the most likely predisposing factor?

chronic urinary tract infection

menopause

diabetes

human immunodeficiency virus (HIV) infection

pediculosis pubis

Explanation

The correct answer is Choice C.

Satellite lesions, vaginal erythema, pain and pruritis are symptoms of vaginal candidiasis. In a 51 year old
female, this would be unusual without an underlying medical condition, due to the relative estrogen
deficiency that occurs with age. Risk factors for developing chronic vulvovaginal candidiasis include use of
broad-spectrum antibiotics, relative excess of estrogen (including use of birth control pills and pregnancy),
longterm use of corticosteroids (non topical), and systemic diseases such as diabetes and HIV/AIDS.

Choice C, diabetes, is a common and likely underlying cause for recurrent vaginal candidiasis.
Undiagnosed (or poorly controlled) diabetes is associated with high blood sugars, which is associated with
overgrowth of mucocutaneous candidiasis (thrush, vaginal candidiasis), and may be the first sign of
hyperglycemia.

Choice A, chronic urinary tract infection, while a plausible response, does not fit clinically with this
case. The dysuria and urinary urgency is likely due to the vaginal inflammation and frequency can be
explained by diabetes. The lack of suprapubic pain and hematuria decrease the likelihood of chronic
urinary tract infections. Urinary tract infections would not explain the findings of vaginal erythema or
satellite lesions. Therefore Choice A is incorrect.

Menopause (Choice B) is associated with a resolution of recurrent vaginal candidiasis, due to the increase
in vaginal pH and atrophy of vaginal mucosa associated with decrease in estrogen. Atrophic vaginitis and
bacterial vaginosis are more commonly associated with menopause. Therefore Choice B is incorrect.

HIV infection (Choice D) may be associated with recurrent vaginal candidiasis as a predisposing factor,
however this patient is very low risk for HIV infection, given her sexual history. It is possible that she has
acquired HIV infection, however diabetes would be a more plausible explanation, making Choice D
incorrect.

Pediculosis pubis (Choice E) would cause vulvar pruritis, however, pubic lice would not affect the vaginal
mucosa, as it is associated with the pubic hair. Therefore Choice E is incorrect.

102.

A 28 year old woman presents to your office for renewal of her oral contraceptives prescription. You perform a well woman
exam, which reveals no significant abnormalties, however the results of the Papanicolaou (Pap) smear return with cervical

119
intraepithelial neoplasia (CIN) 3/high-grade squamous intraepithelial lesion (HSIL).

Which of the following would be the best course of action?

refer the patient for colposcopy

perform a four quadrant punch biopsy of the cervix

repeat the pap test in 6 to 8 weeks

perform human papilloma virus (HPV) DNA probes

refer the patient for a cone biopsy

Explanation

The correct answer is Choice A.

HSIL lesions are generally asymptomatic and non visible to unaided visual inspection and are generally first
identified with screening Pap smear. HSIL lesions have significant malignant potential. It is estimated that
30 to 50% of HSIL lesions will progress to invasive disease if left untreated.

HSIL is strongly associated with oncogenic (high-risk) types of human papilloma viruses (HPV). HPV DNA
testing can be performed on a liquid based cytology sample for detection of high risk HPV, however HPV
DNA testing not indicated for Pap smears read as HSIL since over 90% are HR-HPV DNA positive. Reflex
HPV testing is of more use in Pap smears read as low grade squamous intraepithelial lesion (LSIL) or
atypical squamous cells of undetermined significance (ASCUS) to determine if there is an associated high
risk HPV strain present.

For a women with a Pap smear demonstrating HSIL, the American Society for Colposcopy and Cervical
Pathology (ASCCP) practice consensus guidelines recommend immediate referral for colposcopy (Choice
C) in order to visualize and biopsy the lesion. Endocervical curettage (ECC) is also recommended to
identify dysplastic cells within the endocervical canal that cannot be visualized directly.

4 quadrant punch biopsy (Choice B) of the cervix may be performed under colposcopic direction, however,
on its own, 4 quadrant punch biopsy can miss lesions that maybe identified by direct visualization of the
cervix via colposcopy. 4 punch biopsy also does not include ECC, which is necessary, due to the number of
lesions that arise from the transitional zone. Therefore Choice B is incorrect.

Repeating the Pap smear in 6-8 weeks (Choice C) would be of no value, as there is a false negative rate
associated with screening and diagnostic Pap smears, as well as with gathering another sample after only 6
weeks. Therefore Choice C is incorrect.

Performing HPV testing would be of little value (Choice D) because greater than 90% of HSIL can be
attributed to HPV, and can thus be assumed to be positive. Therefore Choice D is incorrect.

Referral for a cone biopsy (Choice E) would be indicated following the outcome of colposcopy. Therefore,
Choice E, while a step in the diagnostic chain, is not the NEXT step in diagnosis of HSIL and is incorrect.

120
103.

A 29 year-old primigravida mother has a triple screen result at 16 weeks gestation showing lower than
normal levels of alpha-fetoprotein and estriol and higher than normal levels of beta-hCG, resulting in a
1/135 risk of Trisomy 21 (Down syndrome). Declining amniocentesis, the mother accepts the risk and
continues the pregnancy. Ultrasonography conducted at 24 weeks and 28 gestation then reveals that the
fetus has a ventricular septal defect. Following birth, however, chromosomal studies show a normal
female karyotype of 46, XX and the infant is diagnosed with an isolated ventricular septal defect.

All of the following are true concerning this infant's ventricular septal defect, except?:

symptoms may not develop until the infant is 2-3 months age

siblings of this infant would have 3 times the normal risk of having this defect

the infant is at elevated risk of developing Eisenmenger syndrome

this defect makes the heart more like that of an amphibian

the child will be at elevated risk for diastasis recti

Explanation

The correct answer is choice E.

The prenatal screen known as a triple test is a measure of the levels of alpha-fetoprotein (AFP), estriol,
and beta human chorionic gonadotropin (beta-hCG) in the maternal blood. When Down syndrome is
present, AFP and estriol tend to be too low, while beta-hCG tends to be too high for the gestational age.

121
Based on the results a probability is calculated as to whether the fetus may have Down syndrome.

Amniocentesis is a test in which amniotic fluid is drawn through a needle under ultrasound guidance. The
fluid contains fetal cells whose chromosomes can be viewed as a karyotype and counted. Normally,
amniocentesis is recommended for women 35 years of age or older, but even for a younger mother it is
indicated if the result of her triple test suggests a higher than normal probability of a trisomy such as Down
syndrome. When chromosomal or genetic analysis is required earlier than the 15th gestational week,
another sampling procedure, chorionic villus sampling, may be conducted as early as the 10th week.

In this case, the abnormal triple test result may have led the clinician to look especially carefully for
defects, so that the ventricular septal defect (VSD) was no surprise at birth, despite the relieving news that
the child did not have Down syndrome.

During development, the heart begins a tubular structure but then is partitioned by a septum which ends
up dividing the two ventricles. A VSD occurs when the septum does not close completely. Since the fetal
blood is oxygenated by circulating through the mother, a VSD as no consequences during fetal life. After
birth, however, the defect allows for mixing between blood on the left and right sides of the circulation.
This is similar to the three-chambered heart of an amphibian (choice D), consisting of two atria and a
single ventricle. In the case of amphibians, however, evolution has provided a physiology well-suited for
this arrangement. Thus, for example, amphibian blood is oxygenated, not only in the lungs but also in the
skin, allowing them to "breath" oxygen from the water. Together with other features of amphibian
anatomy, this results in very little mixing of oxygenated and deoxygenated blood in the single ventricle.

In humans, in contrast, connection between the left and right circulation results in pathology due to
shunting. Since pressure is much higher in the left ventricle compared to the right, a left-to-right shunt
develops and gradually pressure increases on the right side. When this right side pressure elevation gets to
the point of causing irreversible pulmonary hypertension, it is known as Eisenmenger syndrome (choice C).

Generally, even with a small VSD, routine physical examination of an infant will reveal a murmur. If the
defect is moderate, increased sympathetic activity tends to lead infants to sweat, particularly during
feedings. Pulmonary congestion may lead to respiratory infections, but generally it takes 2-3 months
before symptoms are obvious (choice A). as they result from a decrease in pulmonary vascular resistance
which may be normal just at birth. For the same reason, even fairly large defects may be not be
symptomatic during the first two months of life.

Epidemiological studies have found that infants born to mother who already has given birth to an infant
with an isolated VSD have a three fold incidence of the same defect, compared the general population
(choice B). In addition to familial genetic risk factors, causes of ventricular septal defect include maternal
conditions, particularly diabetes and phenylketonuria (PKU) in the mother.

If the VSD is small, the infant in this question may require only prophylactic antibiotic therapy to prevent
endocarditis. If large, surgical repair of the defect may be in order.

Diastasis recti (choice E) is a defect in the anterior abdominal wall and may be associated with umbilical
hernia. It can be congenital as it occurs in association with Down syndrome, which thankfully this child
turned out not to have, or it can be acquired. The latter occurs most often in connection with pregnancy,
when a woman of narrow girth is stretched from within.

104.

A 15 year old female patient presents to clinic for a check up. She has been sexually active with her 17
year old boyfriend for the past 4 months. This is her first and only partner, however she does not know his
sexual history. As her family doctor you are performing annual health assessment. She has not been using
any form of contraception and cannot remember if she had a menstrual period last month, but denies any
nausea, breast tenderness or pelvic pain.

Each of the following should be included in your routine assessment except:

122
complete pelvic examination (external and internal)

screening for gonorrhea and chlamydia (endocervical or vaginal swab)

obtain vaginal sample for wet mount (microscopy)

urine pregnancy test

screening papanicolau (Pap) smear

Explanation

The correct answer is Choice E

The US Preventive Services Task Force, following consensus guidelines, recommends routine Pap screening
beginning at age 21 or 3 years after sexual debut, whichever is first. In this case, a 15 year old female
with reported sexual activity for 4 months, would not necessitate a Pap smear (Choice E). The following
evaluation measures would be indicated for this patient:

Choice A, complete pelvic exam, would be indicated to evaluate for signs of sexually transmitted infections
(genital warts, vaginitis, cervicitis, PID). Adolescent females are at higher risk for sexually transmitted
infections (STIs) for a variety of biological and behavioral reasons. Among US women, females age 15-19
have the highest rates of acquiring Gonorrhea and Chlamydia. Many of these cases are either
asymptomatic or the symptoms are not recognized by the patient. Therefore, Gonorrhea and Chlamydia
testing would (Choice B) be indicated for this patient.

Obtaining a sample for wet mount (Choice C) would be indicated in order to screen for vaginitis and
vaginosis, due to the asymptomatic nature of bacterial vaginosis. Increased white cell response on wet
mount would also raise the index of suspicion for cervicitis and potentially direct presumptive treatment.

Urine pregnancy test (Choice D) would be indicated based on the patient's inability to recall her last
menstrual period and lack of reliable birth control.

105.

A 39-year-old woman presents to the emergency department 9 days after delivering a full-term healthy
baby by caesarean section. She complains of shortness of breath that started the previous day, has
progressively worsened, and is accompanied by a cough that is minimally productive of white-pink
sputum. She denies chest pain, palpitations, fever, or chills.

Physical examination shows she is severely distressed and tachypneic. Vital signs are: heart rate, 120
beats/min; BP, 230/120 mm Hg; temperature, 37°C; respiratory rate, 34 breaths/min. She has bilateral
diffuse crackles in both lung fields. Cardiac sounds are normal, and no murmurs are heard. She has 2+
pedal edema bilaterally. Chest x-ray reveals bilateral diffuse alveolar infiltrates. An arterial blood gas
measurement on 50% Venturi mask shows: pH, 7.17; PaCO2, 77 mm Hg; PaO2, 54 mm Hg. Urinalysis
reveals no red blood cells (RBCs) per high-power field (HPF) on microscopy, 3 WBCs per HPF on
microscopy, and 3+ protein on dipstick testing.

What is the most likely diagnosis?

Fat embolism leading to adult respiratory distress syndrome (ARDS)

Postpartum cardiomyopathy leading to pulmonary edema

Preeclampsia leading to ARDS

Preeclampsia leading to pulmonary edema

123
Postpartum sepsis leading to ARDS

Explanation

The correct answer is choice D

Preeclampsia which is most often seen when a patient is still gravid, can occur up to two weeks post
delivery.

Symptoms of preeclampsia include:

 hypertension
 edema
 proteinuria

If untreated, this can lead to pulmonary edema. Xray findings typically show marked mottling from the
hilar regions and cover both lung fields. There is dilatation of the major vessels. It might also present
"ground class" appearance with cloudy lung fields due to diffuse interstitial edema.

Fat embolism leading to respiratory distress (Choice A) is associated with a normal chest xray in 30% of
the cases. One would see hemidiaphragm elevation (Hampton's hump), atelectasis and pleural effusion.
Tachycardia is seen but one would not expect hypertension and proteinuria.

Postpartum cardiomyopathy (Choice B) leading to pulmonary edema would not have proteinuria
associated with it. It is heart failure during the last month of pregnancy or within 5 months after delivery.
The cause of postpartum cardiomyopathy is unknown but affects 1 in 2500 pregnant women.

Symptoms include:

 cough
 palpitations
 fatigue
 leg edema
 dyspnea (either with exertion or lying prone)

In Preeclampsia leading to ARDS (Choice C) you would not expect the xray findings that are present in
pulmonary edema as above.

Postpartum sepsis (Choice E) would most likely present with an increase in fever and no proteinuria.

Postpartum infections occur after delivery or due to breastfeeding due to spread of colonized bacteria
following delivery. Causes of infection include:

 postsurgical wound infection


 perineal cellulitis
 mastitis
 respiratory complications from anesthesia
 retained products of conception
 urinary tract infections

124
 septic pelvic phlebitis

106.

A 16 year-old girl of Irish-Scottish ancestry is admitted to hospital with severe pain in the right lower
abdominal quadrant, accompanied by nausea and malaise. She denies use of tobacco or other drugs and
her history is negative for major medical conditions or surgical procedures. Though her last menstrual
period was 6 weeks earlier, she denies being sexually active and notes that her cycles occasionally have
lasted as long as 7 weeks since she began menstruating two years earlier. On physical examination, her
blood pressure, pulse rate, and respiratory rates are normal, at 110/72 mmHg, 68 beats/min, and 16
breaths/min, respectively. Her temperature is slightly elevated at 37.8°C (normal 37.0). You note rebound
tenderness in the abdomen which is most severe around the right iliac fossa. You also note that breast
and public hair development is appropriate for her age, in Tanner stage V, the final stage of development.

The management of this patient should include all of the following, except:

a pregnancy test

ultrasonography

C-reactive protein test

complete blood cell count

analysis of synovial fluid for familial mediterranean fever (FMF)

All of the above are true

Explanation

The correct answer is choice E.

The two leading diagnoses for this presentation are acute appendiciits and ectopic pregnancy. Though it is
a condition that often mimics appendicitis in young people who may not know that they have it, familial
Mediterranean fever (FMF) often includes arthralgia, and synovial fluid drawn from joints (choice E) tends
to be inflammatory, with cell counts that can approach 100,000/µL. However, the notation that the patient
is of Irish-Schottish origin is included to make it clear that FMF is not likely in her case, since FMF occurs
mainly in people of middle eastern origin, such as Armenians, Arabs, and Sephardic Jews.

Although this girl denies sexual activity, this is a datum which must be taken with a grain of salt. Often,
adolescents are reluctant to be truthful about their sexual activity, because they are embarrassed, or
because they fear confrontation with their parents.

The Tanner scale is used to assess development in children and adolescents. It consists of five stages
which define the development pubic hair in male and females, the genitals in males, and breasts in
females. The fact that this patient is only in Tanner V, the state of full maturity, is a clear indication that
she is well-capable of becoming pregnant, despite the fact that her menstrual cycles occasionally are up to
seven weeks in duration. Often for the first few years of mesntruation, cycle length is inconsistant, but the
fact that this patient has them means that she ovulates.

Ectopic pregnancy is condition in which implantation of a fertilized ovum takes place outside of the uterine
endometrium. Often this occurs in a fallopian tube. While the highest risk group for this is in the age range
of 35-44 years, it can happen during adolescence as well. Classically, ectopic pregnancy presents with
pain, amenorrhea, and vaginal bleeding, but only half of patients present in this way. For this reason, for
any woman in her child-bearing years with amenorrhea and abdominal pain, ectopic pregnancy must
remain in the differential diagnosis until a negative pregnancy test (choice A) takes it off the list. In the
event of an ectopic pregnancy, if it occurs in the right tube, the presentation can be very similar to that of
appendicitis, which also is a major possibility in this case.

125
Typically, patients with appendicitis present with an elevated white blood cell count; for this reason a
complete blood cell count (CBC, choice D) is warranted. A CBC contains several components only one of
which is the white cell count but normally white blood cell count is in the range of 4,500-10,000 cells/ul.
Typically, in appendicitis, the count is 11,000 cells/ul or higher. Because of the inflammatory response, C-
reactive protein (choice C) also is elevated in acute appendicitis.

Ultrasonography (choice B) is extremely useful in this case, as it can reveal an inflammed appendix as well
as a gestational sac in a fallopian tube, or elsewhere.

107.

A new patient, a 73-year-old healthy woman, presents to your office for a regular checkup. She has
osteoarthritis, which is controlled with OTC acetaminophen (Tylenol). She has had an appendectomy in
the past. She has no symptoms and feels well and reports that she has had Pap smears in the past but
cannot remember exactly when her last Pap smear was done or what the results were. You obtain her
records from her previous health care provider, which show that she has had 4 Pap smears in the past 10
years, which were all read as negative. She reports that she is sexually active with her long-term partner.
She asks if she needs a Pap smear today.

Based on this information, what is the most appropriate course of action?

Perform the Pap smear and, if normal, another one in 1 year, then stop screening.

Refer her to a gynecologist for further evaluation.

Advise her that screening for cervical cancer should not be necessary at her age in the absence of
any new risk factors.

Obtain a pelvic ultrasound.

Continue to perform yearly Pap smears.

Explanation

The correct answer is Choice C.

Although there are no firm guidelines to dictate the precise age at which screening for cervical cancer is no
longer necessary, the US Preventive Services Task Force recommends stopping screening at age 65. The
American Cancer Society suggests stopping at age 70 for women who have had adequate recent screening
with normal Pap smears and are not otherwise at high risk. According to the American College of
Obstetrics and Gynecology, it is reasonable to discontinue screening in women at 65 to 70 years of age
who have had three or more consecutive normal smears, and no abnormal results in the previous ten
years.

Most providers agree that a woman of this age who has had three or more negative Pap tests and no new
risk factors for cervical cancer (such as multiple or new sex partners) would not need a screening Pap
test. Older women who have not been adequately screened should have Pap tests annually until they have
had three negative tests.

Women who have had a total hysterectomy (including removal of the cervix) do not need to have a Pap
test, unless the surgery was done as a treatment for precancer or cancer.

Current general guidelines recommend that women have a Pap test at least once every 3 years, beginning
about 3 years after they begin to have sexual intercourse, but no later than age 21. It is agreed that
waiting 3 years is safe, because cervical cancer usually develops slowly. Cervical cancer is extremely rare
in women under age 25.

126
Table 1: North American Pap Guidelines

108.

An 18-year-old primigravida gives birth at 38 weeks gestation via uncomplicated vaginal delivery. The
baby weighs 1500 gm and the placenta weighs 330 gm. No congenital anomalies are noted. Her
pregnancy was other wise normal and she had proper prenatal care. She has no medical problems and
denies any medication or drug use during pregnancy.

Which of the following conditions is most likely to produce these findings?

Prematurity

Intrauterine growth retardation

Maternal diabetes mellitus

Cystic fibrosis

Abruptio placenta

Explanation

The correct answer is choice B

A baby at this presenting weight is small for gestational age which occurs with intrauterine growth

127
retardation; it's seen below the 10th percentile for its genstational age. Causes of IUGR can either be
maternal causes or placental or umbilical cord causes including:

 maternal cardiac disease


 chronic high altitudes
 multiple gestations
 Placental insufficiency
 preeclampsia
 substance abuse
 chronic hypertension
 poor nutrition
 smoking
 abnormal cord insertion

A delivery at 38 weeks gestation would not be considered prematurity (Choice A)

Maternal diabetes mellitus (Choice C) is associated with macrosomia or larger than dates.

Infants of maternal cystic fibrosis (Choice D) are not associated with IUGR.

Abruptio placenta (Choice E) is a catastrophic delivery (unlike the normal delivery above) whereby one
sees third trimester bleeding and uterine rigidity prior to delivery as the placenta prematurely separates
totally or partially from the uterine wall.

109.

A 20-year-old G2 P1 woman has had an uncomplicated pregnancy, but she has received no prenatal care.
She goes into labor at term. During the birth process, she experiences sudden extensive hemorrhage with
profuse vaginal bleeding. The baby is liveborn and weighs 2790 gm. Her past medical history is
unremarkable and she is on no medications.

Which of the following is the most likely diagnosis?

Umbilical cord prolapse

Placenta previa

Chorioamnionitis

Placental infarction

Gestational diabetes

Explanation

The correct answer is choice B

Placenta Previa is one of the two most common causes of third trimester bleeding and is caused by the
placenta preceding the baby in the birth canal. It clinically presents with bright red, painless vaginal
bleeding and one must consider this in all patients with bleeding. Classification or degree of the previa is
dependent on the amount of placenta obstructing the os. In a partial previa, there is only some blockage or
marginal is only a portion of the placenta involved. This would allow for delivery of the baby accompanied
by heavy vaginal bleeding. The incidence rate is approximately 4.8/1000 at term. Risk factors include the

128
following:

 age > 35
 smoking
 minority
 multiparity

previous previa
 history elective abortions
 high altitude
 history cesarean section

There are various causes of antepartum haemorrhage and occurs in 2-5% of all pregnancies. These
include the following:

 abruptio placenta- 40%


 unclassified- 35%
 placenta previa- 20%
 lower genital tract lesion- 5%
 other- <1%

Etiology of APH may be divided into the following:

 Cervical: contact bleeding (e.g. intercourse, pap, neoplasia, examination), inflammation (e.g.
infection), effacement and dilatation (e.g. labour, cervical incompetence)
 Placental – abruptio, previa, marginal sinus rupture
 Vasa previa

129
 Other - abnormal coagulation

Diagnostic Procedures should include the following:

 History and physical - No digital pelvic exam


 Ultrasound – definitive test for previa – less useful in abruptio
 Electronic Fetal Monitoring – for fetal compromise and uterine tone
 Speculum – do ultrasound first if possible – No digital pelvic exam

Laboratory tests that you should consider:

 CBC, blood type, Rh, Coombs


 coagulation status – INR, PTT, fibrinogen or TCT
 2-4 units of PRBC cross matched as appropriate
 bedside clot test
 Kleihauer-Betke or Neirhaus test – vaginal and/or maternal blood
 fetal lung maturity indices if appropriate

Umbilical cord prolapse (Choice A) can occur with rupture of membranes whereby a portion of the
umbilical cord comes out of the cervix or vulva ahead of the fetus which endangers the baby's life.

Chorioamnionitis (Choice C) is an inflammatory process of the amnion and chorion usually secondary to a
bacterial infection which can either occur antepartum or intrapartum.

Placental infarction (Choice D) is tissue degeneration of the placenta with scar replacement. Fibrin
deposits are seen which can occur as pregnancy progresses. Unless extensive, the fetus is not usually
affected. Abnormal bleeding is not associated with this condition.

In uncontrolled Gestational diabetes (Choice E), which is likely in a patient who had no prenatal care, is
associated with a large for gestational age baby.

110.

An 5 month-old female infant is brought to your clinic on account of vomiting and diarrhea, along with
irritability. While these symptoms began a day earlier, the infant experienced four similar episodes during
the last 2 months, but these were self-limited and dismissed as viral enteritis. Nevertheless, as the baby is
formula-fed, during the previous episode the parents were advised to try a different formula and this
seemed to have eliminated the symptoms for several weeks. The infant has not lost weight and has
remained near the 50th percentile for length, height, and head circumference since birth. On physical
examination, you notice a few patches of dry skin and a musty odor, which the parents note they smell in
the child's urine.

All of the following are true regarding this disease except, except:

incidence is high among Caucasions

incidence is very high in Turkey

incidence is very low among Ashkenazic Jews

incidence is very low in Japan

incidence is very high in Finland

130
Explanation

The correct answer is choice E.

Phenylketonuria (PKU) results from a deficient ability to metabolize phenylalanine to tyrosine. Classically,
this is due to deficiency in the enzyme phenylalanine hydroxylase (PAH). Although the precise mechanism
is not known, the resulting buildup of phenylalanine in the brain leads to mental retardation, though if
detected early the condition can be managed by way of a diet extremely low in phenylalanine, with
tyrosine added in its place.

The gene for PAH is located on band 12q22-q24.1 of chromosome 12. Since apparently one functional PAH
gene is adequate to provide enough PAH for normal metabolism, in most circumstances, PKU manifests
only in those who are homozygous for the deficiency, while those with one PKU gene are carriers. Thus, it
is an autosomal recessive disease.

Because of widespread screening of newborns for elevated phenylalanine levels, normally PKU is detected
during the first 48 hours of life. When not detected in the newborn, the resulting buildup of phenylalanine
in the brain leads eventually to microcephaly (small head), seizures, sever behavioral disorders, and
mental retardation. Earlier signs include albinism (excessively fair hair and skin), nausea and diarrhea and
a "musty odor" to the baby's sweat and urine. This child seems to be in the early stages; if treated with a
diet that restricts phenylalanine and supplements tyrosine, symptoms should subside and psychomotor
development should improve.

While incidence of PKU genes is such that the disease occurs in 1/10,000 births in the US, incidence differs
sharply among ethnic groups. In North American incidence is relatively high among Caucasians (choice A).
It is high in Irish and Turks, manifesting in 1/4,500 births in Ireland and 1/2600 in Turkey (choice B). Italy
and China have fairly high levels as well, as do Yemenites. Incidence is extremely low in most Jewish
populations, with an incidence of 1/200,000 among Ashenazic Jews (choice C). Incidence also is extremely
low in Japan (choice D) and in Finland (choice E).

111.

A 57-year old woman presents to her gynecologist with the complaint of severe hot flashes, insomnia,
anxiety and increased night sweats. Although she would like to start hormonal replacement therapy to
relieve her symptoms, she is concerned about the treatment increasing her risk for breast cancer.

What can you advise her about the most current trends and possible links with hormonal therapy and
breast cancer?

There is no proven evidence between decreasing hormonal therapy use and breast cancer
incidence

Breast cancer rates are still increasing despite the inverse trend of hormonal replacement

Breast cancer rates have decreased most precipitously among women between 50-60 years of
age

The rate of hormone receptor-positive breast cancer continues to rise

Hormonal therapy use and breast cancer incidence is only linked in obese women

Explanation

The correct answer is choice C

Hormonal replacement therapy increases the risk for hormone receptor-positive tumors only. However,
since the publication of the Women's Health Initiative (WHI) clinical trials, the use of postmenopausal
hormone replacement therapy has declined from 40% of eligible candidates to 20%. New data shows that

131
due to this, breast cancer has dropped most precipitously among women between 50-60 years.

In a report published online August 2010 in the Journal of Cancer Epidemiology, Biomarkers and
Prevention, women using estrogen therapy for more than 15 years had a 19% greater risk of developing
breast cancer over those who never underwent the therapy.

Unlike some common opinion, studies also show that hormone usage did not significantly alter the breast
tumor size.

Studies show that women with a BMI higher than 30 kg/m2 (choice E) had no significant rise in breast
cancer risk, whereas those women with BMIs lower than 25, had a much higher rate of breast cancer.

Figure 1:

Figure 2: Health Effects Observed with Current HRT Use

132
112.

A 35-year-old woman had a firm nodule palpable on the dome of the uterus six years ago recorded on
routine physical examination. The nodule has slowly increased in size and now appears to be about twice
the size it was when first discovered. She remains asymptomatic and is otherwise healthy with no medical
problems. She takes the occasional ibuprofen 200 mg BID for migraine headaches a few days a month but
has no other complaints.

Which of the following neoplasms is she most likely to have?

Adenocarcinoma

Leiomyosarcoma

133
Hematoma

Leiomyoma

Metastasis

Explanation

The correct answer is Choice D.

Leiomyomata (singular= leiomyoma), sometimes referred to as fibroids, are the most common tumors of
the uterus and affect up to 50% of all women at some point in their lives. Estrogen is believed to stimulate
the growth of these benign tumors of smooth muscle, and they do not appear in prepubertal girls.
Leiomyomata may increase in size during pregnancy and tend to regress or shrink after menopause.

As in this case, most leiomyomata do not produce any symptoms. However, bleeding symptoms may result
if the tumors are near the uterine lining, or interfere with the blood flow to the lining. Abnormal vaginal
bleeding, menometrorrhagia, or spotting between menses may occur. Women with excessive bleeding due
to fibroids may develop iron deficiency anemia. Uterine fibroids that are deteriorating can sometimes cause
severe, localized pain.

Fibroids can also cause symptoms relating to their size, location within the uterus, and their anatomical
relationship to adjacent pelvic organs. Large fibroids can cause:

 pressure
 pelvic pain
 pressure on the bladder with frequent or even obstructed urination
 pressure on the rectum with pain during defecation.

Uterine leiomyomata are typically diagnosed by pelvic exam and even more commonly by ultrasound.
Often, ultrasound can be helpful in differentiating a benign leiomyoma from other conditions such as
ovarian tumors.

Malignant transformation of a leiomyoma is rare. Less than 1% of smooth muscle tumors of the uterus are
leiomyosarcomas (malignant).

113.

A 27-year old G1P0, presents for her prenatal visit. She has no prior medical problems and is having an
ultrasound examination to evaluate her twin pregnancy.

Twin A weights 1250 g and twin B weighs 750 g. Hydramnios is noted around twin A, while twin B has
oligohydramnios.

Which statement concerning the ultrasound findings in this twin pregnancy is true?

134
The donor twin often develops polycythemia

The donor twin develops hydramnios more often than the recipient twin

There are gross differences between donor and recipient placentas

The donor twin is more likely to develop widespread thrmoboses

The donor twin usually suffers from a hemolytic anemia

All of the above are true

Explanation

The correct answer is C

In the twin-to-twin transfusion syndrome, also known as Feto-Fetal Transfusion Syndrome and Twin
Oligohydramnios-Polyhydramnios Sequence, the donor placenta is usually pale and somewhat atrophied
while the recipient placenta is congested and enlarged.

It can affect monochorionic multiples, i.e. multiple pregnancies where two or more fetuses share a chorion
and a single placenta.

Despite each fetus utilizing its own placental portion, the connecting blood vessels within the placenta
allows blood to pass from one twin to the other. Depending on the number, type and direction of the
anastomoses, blood can be transferred disproportionately from one twin to the other.

The donor twin is anemic due to the direct transfer of blood to the recipient twin who becomes
polycythemic. The recipient may suffer thromboses secondary to hypertransfusion and thereby
hemoconcentration.

Hydramnios can develop in either twin. It is however, more frequent in the recipient due to circulatory
overload. When hydramnios occurs in the donor, it is due to congestive heart failure caused by severe
anemia.

Treatment consists of serial amniocentesis with draining amniotic fluid from around the recipient twin in an
effort to reduce the pressure of the fluid. Another therapy consists of coagulating the blood in
interconnecting vessels with a laser beam and fetoscopy (fetoscopic laser ablation).

135
114.

A pregnancy of approximately 10 weeks gestation is confirmed in a 30 year old gravida 5, para 4 woman
with an IUD in place. You had councilled her about the potential risk of pregnancy with an IUD during your
last visit with her. After discussing this with the patient, she expresses a strong desire for the pregnancy
to be continued. She is other wise health and in on no medications and only has a history of polycystic
ovarian syndrome and is a smoker. On examination, she appears well and the only finding are that on the
vaginal examination, the strings of the IUD are protruding from the cervical os.

The most appropriate course of action is to do which of the following keeping in mind that she wished to
continue the pregnancy:

leave the IUD in place without any other treatment

leave the IUD in place and continue prophylactic antibiotics throughout pregnancy

remove IUD immediately

teminate the pregnancy because of the near certain risk of infection, abortion or both

perform laparoscopy to rule out an ectopic pregnancy

Explanation

The correct answer is choice C

When a woman is diagnosed with a pregnancy and an IUD is in place, and wishes to keep the pregnancy,
the device should be removed if the strings are visible. If the strings are not visible and the device can not
be readily removed, the patient is asked whether she wishes to continue with the gravida. If she does, she
is informed of the increased risk of spontaneous abortion, infection and ectopic pregnancy.

IUD come in 2 major forms, IUD with copper and IUD with progestogen. The IUD is 98-99% effective at
preventing pregnancies. The T-shaped IUD is made of polyethylene with fine copper wire wrapped around
the vertical stem. The string is clear or white and hangs from the lower limb of the IUD. This device
consists of 308 mg of copper covering portions of its stem and arms. Contraceptive effectiveness continues
for 10 years, after which time it must be replaced. IUDs produce no adverse systemic effects.

Ectopic pregnancies are reduced overall; however, the ratio of extrauterine to intrauterine pregnancy is
increased if conception does occur. IUDs are associated with a risk of uterine perforation at the time of
insertion. Dysmenorrhea, increased menstrual blood loss in the first few cycles, and increased risk of PID

136
(controversial) have been reported. These don't protect against STD's.

Contraindications include a history of previous PID in the past year or active PID, an abnormal or distorted
uterine cavity, undiagnosed genital bleeding, uterine or cervical malignancy, a history of ectopic
pregnancy, increased infection risk or suspected pregnancy.

In this patient, if she does not want to continue with the pregnancy, the IUD is removed and the
pregnancy is terminated.

When a pregnancy is diagnosed, the location of the pregnancy (intrauterine versus ectopic) should be
determined by pelvic examination and ultrasound. If confirmed to be in the uterus, a laparoscopy would be
contraindicated (Choice E)

115.

A 50-year-old woman has noted increasing abdominal enlargement for the past year. On physical
examination, there is a fluid wave, but no tenderness. Bowel sounds are present. She has no vaginal
bleeding. An abdominal ultrasound reveals bilateral 10 and 7 cm adnexal masses. At surgery there are
bilateral mass lesions of the ovaries. A total abdominal hysterectomy is performed. Pathologic examination
of the ovarian masses reveals that they are unilocular, filled with watery fluid, and covered with papillary
excrescences on all surfaces.

Which of the following neoplasms is this woman most likely to have?

Granulosa-theca cell tumors

Mature cystic teratomas

Fibrosarcoma with metastases

Serous cystadenocarcinomas

Clear cell carcinomas

Explanation

The correct answer is choice D

Serous cystadenocarcinomas have a common tendency to form papillary excrescences into the cyst cavities
and on the external surface as well. The simplest variety is a unilocular cyst of moderate size, usually lined
by cuboidal epithelium. The larger serous cysts are lobulated and multilocular with watery fluid contents.
The fluid color may be brownish from a mixture of blood.

When opened, the cyst often displays papillary outgrowths in the compartments from the wall. Some of the
smaller cavities may be filled with this papillary growth. The external surface of the tumor may be covered
with masses of papillae. The solid papillomas may form vast cauliflower growths filling the pelvis. Often
one also finds tiny calcified granules in the wall (psammoma bodies) which represent gradual degeneration
and calcification. There have been instances where the capsule of the tumor has ruptured with massive
intraperitoneal bleeding similar to an ectopic pregnancy.

Granulosa-theca cell tumors (Choice A) may cause pain. Its distinctive trait is that the tumor cells can
produce the estrogenic hormone and can be found as a cause of precocious puberty. It is also associated
with hypermenorrhea or varying effects upon the menstrual cycle.

Mature cystic teratomas, also known as dermoids (Choice B) are mainly of ectodermal origin. Grossly the
mass is characterized by a smooth tense capsule beneath which is the major cystic component. The main

137
content of this cyst is sebaceous matter usually mixed with hair. The contents of the cyst may be clear
fluid. However the sebaceous content tends to solidify on removal. Teeth, bone and neural plexi may also
be found in the mature teratoma.

Fibrosarcoma with metastases (Choice C) grossly appears to be hard and nodular or ulcerative. Mainly this
lesion starts out at the vulva and metastasizes from there. Treatment is surgical and the outcome is
reported as a poor survival. Microscopically the lesion has interlacing bundles of elongated villi with scant
cytoplasm and extreme mitotic activity.

Clear cell carcinomas also known as mesonephroid cancer (Choice E) is more highly recurrent and less
responsive to chemotherapy than serous adenocarcinoma. Histologically this "clear cell" type corresponds
to the clear cell carcinoma of the kidney. There is classic glomerulus-like tufting and hobnail epithelium.

116.

A 12-year-old girl is brought to her primary care physician because of a vaginal discharge. She has not yet
begun menstruating. On examination, she is noted to have tanner stage 4 pubic hair. The external
genitalia appear normal without signs of irritation. She has a copious, white, mucoid vaginal discharge
that has no odor.

Which of the following is the most likely diagnosis?

Bacterial vaginosis

Candidiasis

Foreign body

Physiologic leukorrhea

Trichomoniasis

Explanation

The correct answer is choice D

Although physiologic leukorrhea is seen at any age, it most commonly is seen to increase for 3-6 months
prior to a woman starting an ovulatory cycle. The discharge can be thick and white or clear and watery
with leukocytes, erythrocytes and mucus. It can be a natural defense mechanism that the vagina employs
to maintain its chemical balance in addition to preserving the vaginal tissue flexibility.

The term physiologic leukorrhea is used to refer to the exudate occurring due to estrogen stimulation. It
is a benign condition and needs only reassurance for treatment.

Bacterial vaginosis (Choice A), formerly called Gardnerella or Hemophilus vaginitis, produces a
nonirritating, odoriferous, thin grayish-white vaginal discharge associated with an elevagted vaginal pH
(greater than 4.5) and formation of a fishy odor. Performing a pH test or preparing a slide with the
discharge and potassium hydroxide will help with diagnosis.

Candidiasis (Choice B) may produce a thick,white,cheesy discharge. On microscopic analysis, the yeastlike
buds and hyphae are seen. Symptoms include intense vulvar irritation, burning and pruritus.

Presence of a Foreign body (Choice C), which typically you would find in a young child, produces copious,
foul smelling, greenish thick exudate. In a toddler or young girl, when finding this exudate, it is imperative

138
to rule out this foreign body and to remove if found.

Trichomoniasis (Choice E) which is contracted by sexual contact, produces a thin, watery, malodorous
discharge, greenish and foamy in appearance. Symptoms include vaginal and vulvar pruritus and pain.
Mucosal petechiae (strawberry spots) may be seen. The trichomonad which is a flagellated protooan may
be identified in a hanging drop preparation.

117.

A 15 year-old girl from Lebanon is admitted to hospital with nausea, malaise, fever, and pain which began
in the upper part of her abdomen, but has shifted to the right lower quadrant. On physical examination,
her blood pressure, pulse rate, and respiratory rates are normal, at 110/70 mmHg, 60 beats/min, and 16
breaths/min, respectively. Her temperature is elevated at 38.2°C (normal 37.0). You note rebound
tenderness in the abdomen which is most severe around the right iliac fossa. A pregnancy test is negative.
A white blood cell count is elevated at 13,000 cells/µl (normal 4,500-10,000/µL). C-reactive protein is
elevated as well, and you diagnose the patient with appendicitis. In performing an appendectomy,
however, you find that the appendix is healthy and not inflamed.

Most likely, the cause of her condition includes which of the following:

intussusception

endometriosis

familial Mediterranean fever

nephrolithiasis

acute cystitis

Explanation

The correct answer is choice C.

An appendectomy on a healthy appendix is typical of people with undiagnosed familial Mediterranean fever
(FMF), as this condition often manifests as episodes of peritonitis, often mimicking acute appendicitis. FMF
also includes a component of arthralgia (joint paint), but this may not be present in all attacks. For this
reason, ironically, some experts suggest that elective appendectomy should be performed on patients with
FMF during periods between FMF flareups, so that acute appendicitis can be eliminated from the differential
diagnosis during all subsequent FMF attacks.

FMF is an autosomal recessive condition that appears in people who are homozygous for an MEFV gene, of
which several are known. Some MEFV genes are common in Arab people, occuring with in an estimated
1/50 Arabs, resulting in an FMF incidence of 1/2600 in many Arabic-speaking countries. FMF is even more
common in Turkey with an incidence of 1/1000, partly because the incidence is extremely high among
Armenians, with a gene frequency of 1 in 7 people and a disease incidence of 1/500. Almost as high is the
incidence among Sephardic Jews. Descended from Jews expelled from Spain and Portugal who migrated to
North Africa, but also to Holland and southeast Europe, including Turkey, Sephardic Jews have an
estimated FMF incidence of 1/250-1,000; thus, the gene occurs in 1 out of every 8-16 Sephardic Jews.
While Ashkenazic Jews (which make up most Jews in the US and Canada) previously were thought to have
a fairly low incidence of FMF genes, studies have revealed a very high incidence, 1/5, of a particular FMF
gene, but interestingly it has resulted in a disease incidence of only 1/73,000 suggesting a low penetrance
for this particular gene.

Intussusception (choice B) is a condition in which part of an intestine telescopes in on itself, which can
result in obstruction, nausea, vomiting, and abdominal pain. The incidence of this condition peaks at ages
7-8 months with two out of three cases occuring before age one and is extremely rare after three years of

139
age.

In endometriosis (choice B) , endometrium-like tissue is found outside of the uterus. This can cause a
chronic inflammatory response in areas of the pelvis such as on the vagina, cervix, ovaries, fallopian tubes,
uterosacral ligaments and the rectovaginal septum. Typically, endometriosis limited to the pelvis develops
in women ages 25-30, while outside of the pelvis it develops with a still older peak incidence of 35-40
years. In women under age 20, it is extremely rare.

Nephrolithiasis (choice D) means renal stones (calculi). These tend to occur in the age group of 20-49
years. While movement of the stone can cause pain to move between upper and lower abdominal
quadrants, the typical location of tenderness is the costovertebral angle, and generally peritoneal signs are
absent.

Acute cystitis (choice E) results from a urinary tract infection (UTI) and workup for a UTI certainly should
be included when a young woman has pain, fever, and a negative pregnancy test. Nevertheless,
tenderness in such cases tends to be in the suprapubic area, which is to say over the bladder itself.

118.

A 24 year old female patient presents to the ER with pelvic pain. Her LMP was 8 weeks ago and she reports previously
regular periods and is in a sexual relationship with a male, using condoms infrequently. She is afebrile and experiencing
lower abdominal cramping and heavy vaginal bleeding with clots. Examination reveals a soft abdomen with mild lower
abdominal tenderness. On pelvic exam, the vagina is filled with blood and clots. The cervical os is open and tissue is
protruding. The uterus is slightly enlarged and estimated to be to a 6 week gravid size.

Which of the following is the most likely diagnosis?

ectopic pregnancy

threatened abortion

complete abortion

placenta previa

incomplete abortion

Explanation

The correct answer is Choice E.

Spontaneous abortion is characterized by a loss of pregnancy before 20 weeks gestation and may be
characterized in stages on a continuum: threatened, inevitable, incomplete, and complete.

An incomplete abortion (choice E) is characterized by intense cramping, vaginal bleeding, dilation of the
cervical os, and passage of tissue (products of conception) and blood clots. Transvaginal ultrasound should be
performed to determine if there are products of conception remaining in the uterus, which may eventually lead
to endometritis, if left untreated.

Threatened abortion (Choice B) would be incorrect because this is characterized by light vaginal bleeding with a
closed cervical os. Transvaginal ultrasound should be performed to confirm the presence of a viable
intrauterine pregnancy, as well as serum qualitative beta HcG (human chorionic gonadotropin).

Threatened abortion can also be confused with an ectopic pregnancy (Choice A) on presentation, due to the
lighter vaginal bleeding and closed cervical us. Ectopic pregnancy must be ruled out with transvaginal
ultrasound and qualitative serum beta HcG if this is suspected, due to the morbidity associated with rupture.

140
Complete abortion (Choice C) would be the final stage of abortion, after the products of conception have
successfully passed. At this point, the bleeding would have resolved (or have become minimal) as would the
cramping. Transvaginal ultrasound would confirm an empty uterus.

Choice D, placenta previa is a distractor answer, as this mainly occurs during the third trimester, bleeding is
bright red and usually painless. This case would be far too early in pregnancy for this condition.

Table 1: Abortion types Review

119.

A 55 year old female presents with a “period like” vaginal bleeding for 5 days that stopped
spontaneously. She has not had a menstrual period since age 49, during which time she experienced
typical symptoms of perimenopause. Last pap smear was 2 years ago and was normal. She has had no
associated pain or cramping and has not noticed any bloating, constipation, or weight changes. She is in
a monogamous relationship with her husband of 30 years. She is not taking any hormone replacement
therapy.

The next most appropriate step in managing this patient includes which of the following:

reassure the patient and manage expectantly

administer provera 5 mg daily for 5-10 days

administer cyclic estrogen plus progesterone

perform an endometrial biopsy

perform a pap smear plus endocervical curettage

Explanation

The correct answer is Choice D.

141
Any vaginal bleeding in a postmenopausal woman (previously amenorrheic for greater than 12 months) is
abnormal and should be investigated urgently, due to the possibility of endometrial cancer. About 20% of
women with post menopausal bleeding are diagnosed with associated genital cancers. Endometrial biopsy
(choice D) can detect up to 90% of endometrial cancers, can be done in office, and is currently the first
line diagnostic for postmenopausal bleeding. If a sample cannot be obtained, urgent referral for dilation
and curettage or hysteroscopic or sonohysterographic-guided biopsy should be initiated. Endometrial
hyperplasia, diagnosed via endometrial biopsy, may coexist with uterine cancer, and some forms are
considered premalignant.

Due to potential malignancy, choice A, reassure the patient, is incorrect.

Administration of provera (Choice B) is generally used to acutely manage menorrhagia in women of child
bearing age who are not considered to be at risk of uterine cancer. Therefore choice B is incorrect.

Choice C, administration of cyclic estrogen plus progesterone is used to regulate menstrual cycles in
women of childbearing years as well as to manage perimenopausal women not at risk for uterine cancer.
Choice C is incorrect.

It should be mentioned that hormonal treatment for abnormal uterine bleeding should not be initiated until
and unless genital cancers have been ruled out and other causes have been evaluated.

Choice E, perform a pap smear with endocervical curettage (ECC) would not be diagnostic for uterine
cancer, this is a screen and confirmatory for cervical cancer. While the presence of endometrial cells on
pap smear may indicate suspicion for uterine carcinoma, the pap is not an effective screen for endometrial
carcinoma, which is more likely in this age group than cervical cancer.

120.

A 25-year-old woman is G2 P1. Her first pregnancy resulted in a normal term birth. Now at 15 weeks
gestation, a prenatal checkup reveals that she has a blood pressure of 140/90 mm Hg. An ultrasound is
performed that reveals no fetal cardiac motion. Misoprostol induction is performed for termination of the
pregnancy. Examination of the malformed stillborn fetus reveals that it is small for gestational age and
has 3,4 syndactyly bilaterally, an indented nasal bridge, and a two vessel cord. The placenta is small for
gestation and has scattered 0.5 cm grape-like villi.

A chromosome analysis performed on the placental tissue will most likely demonstrate which of the
following karyotypes?

46, XX

69, XXY

45, X

47, XX, +18

23, X

Explanation

The correct answer is Choice B.

The karyotype of the fetus is a triploid male, 69 XXY. Triploidy has been associated with the development
of a partial hydatidiform mole, as described in this case. Triploidy and partial hydatidiform mole is the
result of fertilization of a haploid ovum by two sperm or duplication of one sperm, resulting in a triploid
karyotype (69 XXY, 69 XXX, 69 XYY). Maternal age has not been found to be associated with any of the

142
mechanisms by which triploidy occurs.

Clinical features associated with triploidy range from a normal phenotype to multiple major birth defects,
as described in this fetus.

Partial moles are the only type of gestational trophoblastic disease that are associated with the presence of
a fetus, and fetal cardiac activity may be detected. However, there is a high rate of intrauterine death
related to triploidy.

In a partial mole, the hydropic changes seen in the placenta are less pronounced than those of a complete
mole and may be admixed with areas of normal-appearing placenta. In this case, the placenta had
scattered 0.5 cm grape-like villi.

Triploidy occurs approximately in 2% of conceptuses and is the most common chromosome abnormality in
first trimester spontaneous abortions. Triploidy has also been observed in premature, stillborn, and full-
term liveborn infants.

121.

You are advised by the obstetrician that the mother of a baby he has delivered is a carrier of hepatitis B
surface antigen (HBsAg).

The most appropriate action in managing this infant is to do which of the following:

screen the infant for HBsAg

isolate the infant for enteric transmission

screen the mother for hepatititis B e antigen (HBeAg)

administer hepatitis B immune globulin and hepatitis B vaccine

do nothing, because transplacentally acquired antibodies will prevent infection

Explanation

The correct answer is choice D.

Hepatitis B is a serious liver infection caused by the hepatitis B virus (HBV). The virus attacks liver cells
and can lead to liver failure, cirrhosis (scarring) or cancer of the liver later in life. The hepatitis B virus is
transmitted through contact with infected blood and bodily fluids.

Pregnant women who are infected with hepatitis B or are carriers of HB virus can transmit the virus to their
newborns during pregnancy or delivery. Almost 90% of these babies will become chronically infected with
hepatitis B at birth if there is no prevention.

Therefore it is necessary to test all pregnant women at the first prenatal visit for hepatitis B surface
antigen (HBsAg). Almost 90% of women found to be HBsAg-positive on routine screening will be HBV
carriers. Pregnant Hepatitis B carriers are advised to:

 Get vaccinated against hepatitis viruses.

 Avoid hepatotoxic drugs such as acetaminophen that may further aggravate liver damage.
 Not to donate blood, body organs, or other tissue.

143
 Not share any personal items that may be infected with their blood (e.g., toothbrushes and
razors).
 Inform the infant’s pediatrician, obstetrician, and labor staff that they are a hepatitis B carrier.

 Make sure that their baby receives complete and proper hepatitis B vaccination.

The obstetrician must inform the baby's pediatrician about the mother's HBsAg-positive status. Newborns
born to hepatitis B carrier mothers should receive hepatitis B immune globulin (HBIG) and hepatitis vaccine
(choice D) within 12 hours of birth. The aim is to provide an immediate protection, with the HBIG, and long
term immunity, with the vaccine, to such babies. If these two injections are given correctly within the first
12 hours of life, a newborn has a 95% chance of being protected against a lifelong hepatitis B infection.
The infant will need additional doses of hepatitis B vaccine at one and six months of age to provide
complete protection.

122.

A 28-year-old woman goes to her physician for a routine examination. She is sexually active. Pelvic
examination reveals no abnormalities. A Pap smear is obtained. The cytopathology report indicates the
presence of severely dysplastic cells. A biopsy of the cervix is performed, and on microscopic examination
shows cervical intraepithelial neoplasia III (CIN III).

Infection with which of the following organisms is most likely to cause her disease?

Herpes simplex virus infection

Epstein-Barr virus

Human papilloma virus 16 and 18

Human papilloma virus 16 and 18 and 31

Human papilloma virus 6 and 11

Human papilloma virus 6 and 11 and 31

Explanation

The correct answer is Choice D

Cervical cancer is the second most common cancer in women globally and certain types of human
papilloma viruses have been established as the central cause of the cancer. Despite the fact that most
women will have an HPV infection at some point in her life, (with current estimates at 70%) the virus is
usually eradicated by the immune system. Persistent HPV infection by high-risk types is the factor that
increases the risk for cervical cancer.

Although infection with HPV is common, cervical cancer is not. There are more than 100 species of HPV
and only a few are associated with cervical cancer. DNA typing is available to determine which type of HPV
with which an individual has been exposed. Low-risk HPV infections are associated with benign
proliferations such as genital warts and low-grade intraepithelial lesions which are prone to resolve.

Types 16 and 18 are generally acknowledged to cause about 70% of cervical cancer cases. Together with
type 31, they are the prime risk factors for cervical cancer.

A 2010 study discovered that E6 and E7 which are HPV proteins associated with cancer, are involved in
beta-catenin nuclear accumulation and actiation of signaling in HPV-induced cancers. These proteins
inactivate two tumor suppressor proteins (p53 and pRb)

144
Guidelines recommend routine HPB screening a Pap smear for all women over 30 years of age and under
30 when an equivocal pap result is obtained (showing atypical squamous cells of undetermined
significance)

The American Cancer Society provides the following list of risk factors for cervical cancer: human
papillomavirus (HPV) infection, smoking, HIV infection, chlamydia infection, stress and stress-related
disorders, dietary factors, hormonal contraception, multiple pregnancies, exposure to the hormonal drug
diethylstilbestrol (DES) and a family history of cervical cancer.

Herpes simplex infection (Choice A) of the vulva is common. Since it may be asymptomatic initially, the
vesicular stage progesses to the symptomatic stage of superfical serpiginous ulceration when the patient
seeks medical care. Located on the cervix, vagina and introitus, herpes simplex is associated with
intranuclear inclusion bodies on smears and have typical scanty cytoplasmic cells with huge nuclei and
crowding. There is a large clear region with a moderate sized irregular intranuclear inclusion body.

Epstein-Barr virus (Choice B) may be contracted during sexual activity in susceptible young women. There
is a hypothetical relationship between this virus and mononucleosis. Infection with Eptein-Barr virus is
often asymptomatic in children. In young adults it appears as infectious mononucleosis with
fever,lymphadenopathy and pharyngitis. In the adult, EBV has been associated with chronic fatigue
syndrome and possibly arthritic symptoms. There have also been studies linking EBV with gastric
carcinoma. Diagnosis is with detection of EBV antibodies (found in serum testing).

123.

A 28-year-old woman goes to her physician for a routine examination. She is sexually active. Pelvic
examination reveals no abnormalities. A Pap smear is obtained. The cytopathology report indicates the
presence of severely dysplastic cells. A biopsy of the cervix is performed, and on microscopic examination
shows cervical intraepithelial neoplasia III (CIN III).

Infection with which of the following organisms is most likely to cause her disease?

Herpes simplex virus infection

Epstein-Barr virus

Human papilloma virus 16 and 18

Human papilloma virus 16 and 18 and 31

Human papilloma virus 6 and 11

Human papilloma virus 6 and 11 and 31

Explanation

The correct answer is Choice D

Cervical cancer is the second most common cancer in women globally and certain types of human
papilloma viruses have been established as the central cause of the cancer. Despite the fact that most
women will have an HPV infection at some point in her life, (with current estimates at 70%) the virus is
usually eradicated by the immune system. Persistent HPV infection by high-risk types is the factor that
increases the risk for cervical cancer.

Although infection with HPV is common, cervical cancer is not. There are more than 100 species of HPV
and only a few are associated with cervical cancer. DNA typing is available to determine which type of HPV
with which an individual has been exposed. Low-risk HPV infections are associated with benign
proliferations such as genital warts and low-grade intraepithelial lesions which are prone to resolve.

145
Types 16 and 18 are generally acknowledged to cause about 70% of cervical cancer cases. Together with
type 31, they are the prime risk factors for cervical cancer.

A 2010 study discovered that E6 and E7 which are HPV proteins associated with cancer, are involved in
beta-catenin nuclear accumulation and actiation of signaling in HPV-induced cancers. These proteins
inactivate two tumor suppressor proteins (p53 and pRb)

Guidelines recommend routine HPB screening a Pap smear for all women over 30 years of age and under
30 when an equivocal pap result is obtained (showing atypical squamous cells of undetermined
significance)

The American Cancer Society provides the following list of risk factors for cervical cancer: human
papillomavirus (HPV) infection, smoking, HIV infection, chlamydia infection, stress and stress-related
disorders, dietary factors, hormonal contraception, multiple pregnancies, exposure to the hormonal drug
diethylstilbestrol (DES) and a family history of cervical cancer.

Herpes simplex infection (Choice A) of the vulva is common. Since it may be asymptomatic initially, the
vesicular stage progesses to the symptomatic stage of superfical serpiginous ulceration when the patient
seeks medical care. Located on the cervix, vagina and introitus, herpes simplex is associated with
intranuclear inclusion bodies on smears and have typical scanty cytoplasmic cells with huge nuclei and
crowding. There is a large clear region with a moderate sized irregular intranuclear inclusion body.

Epstein-Barr virus (Choice B) may be contracted during sexual activity in susceptible young women. There
is a hypothetical relationship between this virus and mononucleosis. Infection with Eptein-Barr virus is
often asymptomatic in children. In young adults it appears as infectious mononucleosis with
fever,lymphadenopathy and pharyngitis. In the adult, EBV has been associated with chronic fatigue
syndrome and possibly arthritic symptoms. There have also been studies linking EBV with gastric
carcinoma. Diagnosis is with detection of EBV antibodies (found in serum testing).

124.

A 29-year-old woman presents to the emergency department after having a 5-minute generalized tonic-
clonic seizure. She is postictal and unable to provide a history. Vital signs are: temperature, 98.6°F;
pulse, 112 beats/min; BP, 160/105 mm Hg; respiratory rate, 14 breaths/min; oxygen saturation, 97%.
Primary examination reveals that her pupils are equal and are reactive to light. She moves her extremities
purposefully to stimulation. Her husband states the patient is generally healthy and had an uncomplicated
pregnancy that resulted in a vaginal delivery 2 weeks ago.

What would be the first-line treatment for her condition?

Lorazepam (Ativan)

Magnesium Sulfate

Phenytoin (Dilantin)

Labetalol HCl (Trandate)

None of the above

Explanation

The correct answer is choice B

Preeclampsia or its progression to eclampsia can occurs after 20 weeks' gestation and can present as late
as 4-6 weeks postpartum although it is typically seen in the third trimester. It is characterized by

146
hypertension, edema and proteinuria followed by seizures if progression to eclampsia occurs.

The only known treatments for eclampsia or advancing pre-eclampsia are abortion or delivery, either by
labor induction or Caesarean section. Antihypertensives may reduce maternal and fetal mortality among
pregnancy patients with hypertension as compared to placebo.

The basic principles of airway, breathing, circulation (the ABCs) should always be followed as a general
principle of seizure management.

Magnesium sulfate is the treatment of choice in that it allays convulsions without depressing the rest of the
nervous system. A loading dose of 4 g should be given by an infusion pump over 5-10 minutes, followed by
an infusion of 1 g/h maintained for 24 hours after the last seizure. An additional consequence may be the
lowering of the blood pressure and increase in urinary output. Prior to each dose however, the reflexes and
respiratory rate should be monitored. Calcium is an antidote (calcium gluconate preparation) which should
be kept at the bedside.

Lorazepam or Ativan (Choice A) is prescribed to relieve anxiety or anxiety combined with depression and
is administered orally.

Phenytoin or Dilantin (Choice C) is an anti-epileptic dur used to control seizures that may begin from the
brain or nervous system. It functions by diminishing abnormal electrical activity in the brain.

Labetalol HCL or Trandate (Choice D) is a beta blocker used to treat hypertension by improving blood flow.
The mechanism is to relax blood vessels and slow heart rate and is given orally. First-line blood pressure
medications are labetalol, given orally or IV; nifedipine, given orally or IV; or hydralazine IV.

125.
A 43-year-old woman presents for a preoperative visit with an asymptomatic vaginal discharge. She is
scheduled for a hysterectomy the following week because of the presence of large fibroids and heavy
bleeding. Physical examination shows a thin, white discharge coating the vaginal walls. The discharge has
a "fishy" odor, and clue cells are identified on the wet mount test.

Which of the following statements concerning this patient's condition is true?

Treatment before her hysterectomy will decrease the risk of infectious complications.

One of the recommended treatments is a single oral dose of metronidazole (Flagyl) 2g.

Treatment of the patient's (male) partner is indicated to prevent recurrent infection.

If treatment is successful, her risk of relapse is very low.

The condition results from overgrowth of a single bacterial species.

xplanation

The correct answer is Choice A.

The presence of clue cells on the wet mount is diagnostic for bacterial vaginosis (BV). BV was formerly
referred to as Gardnerella vaginitis, based upon one of the predominant organisms identified, but the
condition is better characterized as an imbalance in the vaginal bacterial flora that involves a combination
of multiple bacteria. Treatment of this condition will decrease her likelihood of having an infectious
complication in her forthcoming surgery. Bacterial vaginosis is associated with endometrial bacterial
colonization, postpartum fever, post-hysterectomy vaginal cuff cellulitis, and postabortal infection.

A few antibiotics may be used in the treatment of bacterial vaginosis. Metronidazole, administered either

147
orally or vaginally, is an effective cure. Although metronidazole is a recommended treatment, the single-
dose regimen is no longer recommended. Oral clindamycin or vaginal clindamycin cream is also effective in
treating bacterial vaginosis. Tinidazole has fewer side effects than metronidazole and is also an appropriate
antibiotic choice.

Since the condition can develop in women who have never been sexually active, bacterial vaginosis is not
considered to be a sexually-transmitted disease, and treatment of male sex partners is not necessary.

Approximately 30 percent of women with bacterial vaginosis who are treated successfully with
antibiotics will have a recurrence of symptoms within three months, and over half will have a recurrence
within 12 months.

126.

A 26-year-old female smoker with normal stature is 12 weeks post-partum after a spontaneous vaginal
delivery of twins at 37 week gestation, 4 lb, 2 oz boy (growth restricted). She had proper prenatal care
throughout her pregnancy. She presents to your office with respiratory distress and bilateral lower
extremity edema. After a short work-up, a diagnosis of post-partum cardiomyopathy is made.

Which one of the following risk factors is most likely responsible for this presentation?

Multiple pregnancies

Caucasian

Pre-eclampsia during pregnant

Pregnancy-Induced Hypertenstion

Tocolytic agents use during pregnancy

Explanation

The correct answer is choice A

Peripartum cardiomyopathy (PPCM) is a form of dilated cardiomyopathy that is defined as deterioration in


cardiac function presenting typically between the last month of pregnancy and up to five months
postpartum. Peripartum cardiomyopathy occurs in one out of every 3000 to 15,000 pregnancies, with a
higher incidence in AfricaAs with other forms of dilated cardiomyopathy, PPCM involves systolic dysfunction
of the heart with a decrease of the left ventricular ejection fraction (EF) with associated congestive heart
failure and an increased risk of atrial and ventricular arrhythmias, thromboembolism, and even sudden
cardiac death.

The etiology of postpartum cardiomyopathy is currently unknown. Early evidence had suggested that
nutritional deficiencies may play a role because studies noted an increased incidence of peripartum
cardiomyopathy in women who were malnourished. However, more recent studies do not support this
relationship.. Currently, more and more evidence suggests that peripartum cardiomyopathy is actually a
type of myocarditis arising from an infectious, autoimmune, or idiopathic process.

Symptoms usually include one or more of the following: orthopnea (difficulty breathing while lying flat),
dyspnea (shortness of breath on exertion), pitting edema (swelling), cough, frequent night-time urination,
excessive weight gain during the last month of pregnancy (1-2+ kg/week; two to four or more pounds per
week), palpitations (sensation of racing heart-rate, skipping beats, long pauses between beats, or
fluttering), and chest pain. Risk factors include the following:

148
 multiple pregnancies
 obesity
 having a personal history of cardiac disorders such as myocarditis
 use of certain medications
 smoking
 alcoholism
 being African American
 malnourishment

The most recent studies indicate that with newer conventional heart failure treatment consisting of
diuretics, ACE inhibitors and beta blockers, the survival rate is very high at 98% or better, and almost all
PPCM patients improve with treatment.

While the use of tocolytic agents or the development of preeclampsia (toxemia of pregnancy) and
pregnancy-induced hypertension (PIH) may contribute to the worsening of heart failure, they do not cause
PPCM; the majority of women have developed PPCM who neither received tocolytics nor had preeclampsia
nor PIH.

127.

A healthy 25-year-old pregnant woman presents at 24 weeks' gestation with joint pain. Her five-year-old
child recently had a cold and now has a bright red rash on her cheeks that spares her mouth. Mother is
other wise feeling well and has no complaints regarding her current pregnancy. She had had full prenatal
care and regular follow-up.

Which one of the following tests is most important to assess the patient's risk of fetal hydrops from
parvovirus B19?

Serum immunoglobulin M (IgM) testing

Serum immunoglobulin G (IgG) testing

Polymerase chain reaction assay

Nucleic acid hybridization assay

Immediate Ultrasound

Explanation

The correct answer is choice A

In determining if the patient has had recent exposure to the parvovirus, serum immunoglobulin M (IgM) would
show elevation of the antibody since it is the first synthesized by the body to fight a new infection. Testing IgG
simultaneously will determine if the patient has been exposed previously and has an immunity to a particular
disease. IgG can be elevated from long past exposure and would not determine when this occurred (Choice
B).

Parvovirus (also known as 5th's disease) commonly infects humans with 50% having been infected during
childhood. The rash presents as a redness appearing as a "slapped cheek" with a lacy rash on limbs and torso.
When contracting this illness as a child, the indibvidual develops lasting immunity. Pregnancy does not appear
to affect the course of the infection, but infection may affect the pregnancy. The transmission rate of maternal
parvovirus B19 infection to the fetus is 17% to 33%. Most fetuses infected with parvovirus B19 have
spontaneous resolution with no adverse outcomes.

For the susceptible pregnant woman infected with parvovirus, only a mild illness is usually experienced and the
unborn baby usually does not have symptoms either. Rarely, the fetus may experience severe anemia leading

149
to hydrops or spontaneous abortion. Birth defects and mental retardation are not associated with parvovirus.

Presentation of Parvovirus B19 Infection

 Maternal (asymtomatic, erythema infectiousum and morbiliform exanthem, arthropathy, anemia,


myocarditis)
 Fetal (fetal loss, hydrops)

Figure 1: Management of of a pregnant woman exposed to Parvovirus B19 Infection

*adopted from J Obstet Gynaecol Can 2002;24(9):727-34.

The polymerase chain reaction assay (Choice C) analyzes copies of a particular DNA sequence used mainly to
detect hereditary diseases and other viral illnesses like HSV but not routinely parvovirus.

Nucleic acid hybridization assays (Choice D) involve the use of a labeled nucleic acid probe to identify related

150
RNA or DNA molecules on a particular DNA strand and not routinely required for this virus.

128.

A 19-year-old woman is G2 P1. Her previous gestation resulted in a normal term birth at home. Her
current pregnancy results in the birth of a 2990 gm baby at 35 weeks. At birth the infant appears hydropic
and icteric, but no congenital anomalies are present. The baby's hemoglobin is 8.5 g/dL. The placenta is
also hydropic, but microscopic examination of the placenta shows no inflammation of either fetal
membranes or placental villi, and there is no meconium staining.

Which of the following mechanisms is the most likely explanation for these events?

Maternal rubella infection in the first trimester

A maternal chromosomal abnormality

These are findings of erythroblastosis fetalis

Elevated maternal serum glucose

Increased maternal phenylalanine levels

Explanation

The correct answer is choice C

Hydrops fetalis is a condition characterized by an accumulation of fluid in the fetus in at least two
compartments which can be:

 subcutaneous tissue/scalp
 pleura (pleural effusion)
 pericardium (pericardial efusion
 abdomen (ascitis)

The condition is most commonly seen in Erythroblastosis fetalis, also known as hemolytic anemia. This is
caused by transplacental transmission of maternal antibodies to fetal erythrocytes. Incompatibility between
maternal and fetal blood groups (often Rh antigents) is responsible for this state. Treatment may include
transfusion. Maternal and baby blood and Rh typing with reflex antibody screening will confirm this
diagnosis. The most common form of erythroblastosis fetalis is ABO incompatibility, which can vary in
severity. The less common form is called Rh incompatibility, which can cause very severe anemia in the
baby. Symptoms in a newborn baby may include:

 Anemia
 Edema (swelling under the surface of the skin)
 Enlarged liver or spleen
 Hydrops (fluid throughout the body's tissues, including in the spaces containing the lungs, heart,
and abdominal organs)
 Newborn jaundice

Birth defects associated with Rubella infection in the first trimester (Choice A) include:

151
 heart deformities
 hearing loss
 mental retardation
 eye deformities.

Maternal chromosomal abnormality (Choice B) may lead to spontaneous abortion or pregnancy loss in the
first trimester.

Elevated maternal serum glucose (Choice D) or gestational diabetes is associated with large for gestational
age babies but is unrelated to anemia.

Increased maternal phenlalanine levels (choice E) as observed in patients with phenylketonuria interfere
with proper neuronal development. This can lead to psychomotor compromise and mental retardation.

129.

A fetus was stillborn at 28 weeks gestation. After macroscopic and microscopic evaluation, both the fetus
and the placenta were hydropic (enlarged, fluid-filled, balloon-like) in nature. The prenatal course had
been unremarkable and the mother is otherwise healthy. The autopsy of the fetus was completed and
revealed hepatosplenomegaly, cardiomegaly, and areas of necrosis in the brain.

Which of the following conditions best explains these findings:

Ebstein-Barr Virus infection

Gestational diabetes

Erythroblastosis fetalis

Cytomegalovirus infection

Maternal tobacco use

Explanation

The correct answer is choice D

Approximately 10% of newborns with cytomegalovirus can display the following symptoms:

 splenomegaly
 jaundice
 rash
 IUGR
 seizures
 liver dysfunction
 cerebral necrosis

About 20% of babies with CMV symptoms die and 80-90% of survivors develop mental retardation,
cerebral palsy, vision and hearing loss.

There are no known associations between active EBV (Choice A) infection and problems during pregnancy,
such as miscarriages or birth defects. A recent study also suggests that EBV infection during pregnancy
does not represent a major teratogenic risk to the fetus either.

152
Gestational diabetes (Choice B) is associated with larger than normal infants. The fetus must also be
screened with nonstress tests to ensure the regularity of the fetal cardiac status’

Erythroblastosis fetalis (Choice C) also known as hemolytic disease of the newborn occurs most typically
after the first pregnancy with succeeding pregnancies and is due to Rh or Blood type incompatibility. The
alloimmune condition develops when IgG molecules produced by the mother pass through the placenta and
attack red blood cells in the fetal circulation. The baby may have hydrops and anemia. Fetal death can
occur from heart failure.

Maternal tobacco use (Choice E) may cause IUGR in the fetus due to a decrease or compromise in the
vasculature and circulation.

Additional perinatal complications associated with usage of maternal tobacco include:

 placenta previa
 abruptio placentae
 premature rupture of the membranes
 preterm birth
 intrauterine growth restriction
 sudden infant death syndrome

Studies suggest that smoking is responsible for 15% of preterm births and a 150% elevation in overall
perinatal mortality.

130.

A 28-year-old woman presents in your office to discuss the episodic lower abdominal and pelvic pain she
has been experiencing for the past 2 years. A physical examination, including pelvic exam, reveals no
abnormalities. A Pap smear shows only a few trichomonads and no dysplastic cells. A laparoscopy is
performed, and the gynecologist notes the presence of several blue to red 0.2 to 0.4 cm slighted raised
lesions scattered on the pelvic peritoneum in the cul-de-sac and broad ligaments.

Which of the following is the most likely diagnosis?

Endometriosis

Metastatic adenocarcinoma

Neisseria gonorrheae infection

Candidiasis

Leiomyomata

Explanation

The correct answer is choice A.

These 'powder burns' are typical for the small hemorrhagic foci of endometriosis on pelvic peritoneum that
can produce so much discomfort for their size.

Endometriosis affects an estimated 1 in 10 women during their reproductive years. This is approximately
176 million women worlwide, who have endometriosis - regardless of their ethnic and social background.
Endometriosis is a condition where tissue similar to the lining of the uterus (the endometrial stroma and

153
glands, which should only be located inside the uterus) is found elsewhere in the body. Endometriosis
lesions can be found anywhere in the pelvic cavity: on the ovaries, the fallopian tubes, and on the pelvic
sidewall. Other common sites include the uterosacral ligaments, the cul-de-sac, the Pouch of Douglas, and
in the rectal-vaginal septum. In addition, it can be found in caecarian-section scars, laparoscopy or
laparotomy scars, and on the bladder, bowel, intestines, colon, appendix, and rectum. But these locations
are not so common. In even more rare cases, endometriosis has been found inside the vagina, inside the
bladder, on the skin, even in the lung, spine, and brain. Endometriosis can also cause scar tissue and
adhesions to develop that can distort a woman’s internal anatomy. In advanced stages, internal organs
may fuse together, causing a condition known as a "frozen pelvis." The cause of endometriosis remains
unknown.

The most common symptom of endometriosis is pelvic pain. The pain often correlates to the menstrual
cycle, but a woman with endometriosis may also experience pain that doesn’t correlate to her cycle.
Additionally, some women experience pain during ovulation, in the bowl (particularly during menstruation),
dyspareunia, and in the lower back region. The way endometriosis causes pain is the subject of much
research. Because many women with endometriosis feel pain during or around their periods and may spill
further menstrual flow into the pelvis with each menstruation, some researchers are trying to reduce
menstrual events in patients with endometriosis. Endometriosis lesions react to hormonal stimulation and
may "bleed" at the time of menstruation. The blood accumulates locally, causes swelling, and triggers
inflammatory responses with the activation of cytokines. It is thought that this process may cause pain.
Pain can also occur from adhesions (internal scar tissue) binding internal organs to each other, causing
organ dislocation. Fallopian tubes, ovaries, the uterus, the bowels, and the bladder can be bound together
in ways that are painful on a daily basis, not just during menstrual periods. For many women, the pain of
endometriosis is so severe and debilitating that it impacts their lives in significant ways. In addition to
pain, endometriosis causes diarrhea or constipation during menstruation, abdominal bloating during
menstruation, heavy or irregular bleeding, and fatigue. It is estimated that 30-40% of women with
endometriosis are subfertile.

There is no simple test that can be used to diagnose endometriosis, which may be why there is a
diagnostic delay of up to 12 years in some healthcare settings. At present the only reliable way to
definitively diagnose endometriosis is by performing a laparoscopy and to take a biopsy of the tissue. This
is what is known as "the gold standard". However, this is an expensive, invasive proceduce. Further,
endometrial adhesions frequently adhere to scar tissue, so the procedure itself may worsen the symptoms
for the patient. It the best of cases, an experienced gynecologist should be able to recognise symptoms
suggestive of endometriosis through talking with the woman and obtain a history of her symptoms.

A treatment which fully cures endometriosis has yet to be developed, and there is no overwhelming
medical evidence to support one specific type of treatment for endometriosis over another. Choosing a
treatment therefore comes down to the individual woman's needs, depending on her symptoms, her age,
and her fertility wishes. She should discuss these with her physician so that they, together, can determine,
which long term, holistic, treatment plan is best for her individual needs. For many women, this can be a
combination of more than one treatment over longer periods of time. Recently, several new, non-invasive
techniques have been developed to help reduce the pain associated with the adhesions of endometriosis,
such as the Wurn Technique, which show some promise in improving the quality of life for women with
endometriosis.

Metastatic adenocarcinomas (choice B) should produce mass lesions, and the patient would become
severly debilitated after several years with an extensive carcinoma.

Gonorrheal infections (choice C) can produce salpingitis that can be severe, but it usually resolves in
several weeks, though pelvic inflammatory disease can continue to produce pain. There is scarring, but no
focal red or blue lesions.

Yeast infections like Candidiasis (choice D) involve the vagina, producing discharge and local irritation.

Leiomyomata (choice E) can be large enough to cause pelvic discomfort, but they are firm masses.

131.

154
A 24 year old primigravid woman, who is intent on breast feeding, decides upon a home delivery.
Immediately after the birth of a 4.1 kg, (9 lbs) infant, the patient bleeds massively from extensive vaginal
and cervical lacerations. She is brought to the nearest hospital in shock. Over 2 hours, 9 units of blood are
transfused, and the patient’s blood pressure returns to a reasonable level. A hamoglobin value the next
day is 7.5 g/dL, and 3 units of packed red blood cells are given.

The most likely late sequela to consider in this patient includes which of the following:

hemochromatosis

stein-levanthal syndrome

Sheehan syndrome

Simmonds syndrome

Cushing syndrome

Hypothyroidism

Explanation

The correct answer is choice C

Sheehan syndrome or necrosis of the pituitary gland, also known as postpartum hypopituitarism, is a rare
compication of postpartum hemorrhage. Women then develop hypopituitarism with faiure to lactate or
difficulties with lactaion. Many women also develop amenorrhea or oligomenorrhea after delivery. This
may not be diagnosed at the postpartum period. In some cases, a woman with Sheehan syndrome might
be relatively asymptomatic, and the diagnosis is not made until years later, with features of
hypopituitarism. Uncommonly, Sheehan's syndrome may also appear acutely after delivery, mainly by
hyponatremia.

The etiology of this condition is believed to a result of the pituitary gland physiologically being enlarged in
pregnancy and becomes sensitive to the decreased blood flow caused by massive hemorrhage and
hypovolemic shock. Women with Sheehan syndrome have varying degrees of hypopituitarism, ranging
from panhypopituitarism to only selective pituitary deficiencies. The anterior pituitary is more susceptible
to damage than the posterior pituitary. Treatment of young women with hypopituitarism usually includes
replacement of hydrocortisone first and then replacement of thyroid hormone and estrogen with or without
progesterone depending on whether she has a uterus. Both thyroxine replacement and gonadotropin
replacement are common, and doses are titrated to each individual. Replacement of growth hormone is
necessary in children with hypopituitarism but is controversial in adults.

Hemochromatosis (Choice A) is a condition of iron level elevation which is a genetic disorder and may
denote hepatic disorders. There are two types.

1) Primary or Hereditary Hemochromatosis

 most common type


 inherited
 genes cause stomach and intestines to absorb excess iron

2) Secondary Hemochromatosis

 dietary iron overload


 anemias (eg. thalassemia)

155
 chronic liver disease
 juvenile hemochromatosis

Untreated, hemochromatosis leads to severe organ damage and is associated with cirrhosis of the liver,
diabetes, heart and joint disease.

Stein-leventhal syndrome (Choice B) which is also known as polycystic ovarian disease, is characterized by
cystic ovaries, secondary amenorrhea, and infertility. Associated symptoms are obesity,acne and
hirsutism. Females with polycystic ovarian syndrome do not ovulate regularly and are at a higher risk for:

 high blood pressure


 diabetes
 heart disease
 endometrial cancer

Simmonds Syndrome (Choice D) also known as panhypopituitarism, demonstrates total absence of all
pituitary secretions. It is found usually in the postpubertal period. It is also seen associated with
acromegaly.

Cushing syndrome (Choice E) is characterized by:

 truncal obesity
 hypertension
 weakness
 amenorrhea
 hirsuitism
 edema

 osteoporosis

132.

A sexually active 18-year-old woman presents with fever for the past 24 hours and lower abdominal pain
and anorexia for the past 5 days. On physical examination, there is generalized tenderness of the
abdomen especially at the adnexa and the cervix is erythematous with motion tenderness. There is no
rash nor any lesions on the external genitalia. A smear of the odorless cervical discharge contains
sloughed epithelial cells and scant neutrophils.

Which of the following would likely be found in the exudate?

Giemsa-staining intraepithelial inclusion bodies

Non-enveloped icosahedral double-stranded DNA virus

Intranuclear owl''s eye inclusion bodies

Pear-shaped flagellated protozoa

Pleomorphic, gram-negative rods

Explanation

The correct answer is choice A.

156
This is a case of acute pelvic inflammatory disease (PID), an infectious and inflammatory disorder of the
upper female reproductive tract, including the uterus, fallopian tubes, and adjacent pelvic structures. The
organisms most commonly isolated in many, if not most, cases of acute PID are Neisseria gonorrhoeae and
Chlamydia trachomatis.

Chlamydia trachomatis is trachomatis is an obligate, intracellular bacterium with 15 immunotypes, as


follows: A-C cause trachoma (chronic conjunctivitis endemic in Africa and Asia); D-K, genital tract
infections; and L1-L3, lymphogranuloma venereum (associated with genital ulcer disease in tropical
countries). It can be visualized inside epithelial cells with Giemsa or by immunofluorescent staining with
monoclonal antibodies appearing as intraepithelial inclusion bodies. Neisseria gonorrhoeae appears as
intraneutrophilic gram-negative diplococci.

Non-enveloped icosahedral double-stranded DNA virus (choice B) is consistent with human papilloma virus
which produces anogenital warts (not seen in this case).

Intranuclear owl's eye inclusion bodies (choice C) is consistent with cytomegalovirus. It is a common cause
of STDs but it is a rare cause of PID. Adult cases are usually presented with mononucleosis, a disease
characterized by fever, malaise, pharyngitis and splenomegaly.

Pear-shaped flagellated protozoa (choice D) is consistent with Trichomonas vaginalis infection which is
characterized by frothy yellowish-green malodorous vaginal discharge, vulvovaginal itching and soreness,
dyspareunia and dysuria.

Pleomorphic, gram-negative rods (choice E) is consistent with Hemophilus ducreyi that produces chncroid
which is characterized by painful genital ulceration and inguinal adenitis.

Complication of PID include tubo-ovarian abscess formation, chronic pelvic pain, ectopic pregnancy, tubal
factor infertility, and implantation failure

133.

A 32-year-old woman presents with long standing pelvic pain and dyspareunia. She is otherwise healthy
with no medical problems. Her mother and sister have a know history of endometriosis. A tender nodule is
palpated in the vaginal fornix.

Which one of the following is the most effective first-line therapy for endometriosis?

A nonsteroidal anti-inflammatory drug

An oral contraceptive

A gonadotropin-releasing hormone analogue

Levonorgestrel-releasing intrauterine system (Mirena)

Danazol (Danocrine)

Explanation

The correct answer is choice B

Endometriosis is a debilitating condition in women whereby endometrial cells appear and grow in areas
outside the uterine cavity. These cells respond to hormonal fluctuations and cause increased pain or other
symptoms during menses. Symptoms of endometriosis include:

 dysmenorrhea

157
 chronic pelvic pain
 dyspareunia
 dysuria
 back pain
 nausea
 depression

There is a familial tendency to have this condition. There is a ten-fold increase in women with an affected
first-degree relative. While there is no cure for endometriosis,[citation needed] in many patients
menopause (natural or surgical) will abate the process. Because endometriosis is estrogen dependent, oral
contraceptives to lower the body's estrogen level is very effective. Progesterone counteracts estrogen and
inhibits the growth of the endometrium. Such therapy can reduce or eliminate menstruation in a controlled
and reversible fashion.

A nonsteroidal anti-inflammatory drug (choice A) may lessen the pain of endometriosis, but will not treat
the implants which may still be stimulated to grow during the rising phase of estrogen production.

Choices C and D have been used effectively to treat endometriosis but there are markedly severe side
effects that accompany the medication. These include:

 hirsutism
 voice deepening
 acne
 increased risk of ovarian cancer

 increased hepatic damage

134.

A 28-year-old woman who is 14 weeks pregnant comes to your office for routine follow-up. She has no
complaints at this time. She is taking appropriate folic acid supplementation. A screening test for
gestational diabetes is negative. Urinalysis shows a WBC count of 3 per high-power field and no RBCs,
protein, or glucose. Urine culture grows 10^5colony-forming units/mL of E coli, which is sensitive to all
antibiotics. The patient is asymptomatic.

What is the most appropriate next step?

No treatment; reassurance

A second urinalysis and culture in the third trimester

Ciprofloxacin (Cipro) for 3 days

Double-strength trimethoprim/sulfamethoxazole (TMP/SMX; Bactrim DS, Septra DS) for 3 days,


followed by prophylactic TMP/SMX until the time of delivery

Nitrofurantoin (Macrobid, Macrodantin) for 7 days, followed by frequent screening for


recurrence

Explanation

The correct answer is choice E

The patient should be treated for her asymptomatic bacteriuria. If it is ignored (Choice A), she risks
progression to a full blown urinary tract infection (cystitis) or upper tract UTI (pyelonephritis). UTIs are

158
associated with risks to both the fetus and the mother, including pyelonephritis, preterm birth, low birth
weight, and increased perinatal mortality, preeclampsia and amnionitis. Asymptomatic bacteriuria
increases the risk for an upper tract UTI, also known as pyelonephritis. Treatment of asymptomatic
bacteriuria reduces the risk of a symptomatic infection. Causes of bacteriuria include the following
organisms:

 Escherichia coli (most common, in as many as 70% of cases)


 Group B Streptococcus (10%)
 Klebsiella or Enterobacter species (3%)
 Proteus species (2%)

Empiric antimicrobial therapy must be comprehensive and should cover all likely pathogens in the context
of the clinical setting. Empiric coverage for E coli and Klebsiella, Proteus, and Enterobacter species should
be provided. Penicillins and cephalosporins are safe for use during pregnancy. Ceftriaxone should be
withheld close to parturition due to the possibility of neonatal kernicterus secondary to bilirubin
displacement.

Nitrofurantoin (Choice E) is a mild broad spectrum antibiotic that treats e.coli and most bacteria within the
bladder and does not harm the developing fetus.

If the patient is symptom free, you can do a dipstick in the third trimester but there is no indication for a
second urinalysis (Choice B)

Fluoroquinolones (Choice C) and tetracyclines are known teratogens and are contraindicated in pregnancy.

Trimethoprim is a folic acid antagonist and should be avoided, especially during the first trimester. Use of
TMP/SMX close to the time of delivery (Choice D) is associated with fetal jaundice.

135.

A 24-year-old woman in active labor during her first pregnancy undergoes amniotomy and immediately
begins to hemorrhage from the vagina. Her pregnancy prior to this was uneventful and she is otherwise
healthy. Her prior pregnancies were also uneventful via normal vaginal delivery.

Which one of the following diagnoses is most likely?

Placental abruption

Placenta previa

Vasa previa

Rupture of the uterus

Excessive bloody show

Explanation

The correct answer is choice C

Vasa previa is a grave obstetrical complication where blood vessels involved in fetal circulation (ie.umbilical
cord or placenta) grow along the membranes in the lower uterine segment at the cervical opening. It can
be a complication of placenta previa, velamentous umbilical cord or multi-lobed placenta. When not
detected, the blood vessels can rupture during labor or inadvertently be punctured during
amniotomy. Treatment immediately with an emergency cesarean delivery is usually indicated. Fetal demise

159
is an unfortunate complication due to the massive blood loss that occurs.

Abruptio placenta (Choice A) is an obstetrical complication occurring in the third trimester whereupon the
placenta prematurely separates from the wall of the uterus. Symptoms include a rigid uterus, heavy
bleeding or hemorrhage (depending on the extent of separation) abdominal and pelvic pain.

Placenta Previa (Choice B) is an obstetrical complication seen in the late 2nd or third trimester with
painless vaginal bleeding. The condition involves placental obstruction of the cervical os (either partially or
totally) and precedes the fetus in the birth canal.

Uterine rupture (Choice D) is a catastrophic obstetrical event during childbirth where the myometrial wall
integrity is breached with a high incidence of fetal and maternal morbidity. The causes are previous
surgery as in Cesarean section or myomectomy where the cavity was entered.

Bloody show (Choice E) is the passage of a small amount of blood-tinged mucus or a small amount of
blood per vagina in early labor as the cervix effaces and starts to dilate, freeing blood and mucus from the
cervical glands. While slightly heavier shows are not dangerous to mother or fetus, very heavy show
should be further investigated to rule out abruption or placenta previa.

136.

A 58 year-old patient presents for her annual gynecological examination and is found to have a fixed,
approximately 10 cm, adnexal mass with associated diffuse abdominal tenderness and irregular, light
vaginal bleeding. Her last spontaneous menstrual period was 7 years ago. Stool occult blood is negative,
and the patient has no gastrointestinal complaints.

Which of the following statements about adnexal masses is most accurate?

Adnexal masses in young girls (prior to puberty) have a low risk of malignancy.

Elevation of the patient's CA 125 level is diagnostic for advanced ovarian carcinoma.

A dermoid cyst (mature cystic teratoma) is a common malignant tumor of premenopausal


women.

At least 30 percent of ovarian masses in women over age 50 are malignant.

Metastatic disease is an unlikely cause of adnexal masses in postmenopausal women.

Explanation

The correct answer is Choice D.

Most adnexal masses arise from the ovary. The most important diagnostic considerations in the evaluation
of an adnexal mass are to confirm that the adnexal mass is ovarian and determine whether it is benign or
malignant. Imaging studies are the most valuable preoperative diagnostic tools to characterize adnexal
masses.

The differential diagnosis of adnexal masses includes the following:

 Physiologic/functional cysts of the ovary


 pregnancy-related conditions (ectopic pregnancy, corpus luteum cyst)
 abscesses
 benign ovarian tumors
 polycystic ovary syndrome (PCOS)

160
 leiomyomata
 tumors of the broad ligament of fallopian tube
 ovarian carcinoma
 metastatic carcinoma

The serum CA 125 concentration (normal <35 U/mL) is elevated in over 80 percent of women with
advanced ovarian cancer. Numerous benign conditions such as fibroids and endometriomas, especially in
premenopausal women, can also cause an elevation of CA 125 levels.

In girls younger than 15 years of age, a high percent of ovarian tumors are malignant and adnexal masses
are not common. After puberty, the risk of malignancy of an adnexal mass inceases with age. At least 30
percent of ovarian masses in women over age 50 are malignant.

The following factors increase the suspicion for malignancy in an adnexal mass:

 masses that have a solid component


 masses with a complex structure (admixed cystic and solid areas)
 masses associated with pelvic pain
 large cysts greater than 10 cm in diameter
 In menstruating women, masses that persist beyond the length of a normal menstrual cycle
without typical characteristics of a benign process such as a hemorrhagic cyst

The mature cystic teratoma (dermoid cyst) (choice C) is a benign germ cell tumor and is the most common
ovarian tumor in the second and third decades of life.

137.

A 20-year old woman presents for a physical examination with a chief complaint of irregular menses. Her
last menstrual period was approximately 3 months ago and she denies sexual activity. She is noted to
have central obesity with moderate cystic acne and some upper lip hair. Her blood pressure is 134/84
mm Hg and her random serum glucose is 132.

Which of the biochemical studies would be most appropriate in the initial work-up of this patient?

serum free testosterone

total testosterone and serum TSH

urine pregnancy test, serum LH, serum FSH

urine pregnancy test, serum prolactin, serum TSH and serum androgen levels

serum prolactin, serum TSH and serum androgen levels

Explanation

The correct answer is choice D

The description of the disorder that this patient has is polycystic ovary syndrome. Although an irregular
menstrual cycle and hirsutism is strongly suggestive of PCO, clinical or biochemical signs of
hyperandrogenism and polycystic ovaries on ultrasound would be needed to confirm the diagnosis.

Despite the patient's protest as to sexual activity, it is important to rule pregnancy out with her
amenorrhea so a UCG would be among the first steps taken.

161
TSH and prolactin levels should be evaluated as both of these are associated with amenorrhea or irregular
cycles. Prolactin levels can be elevated in stress and in drug usage eg. marijuana in addition to medicinal
herbs and heavy metals. Hyperprolactinemia is also found in pituitary adenomas. Deficient thyroid
hormone levels can cause primary or secondary amenorrhea.

PCOS is characterized by ovulatory dysfunction. Periods do not necessarily have to be absent. Many
women with PCOS continue to ovulate, but do so either irregularly or with compromised progesterone
production. Many women with PCOS are not obese, and many women with PCOS do not have excess hair
growth, but to some extent virtually all women with PCOS have some degree of insulin resistance. Insulin
resistance implies that the peripheral tissues - skin, muscle, fat etc, do not respond to insulin normally.
The pancreas responds to this by increasing insulin production which results in increased insulin levels.

One of the primary goals of laboratory evaluation is to be sure there is not some other problem affecting
the ovaries and their function. For example, elevated Prolactin levels can cause irregular periods. (Prolactin
is a pituitary hormone that controls breast milk production. If mildly elevated, it can adversely affect
ovulation.) Abnormal thyroid function can also alter ovarian function, be it hypo- (low) or hyper- (high)
thyroidism. LH and FSH are the pituitary hormones that control ovarian function. Normally, FSH is higher
than LH. In women with PCOS, this ratio can be reversed, with LH being higher than FSH.

Specific bloodwork:

 Testosterone
 Androstenedione
 DHEAS
 17-OH Progesterone
 Prolactin
 Insulin Resistance (2 hour GTT, fasting sugars)
 TSH
 LH
 FSH

Whether to treat PCOS by regulating the menstrual cycle should be an individual decision on the part of
each woman in consultation with her physician. Many women with this condition are unaware that they
have it.

The treatment of PCOS also depends on the goal of treatment. The goal of treatment may be to produce a
pregnancy or to control any or all of the symptoms of PCOS without producing a pregnancy.

Women with symptoms such as infertility, severe menstrual problems, progressive hirsutism, male-pattern
baldness or severe acne will be helped by treatment. Treating to prevent endometrial hyperplasia or
increased heart disease risk is less immediate and should be an individual decision. Over 50% of patients
with PCOS are overweight. Sometimes even modest weight loss can have a substantial impact on some
symptoms. Proper diet and exercise are also beneficial. The first line medical treatment of PCOS is an oral
contraceptive.Other medications to consider include Spironolactone, cyproterone acetate and finasteride.
Clomiphene citrate is used to induce ovulation when infertility is a complaint. Metformin can be beneficial
to treat insulin resistance.

138.

A clinical study is performed to determine when fetal malformations are most likely to occur during
gestation when mothers who have not received prior immunization become infected by rubella virus. The
study examines abortuses for congenital anomalies.

When during gestation are malformations most likely to arise with rubella infection?

162
4 weeks

5 weeks

6 weeks

7 weeks

8 weeks

Explanation

The correct answer is choice B

Malformations are most likely to occur at 5 weeks or early in the first trimester. Other than brain and
neurological development which can occur over the first year, most organs have complete development by
the 12th gestational week and then merely need maturing of their systems.

Congenital rubella syndrome (CRS) can occur in a developing fetus of a pregnant woman who has
contracted rubella during her first trimester. If infection occurs 0–28 days before conception, there is a
43% chance the infant will be affected. If the infection occurs 0–12 weeks after conception, there is a 51%
chance the infant will be affected. If the infection occurs 13–26 weeks after conception there is a 23%
chance the infant will be affected by the disease. Infants are not generally affected if rubella is contracted
during the third trimester, or 26–40 weeks after conception. Problems rarely occur when rubella is
contracted by the mother after 20 weeks of gestation.

The classic triad for congenital rubella syndrome is:

 Sensorineural deafness (58% of patients)


 Eye abnormalities—especially cataract and microphthalmia (43% of patients)
 Congenital heart disease—especially patent ductus arteriosus (50% of patients)

Other manifestations of CRS may include:

 Spleen, liver or bone marrow problems (some of which may disappear shortly after birth)
 Mental retardation
 Small head size (microcephaly)
 Eye defects
 Low birth weight
 Thrombocytopenic purpura (presents as a characteristic blueberry muffin rash)
 Hepatomegaly
 Micrognathia

Children who have been exposed to rubella in the womb should also be watched closely as they age for any
indication of the following:

 Developmental delay
 Schizophrenia
 Growth retardation
 learning disabilities
 Diabetes

 Glaucoma

139.

163
A 72-year-old woman has been feeling tired for the past year. She has had episodes of vaginal bleeding
during this time. On physical examination there are no abnormal findings. Laboratory studies show Hgb
9.1 g/dL, Hct 26.5%, MCV 72 fL, platelet count 158,000/microliter, and WBC count 7150/microliter. An
endometrial biopsy is performed and on microscopic examination shows atypical adenomatous
hyperplasia.

Which of the following is the most likely risk factor for development of her disease?

Human papillomavirus infection

Long term use of an intrauterine contraceptive device

Chronic endometritis

Pelvic inflammatory disease

Unopposed estrogenic stimulation


Explanation

The correct answer is Choice E.

Endometrial hyperplasia, or the proliferation of endometrial glands relative to stroma, is a consequence of


unopposed (without the balancing effects of progesterone) estrogen stimulation. The hyperplastic
endometrial glands may begin to show features of atypia and/or progress to carcinoma.

Risk factors for endometrial hyperplasia are the same as those for endometrial cancer and are factors that
lead to estrogenic stimulation of the endometrium. In particular, the risk for these conditions is increased
by 10-fold in women who use unopposed estrogen therapy (and have not undergone hysterectomy). The
risk for endometrial hyperplasia and cancer also increases with increasing degree of overweight or obesity.

Other risk factors that lead to chronic estrogen stimulation include chronic anovulation and estrogen-
secreting tumors, such as granulaosa cell tumors of the ovary, which may secrete estradiol.

Hormone therapy consisting of estrogen without progesterone also confers a risk of endometrial
hyperplasia. Estrogen therapy (without progesterone) used for one to two years significantly increases the
risk of endometrial hyperplasia in a dose- and duration-dependent manner.

As in this case, vaginal bleeding in a postmenopausal woman is a characetristic symptom of endometrial


hyperplasia. Irregular, prolonged, heavy, or frequent bleeding are other symptoms. Any vaginal bleeding in
a postmenopausal woman is abnormal and warrants further diagnostic evaluation.

140.

A 28-year-old woman is brought to the emergency room with a 1 day history of increasing obtundation.
On physical examination her temperature is 37.3 C, pulse 85/minute, respirations 16/minute, and blood
pressure 90/45 mm Hg. A culdocentesis yields no blood. Abdominal ultrasound shows an intrauterine
gestational sac and 12 week fetus. Laboratory studies show a positive pregnancy test. She has a WBC
count of 11,400/microliter, Hgb 10.6 g/dL, Hct 31.1%, MCV 95 fL, and platelet count 26,400/microliter.
Schistocytes are noted on her peripheral blood smear. Her protime is 44 sec, partial thromboplastin time
61 sec, and D-Dimer 16 microgm/mL. A urinalysis dipstick examination shows no blood, protein, or
glucose.

Which of the following complications of pregnancy is most likely to explain her condition?

164
Eclampsia

Placental infarction

Amniotic fluid embolus

Ruptured tubal ectopic

Retained dead fetus

Explanation

The correct answer is choice E.

The complication of retained dead fetus, usually longer than 4 weeks, results in release of thromboplastin
into the circulation, promoting disseminated intravascular coagulation (DIC) with consumption of clotting
factors and thrombocytopenia. In DIC, the processes of coagulation and fibrinolysis are dysregulated, and
the result is widespread clotting with resultant bleeding.

In mild cases, without hemorrhaging, diagnosis of DIC is suggested by increased synthesis of coagulation
factors and platelets. PT, APTT, and platelet counts are normal, and fibrin degradation products are raised.
In severe cases with hemorrhaging, DIC is diagnosed by very prolonged PT and APTT, and a markedly
reduced fibrinogen level. High levels of fibrin degradation products, including D-dimer, are found owing to
the intense fibrinolytic activity stimulated by the presence of fibrin in the circulation. There is severe
thrombocytopenia. The blood film may show schistocytes.

The only effective treatment is the reversal of the underlying cause. For this patient, immediate
termination of the pregnancy. Anticoagulants are given exceedingly rarely when thrombus formation is
likely to lead to imminent death (such as in coronary artery thrombosis or cerebrovascular thrombosis).
Platelets may be transfused if counts are less than 5,000-10,000/mm3 and massive hemorrhage is
occurring, and fresh frozen plasma may be administered in an attempt to replenish coagulation factors and
anti-thrombotic factors, although these are only temporizing measures and may result in the increased
development of thrombosis.

Prognosis varies depending on the underlying disorder, and the extent of the intravascular thrombosis. The
prognosis for those with DIC, regardless of cause, is often grim: Between 10% and 50% of patients will
die. DIC with sepsis has a significantly higher rate of death than DIC associated with trauma.

Eclampsia (choice A) can lead to endothelial injury that promotes disseminated intravascular coagulation
(DIC). However, a patient with DIC caused by eclampsia should be accompanied by hypertension,
proteinuria, and seizures.

Most placental infarcts (choice B) are small and insignificant and do not lead to disseminated intravascular
coagulation (DIC).

Amniotic fluid embolis (choice C) is a rare complication of late third trimester pregnancy with a high
mortality rate. Thromboplastins released into the circulation promote disseminated intravascular
coagulation (DIC). However, patients typically present with dypsnea.

The shock resulting from a ruptured tube due to ectopic pregnancy (choice D) leads to hypoxia and
endothelial damage with resultant disseminated intravascular coagulation (DIC). However, in this case the
pregnancy was intrauterine.

141.

A 20 year old unmarried (sexually inactive) female comes with complaints of a 5 month history of
amenorrhea . Her menarche was at 12 years and she feels that she has gained weight over the past year
despite her strict dieting. Her Physical examination shows normal secondary sexual characteristics. Her
past medical history is non-contributory although there is a family history of depression. Serum Prolactin,

165
Estradiol and HCG levels are normal.

What is the most likely diagnosis:

Asherman's syndrome

Anorexia nervosa

Pregnancy

Body dysmorphic disorder (BDD)

Hypothyroidism

Explanation

The correct answer is choice B

Anorexia nervosa is an eating disorder where food or type of food is restricted on a daily basis. The
sufferer has less than 85% of normal body weight. Symptoms may include:

 constipation
 thining hair
 brittle nails
 dry skin
 shrunken breasts
 amenorrhea
 bradycardia
 purplish skin over arms and legs from decreased circulation

Despite the amenorrhea, serum hormonal levels tend to be normal.

Asherman's syndrome (also known as uterine adhesions) (Choice A) which can present as amenorrhea
occurs as a state of adhesions in the uterine cavity secondary to scars from instrumentation, the most
common of which is a dilatation and curettage. The adhesions can also occur after myomectomy, cesarean
section and IUD insertion. A severe form can occur after chronic endometritis from genital tuberculosis.

Pregnancy (Choice C) presents as amenorrhea but the HCG would be elevated.

Body dysmorphic disorder (BDD) (Choice D) is listed as a somatoform disorder that affects up to 2% of the
population. BDD is characterized by excessive rumination over an actual or perceived physical flaw. BDD
has been diagnosed equally among men and women. While BDD has been misdiagnosed as anorexia
nervosa, it also occurs comorbidly in 25% to 39% of AN cases.

Thyroid dysfunction (Choice E) can also cause amenorrhea. However a TSH would be abnormal as would a
thyroid panel and additional symptoms would be present. Symptoms include the following:

 fatigue
 weakness
 weight gain
 coarse dry hair
 hair loss
 cold intolerance
 muscle cramps

166
 depression
 memory loss
 amenorrhea

 decreased libido

142.

A 21 year-old gravida 2, para 1 mother presents at your clinic at 17 weeks gestation. Her history includes
gestational diabetes mellitus during her first pregnancy which was treated successfully with dietary
therapy and glyburide. The pregnancy resulted in a healthy child who is now 2 years of age, and since the
diabetes had resolved the glyburide was discontinued. Tests for diabetes, including oral glucose tolerance
tests and assays for hemoglobin A1C ,all are normal during the current pregnancy. However, a triple test
shows alpha-fetoprotein levels higher than normal for the gestational age.

The high alpha-fetoprotein finding may be associated with all of the following except:

multiple gestation

a neural tube defect

22q11 deletion syndrome

folic acid deficiency

maternal age

All of the above are true

Explanation

The correct answer is choice C.

A higher than normal level of AFP may mean that the mother is carrying twins or triplets (choice A). Of
course, generally, this is something that would be known earlier due to ultrasound testing demonstrating
multiple gestation.

High AFP also suggests that the fetus is at risk for conditions that develop when parts of the embryo do not
close when they should. Such conditions include omphalocele, gastroschisis, and neural tube defects
(choice B), such as spina bifida.

Gastroschisis is a defect in the abdominal wall, resulting in intestines developing outside of the abdomen of
the fetus. Other organs also may develop outside of the abdomen. Omphalocele is similar to gastroschisis,
except that in omphalocele intestines, and organs such as the liver, though outside the abdomen are
contained within a sac.

The neural tube is at the midline of the embryo and develops into the central nervous system. When it
does not close completely, parts of the spinal cord can be covered incompletely, usually in the lumbar and
sacral levels.

Nonclosure of the neural tube also can occur at the level of the head, resulting in anencephaly, resulting in
a lack of a forebrain and parts of the skull and scalp that go over it. Neural tube defects are associated
with deficiency of folic acid (choice D) and, in a rather complex way, with maternal age (choice E). While
some studies have demonstrated a U-shaped curve with high risk for both older and younger mothers,
other studies have suggested an elevated risk at one age extreme or the other.

22q11 deletion syndrome (diGeorge syndrome) (choice C) results from deletions in chromosome 22

167
manifests with several abnormalities that may immune deficiency, hypoparathyroidism, and cardiac
anomalies, such as tetrology of Fallot, which consists of pulmonary stenosis, right ventricular hypertrophy,
a ventricular septal defect, and an aorta that overrides the right ventricle in part. An aorta overriding the
right ventricle does not mean that the aorta is completely over the right ventricle; generally, it overrides
both ventricles, instead of overriding only the left. This places the aorta over the ventricular septal defect
so that circulation in the heart is more like the circulation in the three-chambered heart of an amphibian.
While diGeorge syndrome is associated with fetal alcohol syndrome, phenylketonuria (PKU) in the mother,
and fetal hydantoin and carbamazepine syndromes, it is not associated with an elevated AFP.

143.

A 55 year old woman presents for her annual examination. Her last spontaneous menstrual period was 3
years ago, and she has been reluctant to use hormone therapy because of a strong family history of
breast cancer. She complains of diminished interest in sexual activity and discomfort during sexual
intercourse for the past few years.

Which of the following is the most likely cause of her complaint?

Decreased vaginal length

Decreased ovarian function

Increased alienation from partner

Untreatable sexual dysfunction

Physiologic anorgasmia

Explanation

The correct answer is choice B.

With the decline in ovarian function and resultant decline in estrogen levels, postmenopausal women may
experience a number of physiologic and psychological changes. Decreased levels of cicrulating estrogens
lead to dryness and thinning of the vaginal walls, which can contribute to dyspareunia and decreased
interest in sexual activity. While systemic hormone therapy with estrogen is the most effective treatment
for symptoms associated with the menopausal transition, in this case the patient is reluctant to use
hormone therapy due to a strong family history for breast cancer. Alternative options to systemic estrogen
therapy that may be appropriate in this case include local estrogen therapies such as tablets or
suppositories that are applied directly into the vagina. Vaginal estrogen tablets have been shown to
provide relief for the symptoms of atrophic vaginitis related to menopause and do not yield a marked
elevation in circulating estrogen levels. Other non-hormonal options include the use of vaginal lubricants
during intercourse.

A decreased vaginal length (choice A) is not likely to be the cause of decreased interest in sexual activity
following menopause.

Alienation from her partner (choice C) is a less likely than declining ovarian function as an explanation for
her lack of interest in sexual activity and discomfort during intercourse.

Vaginal changes during menopause can be managed to some extent with systemic or local hormone
therapy and do not represent an untreatable sexual dysfunction (choice D).

Physiologic anorgasmia (choice E) is unlikely if the patient's complaints are recent and typically does not
present in the postmenopausal years.

168
144.

A 16-year-old girl presents with her mother to the gynecologist because her menstrual periods have not
started. History reveals that the patient’s mother and older sister started their menstrual periods at age
13 years. Physical examination reveals normal breast size and normal pubic and axillary hair. Abdominal
exam revealed no masses. Karyotyping showed normal chromosomes. The patient is 5 ft 3 in tall and
weighs 120 lb.

What is the most probable diagnosis?

Hypothalamic dysfunction

Pituitary dysfunction

Testicular feminization syndrome

Imperforate hymen

Rokitansky-Küster-Hauser syndrome (vaginal and uterine agenesis)

Explanation

The correct answer is choice E

Rokitansky-Kuster-Hauser Syndrome is a rare condition of vaginal and uterine agenesis. Despite the
absence of the vagina or presence of a short vaginal pouch and absence of the uterus, normal ovaries are
present. This congenital malformation occurs due to failure of the Mullerian ducts to develop and is the
second most common cause of primary amenorrhea. It is discovered when a female presents with primary
amenorrhea and normal breast developmentas well as normal adrenarche (pubic hair). Diagnosis can be
made by pelvic examination and ultrasound for confirmation. Less common associated symptoms include:

 kidney problems
 Hearing loss
 bone malformations

Renal anomalies occur in 25-35% of females with Mullerian agenesis.

Patients with Androgen insensitivity (testicular feminization syndrome) (Choice C) also present with
primary amenorrhea and normal breast development. However, they have a slightly elevated serum
testosterone levels and have a normal-appearing tests in the abdomen. Karyotype is male (46 XY). The
condition is X-linked recessive and is due to a lack of androgen receptors.

An imperforate hymen (Choice D) can be easily detected by pelvic examination. If the female has already
started menses, one may find a bulge in the skin of the hymen where the flow was obstructed.

Normal menstrual cycles are regulated by gonadotropin secretion. FSH which is present at low levels
during menses, rises slightly in the proliferative phase of the cycle until just before ovulation when both
FSH and LH become variable and finally rise rapidly. After ovulation, the titer of both hormones drop and
the next menses is triggered. If there is hypothalamic dysfunction (Choice A), there is a deficiency of the
FSH and LH so that no fluctuations exist and so the menses is not signaled.

In pituitary dysfunction (choice B) there is an elevation of the prolactin level. The body is tricked into
believing that there is a pregnancy and amenorrhea occurs. A simple blood test can verify this state.

145.

A 25 year old primigravida at 34 weeks gestation is thought to be small for dates by her physician and is

169
sent for a sonographic evaluation. The ultralsound shows the biparietal diameter to be appropriate for a
34 weeks gestation. The abdominal circumference is appropriate for 30 weeks gestation. The
head:abdominal circumference ration is <1. The estimated fetal weight is <10th percentile for 34 weeks
gestation. The amniotic fluid is decreased.

Which of the following is the most likely diagnosis?

symmetrical IUGR

asymmetrical IUGR

congenital anomaly

congenital infection

unknown gestational age

Explanation

The correct answer is choice B

The most common usage for Intrauterine growth retardation is a fetus whose estimated weight is below
the 10th percentile for its gestational age and whose abdominal circumference is below the 2.5th
percentile. The etiology of intrauterine growth retardation (IUGR) can be either fetal or maternal.

Fetal causes:

 aneuploidy
 trisomy 13
 trisomy 18
 triplody
 infection (with cytomegalovirus or toxoplasmosis)

Maternal causes of IUGR:

 use of recreational durgs eg. marijuana


 alcohol consumption
 placental insufficiency
 diabetes
 late conception
 history of having IUGR infant in the past

Asymmetrical vs symmetrical IUGR- two types of growth restriction:

 Head-Sparing effect is when the fetal abdomen is small but the head and extremities are normal
or near normal. This occurs in most cases of IUGR, especially those secondary to primary
placental insufficiency. This is asymmetrical IUGR.
 Symmetrical IUGR is when the fetus is small overall, which occurs in severe, early onset cases
including those due to chromosomal anomalies.

This baby is suffering from asymmetrical IUGR. If it were symmetrical (Choice A), both head
circumference and abdominal circumference would be less than the gestation age simultaneously and the
ratio would be one. Asymmetric growth restriction is due to a fetus who is undernourished and is directing
its nutrition and energy to maintaining vital organ growth such as the heart and brain at the expense of

170
the liver, muscle and fat. This type of growth restriction is usually the result of placental insufficiency.

If this baby had a congenital anomaly (Choice C), it would be detected during the ultrasound.

Congenital infections (Choice D) cause IUGR but it would be symmetric. Oligohydramnios is also seen.

Unknown gestational age does not play into this case because the head and abdominal circumference ratio
would be one.

146.

A 24-year-old woman, gravida 2, para 1, presents to the emergency department at 36 weeks’ gestation
after a motor vehicle accident in which she was a restrained driver. She is complaining of abdominal pain
and light vaginal bleeding, with a lot of vaginal pressure. Vital signs are: blood pressure, 100/50 mm Hg;
pulse, 99 beats/min; respirations, 20 breaths/min.

She has a history of a previous cesarean section for an “abnormal fetal heart rate.” During the secondary
survey, the fetus is evaluated. The nurse and the emergency department resident call you to evaluate the
fetal heart rate tracing. The late decelerations found by the fetal heart rate monitoring are combined with
maternal uterine contractions.

What is the next step in management?

Continued observation

Emergency cesarean section

Ultrasound to rule out placental abruption

Vaginal examination to rule out labor


Click on image to Zoom
Induction of labour with forceps delivery

Explanation

The correct answer is choice B

This patient may be having pain secondary to the start of uterine rupture with a history of a previous
Cesarean Section and trauma. This is a catastrophic event if not treated immediately with the possibility of
death to both mother and fetus. Another diagnosis in the differential is labour brought on by trauma.

However, the Cesarean Section should be done to hasten the birth of the baby who is suffering from fetal
distress as witnessed by the late decelerations. Common causes of late decelerations include:

 Excessive uterine contractions


 Fetal acidosis
 Fetal Hypoxemia
 Maternal hypotension
 Uteroplacental insufficiency
 Variable decelerations vary in the timing of onset and length of duration; they represent cord
compression

Late decelerations occur when a fall in the level of oxygen in the fetal blood triggers chemoreceptors in the
fetus to cause reflex constriction of blood vessels in nonvital peripheral areas in order to divert more blood

171
flow to vital organs such as the adrenal glands, heart, and brain. Constriction of peripheral blood vessels
causes hypertension that stimulates a baroreceptor mediated vagal response which slows the heart rate.
The time consumed in this two step process accounts for the delay in the timing of the deceleration relative
to the contraction.

Continued observation (Choice A) would be detrimental as the fetus is already compromised and needs to
be delivered as soon as possible.

The remaining three choices are not viable for the above reasons.

147.

After 12 hours of labor, a healthy term baby boy is born. Soon after birth, the neonate exhibits irritability
and seizure activity. There are no congenital anomalies noted in the new born. The baby weighs 4120 gm
and had Apgar scores of 8 at 1 minute and 9 at 5 minutes. Mother is slightly obese and smoked through-
out the pregnancy.

What is the most likely cause of the neonates seizures:

Hyperglycemia

hyponatremia

Hyperbilirubinemia

Hypercalcemia

Hypoglycemia

Explanation

The correct answer is choice E

The mother likely exhibited gestational diabetes during pregnancy as the baby has LGA (larger than
expected size which stems from macrosomia). Hypoglycemia with associated sequelae is a large risk for
the newborn because of the increased amount of circulating insulin. While the mother may be insulin
resistant, leading to an increase in insulin manufacture, this is circulated to the baby prior to birth which
causes the sharp decrease in the neonatal blood sugar. This must be recognized with diabetic mothers and
checked immediately upon birth. Hyperinsulinism, or persistent hyperinsulinemic hypoglycemia of infancy
(PHHI), is the most common cause of hypoglycemia in the first 3 months of life. It is well recognized in
infants of mothers with diabetes.

Hypoglycemia in the first few days after birth is defined as blood glucose <40

mg/dL. In preterm infants, repeated blood glucose levels below 50 mg/dL may be associated with
neurodevelopmental delay. SIGNS AND SYMPTOMS of hypoglycemia are nonspecific and include:
jitteriness, irritability, lethargy, seizures, apnea, grunting and sweating (uncommon). Hypoglycemic
infants may not always be symptomatic. Therefore, routine glucose monitoring for at-risk infants is
mandatory. Lack of symptoms does not guarantee absence of long term sequelae.

Management of hypoglycemia in infants includes:

 Draw blood for stat glucose measurement.


 Give IV bolus of 2-3 mL/kg of D10W.
 Begin continuous infusion of D10W at 4-6 mg/kg/min.
 If infant of diabetic mother, begin D10W at 8-10 mg/kg/min (100-125 cc/kg/d).

172
 Repeat blood glucose in 20 min and pursue treatment until blood sugar >40 mg/dL

Hyperbilirubinemia (Choice C) in the newborn causes jaundice and is most often due to ABO
incompatibilities or Rh incompatibility. In utero one may see hydrops fetalis. It can also occur with red
blood cell breakdown due to sepsis. The first symptom is jaundice and poor feeding. However if left
untreated, later symptoms include muscle stiffening, back arching, seizures and fever.

Hypercalcemia (Choice D) in the newborn is rare but if present, can cause cardiovascular collapse and
neonatal seizures.

Hyponatremia (Choice B) is very rare in newborns and is generally the result of excess fluid intake unless a
salt-losing pathology is present and as such, can generally be prevented. No the correct diagnosis in this
case although can present in a similar clinical fashion.

148.

A 35-year old woman at 28 weeks of gestation is at risk for preeclampsia. She is referred to your
maternal-fetal medicine colleagues at the hospital for further work-up of her condition. From history, her
previous pregnancies have been otherwise normal and there was no history of diabetes, hypertension or
fetal abnormalities. She had had proper pre-natal care thus far.

Which of the following results would be improved by starting low-dose aspirin at 14 weeks of gestation?

Preeclampsia, but not IUGR or preterm birth

IUGR and preterm birth, but not preeclampsia or severe maternal hypertension

Preeclampsia, IUGR, preterm birth, and gestational hypertension

Severe preeclampsia and maternal hypertension, but not IUGR

Prevents IUGR and postdates, but not preeclampsia or hypertension

Explanation

The correct answer is choice C

According to scientific reviews by Bujold and colleagues, women who initiated aspirin therapy at 16 weeks
of gestation or less, had significantly less risk for preeclampsia (prevalence 9.3% vs. 21.3% in the control
group.

They also had much less risk for IUGR (7% vs. 16.3% in the control group). Low-dose aspirin was also
associated with a reduced incidence of severe preeclampsia, gestational hypertension and preterm birth.

Dr. Emmanuel Bujold from Universite Laval, quebec stated "women considered at high-risk to develop
preeclampsia should take low-dose aspirin every day starting at 16 weeks". Because preeclampsia and
IUGR are related to defective placentation, inadequate perfusion and placental ischemia lead to endothelial
dysfunction with platelet and clotting system activation.

It was also discovered that the gestational age was 1.4 weeks greater with treatment than with placebo
with no increased risk for antepartum bleeding.

149.

A 62-year-old woman has a 1.5-cm invasive squamous-cell carcinoma of the vulva arising from the right
labia majus.

173
Which of the following anatomical routes best describes the potential path of lymphatic spread of this
vulvar carcinoma?

Superficial inguinal lymph nodes to Cloquet’s node to deep inguinal (femoral) lymph nodes to iliac
lymph nodes

Superficial inguinal lymph nodes to deep inguinal (femoral) lymph nodes to Cloquet’s node to iliac
lymph nodes

Cloquet’s node to superficial inguinal lymph nodes to deep inguinal (femoral) lymph nodes to iliac
lymph node

Superficial inguinal lymph nodes to deep inguinal (femoral) lymph nodes to iliac lymph nodes to
Cloquet’s node

Cloquet's node to deep inguinal lymph nodes to iliac nodes

Explanation

The correct answer is Choice B.

Lymphatic drainage from vulvar lesions proceeds first to the ipsilateral superficial inguinal lymph nodes
located in the femoral triangle. This area can be considered as a 'sentinel" node area for potential biopsy to
evaluate if lymphatic spread has occurred. The superficial inguinal lymph nodes drain to the deep inguinal
nodes, the most cephalad of which is known as Cloquet's node. Drainage then follows to the medial portion
of the external iliac nodes and then through pelvic to aortic lymph channels.

Vulvar cancer metastasizes by both lymphatic and hematogenous routes as well as direct spread to
adjacent anatomical structures. Lymphatic apread to regional lymph nodes can occur early in the course of
disease, even in patients with small lesions. Hematogenous spread typically occurs later in the course of
the disease and is rare in patients without inguinofemoral lymph node involvement.

150.

A neural tube defect may present as a skull abnormality, spinal defect, or both in the same individual.
"Open" lesions are associated with the appearance of alpha-fetoprotein and a CNS-specific cholinesterase
in amniotic and certain other fluids.

Which of the following prenatal diagnostic procedures can detect a pregnancy involving an individual with
a neural tube defect the earliest?

chorionic villus sampling

maternal serum screen

Ultrasound

amniocentesis

periumbilical blood sampling

Explanation

174
The correct answer is choice D

Alpha-fetoprotein is a major plasma protein manufactured by the yolk sac and liver during fetal life and is
measured using maternal blood or amniotic fluid as a screening test for developmental abnormalities. It is
increased mainly in open neural tube defects and omphalocoele and decreased in Down syndrome.

The AFP (alpha-fetoprotein) is measured in a blood test performed between 16 and 20 weeks gestation. If
the levels are abnormal, it denotes a possible problem with the fetal spine or brain. While a problem has
been detected, it is not definitive, only that this patient should have a more thorough investigation- done
by an amniocentesis to determine if the AFP is abnormal in the fluid surrounding the fetus. This test can
be done accurately as early as 15 weeks after conception. Karyotype is done simultaneously to rule out
Down syndrome or other chromosomal abnormality as well as a neural tube defect.

Chorionic villus sampling (Choice A) is performed between 10-12 weeks gestation and samples the
placenta for a karyotype to rule out chromosomal abnormalities. However, it is not definitive for neural
tube defects.

Maternal Serum Alpha Fetoprotein (MSAFP) (Choice B), a screening test that is performed on a pregnant
woman's blood at approximately 16-18 weeks of pregnancy and is most sensitive for detection during this
time period.

Blastomere diagnosis (Choice C) involves the removal of one or two blastomeres from an in vitro fertilized
embryo with a micromanipulator (biopsy) without affecting the viability of the embryo to determine any
chromosomal defect. It will not denote any neural tube defect.

Periumbilical blood sampling (Choice E) which is performed at approximately 18 weeks gestation, takes a
sample of fetal blood from the umbilical cord under ultrasound to identify disorders such as Rh
incompatibility or chromosomal problem.

151.

A 25-year-old gravida 3 para 2 woman, carrying intrauterine twin gestations presents near term for her
ultrasound evaluation. At this time an estimated fetal weight of Twin A is 2290 gm and twin B weighs an
estimated 2910 gm. Twin B exhibits hydrops fetalis.

Which of the following intrauterine treatments may not be useful?

repeated amniocentesis

fetal exchange transfusions

intrauterine laser surgery

broad spectrum antibiotics

serial ultrasound assessments

Explanation

The correct answer is D

Twin-to-twin transfusion Syndrome, also known as TOPS (twin oligohydramnios-polyhydramnios Sequence)


is a complication of disproportionate blood supply in monochorionic pregnancies where two or more fetuses
share a chorion and a single placenta. It results in a high morbidity and mortality rate (60-100%). In
sharing a single placenta, the blood supplies can become connected so that they share blood circulation.
Although each fetus has its own placental portion, the connecting blood vessels allow blood to pass from

175
one to the other.

Depending on the type and extent of the anastomoses, blood can be transferred disproportionately causing
the donor twin to have decreased blood volume thereby retarding the donor's development and growth.
This also causes the donor to have a decreased urinary output leading to oligohydramnios. The strain on
the recipient's cardiac output can eventually lead to heart failure with higher urinary output leading to
polyhydramnios.

Treatment may require repeated amniocentesis during pregnancy with exchange transfusion to the
recipient twin for body fluid reduction and fetal laser surgery may be done to interrrupt the blood flow from
one twin to the other.

Antibiotics would be administered for either an obvious infection like cystitis or chorioamnionitis which
presents with possible high temperature, tachycardia, tender uterus and foul vaginal discharge. These
conditions would not cause the weight disparity in the twins, nor the hydrops.

152.

During a routine speculum exam you note that while the introitus, vagina, and cervix appear normal,
there is thin, white, homogeneous discharge that adheres to the vaginal wall and vestibule, and has a pH
of 5.0. On further questioning, the patient notices some slight urinary frequency, but no vulvar irritation,
pelvic pain, or dysuria.

Which of the following would you expect to find on microscopic examination?

25-50 white blood cells per high power field and motile trichomonads

addition of KOH yields a fishy or amine odor

with the addition of KOH may see hyphae or spores

0-2 white blood cells per high power field and budding yeast

2-10 white blood cells per high power field and clue cells less than 20% per high power field

Explanation

The correct answer is Choice B.

The symptoms above are suggestive of Bacterial Vaginosis. While some women will complain of a fishy
vaginal odor, 50-75% of women are asymptomatic, and diagnosis is usually made based on wet mount.
Amsel criteria requires three of the following:

 Thin, white, homogeneous discharge adhering smoothly to the vaginal wall.

 Vaginal fluid pH greater than 4.5


 Clue cells on wet mount greater than 20% per high-power field (HPF)
 Fishy or amine odor of discharge upon addition of KOH or positive whiff test

Bacterial vaginosis (BV) is an extremely common polymicrobial syndrome and is characterized by thin,
white, homogeneous vaginal discharge with fishy odor without other vulvovaginal symptoms like irritation,
pruritus, or dyspareunia. Due to the lack of demonstrable inflammatory response, clinically and via
microscopy (0-2 white blood cells per high power field) the term vaginosis was adopted instead of
vaginitis. Treatment for BV includes antibiotics, such as oral or topical metronidazole and clindamycin.
Recurrances are common. While considered an overgrowth of normal flora and generally benign by some,
there are some serious potential complications associated with BV, especially in regards to pregnancy, such

176
as preterm delivery, preterm premature rupture of membranes, chorioamnionitis, and postabortion pelvic
inflammatory disease (PID). BV can also be associated with infections after invasive gynecologic
procedures, urinary tract infections (UTIs), and human immunodeficiency virus (HIV) susceptibility. While
not considered a sexually transmitted infection (STI), BV prevalence is higher in women who are sexually
active. Differential diagnosis include:

 Trichomoniasis, which is an infection of trichomonas vaginalis, a sexually transmitted protozoan


infection characterized by frothy yellow-green malodorous vaginal discharge, vaginal pruritus,
superficial vulvovaginal erythema, punctuate "strawberry" cervix, cervical friability, vaginal pH
greater than 4.5, and white blood cells on wet mount microscopy. Whiff test is usually positive.

 Vulvovaginal candidiasis, which is caused by candida albicans (yeast) with findings of cottage
cheese-like non-odorous vaginal discharge, vulvovaginal pruritus, pain, and erythema. Normal
vaginal pH of <4.5 and wet mount findings of yeast, pseudohyphae, and 2-10 or greater white
blood cells per high power field are diagnostic. Whiff test is usually negative.
 Cervicitis caused by sexually transmitted infections (ie Gonorrhea and Chlamydia) can be
asymptomatic in women or present with mild symptoms similar to those of BV. Endocervix will
often be friable on exam. Wet mount should indicate a white blood cell response over 10 per high
power field. Diagnosis made via DNA amplification, culture or PCR for the specific pathogen
tested.
 Atrophic vaginitis is caused by low estrogen and generally seen in postmenopausal
women. Symptoms include dysuria, vulvar pain, pruritis, and spotting. Clinical findings include
vulvar atrophy with loss of rugal folds, vaginal pH >5. Microscopy usually demonstrates white
blood cells, negative whiff test, and decreased normal flora.

Choice B describes a positive whiff test, which along with the clinical examination, completes the Amsel
criteria for diagnosis of Bacterial Vaginosis. In the absence of a microscope, whiff test can be a fairly
reliable diagnostic measure for Bacterial Vaginosis. In a prospective observational study of 269 women, the
most specific variable is the positive amine odor (93%) with the addition of KOH (positive whiff test).

Choice A is incorrect because it describes the findings of trichomonas vaginalis.

Choice C and D are incorrect because they describe the findings of vaginal candidiasis. Choice D is also
incorrect because vaginal candidiasis usually has a more evident white blood cell response on wet mount.

Choice E is incorrect because BV generally does not have a white blood cell response on wet mount and in
addition, diagnostic criteria for BV requires greater than 20% clue cell coverage.

153.

A 14 year old presents with vaginal bleeding that is more prolonged and profuse than her usual periods,
which are irregular and first started 13 months ago. She has pallor and tachycardia. She has no history of
nose bleeds or bruising and no petechiae. She is also not on any medication currently and has a negative
past history.

The next step in her evaluation includes which of the following:

Administer medroxyprogesterone (provera)

Administer conjugated estrogens (premarin)

Determine the hematocrit

Determine the estrogen level

Determine the platelet count

177
Explanation

The correct answer is choice C

The patient is bleeding heavily and has lost enough blood to display symptoms (i.e. the pallor and
tachycardia). Therefore, besides trying to stop the bleeding, it is important to see if the patient needs a
blood transfusion(which may strongly be the case due to the severity of her dsplayed symptoms.)

The key to management of abnormal bleeding is accurate diagnosis. The diagnosis starts by a scrutiny of
history and physical examination. Pregnancy, thyroid disease and blood dyscrasias need to be ruled out as
well as a pituitary problem.

So, in addition to a hematocrit, the following tests would be prudent:

 CBC with differential


 Platelet count
 Prothrombin time and partial prothrombin time
 Pregnancy test
 TSH, T3RIA and T4
 prolactin level

A pelvic sonogram will also be helpful in ruling out ovarian cysts as etiology or uterine polyps.

The fact that the periods are irregular is suggestive of anovulatory cycles in a 14 year old woman.

The anovulatory cycle has cycle lengths of varying degrees and may manifest with estrogen breakthrough
bleeding. Normal menstrual bleeding in the ovulatory cycle is a result of a progesterone decline with
corpus luteum demise; often this is described as progesterone withdrawal bleeding. As there is no
progesterone in the anovulatory cycle, bleeding is caused by the inability of estrogen that needs to be at a
level sufficient to stimulate the endometrium and support a growing endometrium.

Common causes of anovulation include:

 polycystic ovary syndrome


 thyroid disorders
 hypothalamic dysfunction
 hyperprolactinemia (which can be brought on by drug abuse)

These must be ruled out. In addition, one must rule out hormonal imbalances. Administering provera
(choice A) can stop the bleeding but an examination and blood tests including pregnancy test would be
performed first.

Bleeding is occuring on a regular basis until now whereupon she is displaying a buildup of her endometrial
layer which is sloughing. This is suggestive of unopposed estrogen. Therefore, it is not prudent to treat
the patient with further estrogen therapy at this time (Choice B)

Oral contraceptives can be used, but they may escalate an already suppressed hypothalamic-ovarian axis.
Hormonal regulation has a cure rate of nearly 90% in anovulatory patients, however.

Estrogen level (choice C) and platelet count (choice D) will be screened as part of the long term evaluation.
Since the patient has not had hypermenorrhea or abnormal bleeding in other areas until this point, a
problem with platelets is unlikely.

Cyclic oral contraceptives, cyclic progesterone therapy or oral progestins for 10-14 days, are appropriate

178
therapies.

154.

A 52 year old woman presents to your office for her annual gynecological examination. She stopped
menstruating about 6 months ago and is experiencing some hot flashes and occasional headaches.
She reveals that she drinks one glass of wine per day and smokes about 10 cigarettes per day. She
reports that she walks in her neighborhood occasionally but otherwise does not exercise much. Her body
mass index (BMI) is 27.

Her most significant risk factor for developing osteoporosis includes which of the following:

Cigarette smoking

Alcohol use

Lack of physical activity

Advancing age

Obesity

Explanation

The correct answer is choice D.

Age, female gender, and Caucasian or Asian race are the greatest risk factors for the development of
osteoporosis. The risk for developing an osteoporotic fracture increases with increasing age.

Women who are exceptionally thin and have a small frame (with a body mass index or BMI less than 19)
are also at increased risk of osteoporosis and related fractures. A sedentary lifestyle and the use of tobacco
are additional factors that have been shown to increase an individual's risk for osteoporosis. The
consumption of more than two alcoholic drinks a day also increases the risk of osteoporosis.Other risk
factors for osteoporosis include low calcium intake, corticosteroid medications, family history,
hyperthyroidism, and taking certain other prescription medications.

Osteoporosis is diagnosed based upon a dual-energy x-ray absorptiometry (DXA) test for measurement of
bone density. DXA test results are compared to the ideal or peak bone mineral density of a healthy 30-
year-old adult and reported as T-scores. A T-score of 0 means ythe BMD is equal to the norm for a healthy
young adult. Differences between your BMD and that of the healthy young adult norm are reported as
standard deviations (SDs).

The following Table shows the World Health Organization (WHO) definitions of osteoporosis.

 Normal: Bone density is within 1 SD (+1 or –1) of the young adult mean.
 Low bone mass: Bone density is between 1 and 2.5 SD below the young adult mean (–1 to –2.5
SD).
 Osteoporosis: Bone density is 2.5 SD or more below the young adult mean (–2.5 SD or lower).
 Severe (established) osteoporosis: Bone density is more than 2.5 SD below the young adult
mean, and there have been one or more osteoporotic fractures.

The U.S. Preventive Services Task Force recommends that women age 65 and older be screened routinely
for osteoporosis. The task force also recommends that routine screening begin at age 60 for women who
are at increased risk for osteoporotic fractures. Additionally, a panel convened by the U.S. National
Institutes of Health ecommended that bone density testing be considered in people taking glucocorticoid

179
medications for 2 months or more and in those with conditions that place them at high risk for an
osteoporosis-related fracture.

155.

A 21-year-old white woman gave birth to a full-term boy by an uncomplicated spontaneous vaginal
delivery about 4 hours ago. The baby’s Apgar scores were 81, 95. The prenatal course of this first
pregnancy was unremarkable, except for a maternal group-B streptococcal (GBS)-positive culture. The
membranes ruptured spontaneously, and she passed clear fluid 8 hours before birth. She received 2 doses
of penicillin at least 4 hours before delivery.

Which of the following options is appropriate management for GBS in a newborn?

A limited evaluation, including observation for 48 hours, complete blood cell (CBC) count, and blood
culture

A full diagnostic evaluation, including CBC count, blood culture, and lumbar puncture

Since the newborn received several doses of penicillin more than 4 hours before delivery, no
evaluation or therapy is needed other than 24 hours of observation

The newborn must be observed for at least 72 hours in the hospital, regardless of intrapartum
antibiotic administration

Continue with Penicillin G every 4 hours until 24 hours after the delivery

Explanation

The correct answer is choice C

Group B Streptococcus (GBS) which is found in the vagina, rectum or urinary bladder of women can be life-threatening to babies w
during or shortly after birth. Babies with GBS usually present with sepsis, pneumonia, or meningitis. Early onset is detected withi
birth. There is no sure way to prevent the GBS bacteria from being passed to a newborn at the time of birth and although it is ver
despite medical treatment, some babies still die as a result of complications from a GBS infection.

Additional symptoms include:

 emesis
 irregular heartbeat
 dyspneia
 hypothermia or fever
 drowsy, irritable, convulsions

Screening tests for GBS in mothers are done during the 35th to 37th week of pregnancy. Antibiotics are given to the mother at th
However, medicine given during labor prevent passing GBS to the infant.

Since the newborn received several doses of penicillin more than 4 hours before delivery, no additional therapy is needed and eva
needed only for 24 hours of observation.

180
181
Recommended treatment regimens for intrapartum prophylaxis for GBS disease prevention

182
**Figures adopted from the CDC website at www.cdc.gov - "Prevention of Perinatal Group B Streptococcal Disease"

156.

A 44 year old immigrant from Cuba has a 12 x 10 x 7 cm. mass in her left breast. It has been present for
the past year and has grown rapidly to its present size. Upon palpation, the mass is firm, rubbery,
completely movable. It is not attached to chest wall or to the overlying skin. There are no palpable axillary
nodes.

The most likely diagnosis will include which of the following?

183
cystosarcoma phylloides

intraductal carcinoma

Duct ectasia

fibroadenoma

Granulomatous mastitis

Explanation

The correct answer is choice A

Cystosarcoma phylloides is typically a large, fast growing mass that forms from the periductal stromal cells
of the breast. They account for less than 1% of all breast neoplasms. This is predominantly a tumor of
adult women, with very few examples reported in adolescents. Patients typically present with a firm,
palpable mass. These tumors are very fast growing, and can increase in size in just a few weeks.

Occurrence is most common between the ages of 40 and 50, prior to menopause. This is about 15 years
older than the typical age of patients with fibroadenoma, a condition with which Phyllodes tumors may be
confused. Phyllodes tumors are considered to be on a spectrum of disease that consists of fibroadenoma,
fibroadenoma variant and benign phyllodes. Some would extend the spectrum to include malignant
phyllodes tumors and frank sarcoma.

The common treatment for phyllodes is wide local excision. Other than surgery, there is no cure for
phyllodes, as chemotherapy & radiation therapy are not effective. The risk of developing local recurrence or
metastases is related to the histologic grade, according to the above-named features.

intraductal carcinoma (choice B) is the most common type of breast cancer in women. It comes in two
forms: invasive ductal carcinoma(IDC), an infiltrating, malignant and abnormal proliferation of neoplastic
cells in the breast tissue, or ductal carcinoma in situ (DCIS), a noninvasive, possibly malignant, neoplasm
that is still confined to the milk ducts (lactiferous ducts), where breast cancer most often originates. On
physical examination, this lump usually feels much harder or firmer than benign breast lesions such as
fibroadenoma.

Duct ectasia (choice C) is a condition in which there is an obstruction of the lactiferous duct. Mammary
duct ectasia can mimic breast cancer. It is a disorder of premenopausal age. Signs of duct ectasia can
include nipple retraction, inversion, pain, and sometimes bloody discharge. Histologically, dilation of the
large duct is prominent. Pathogenesis may be a reaction to stagnant colostrum.

Fibroadenomas (choice D) of the breast are small, solid, rubbery, noncancerous, harmless lumps composed
of fibrous and glandular tissue. Unlike typical lumps from breast cancer, fibroadenomas are easy to move,
with clearly defined edges. A fibroadenoma is usually diagnosed through clinical examination, ultrasound or
mammography, and often a needle biopsy sample of the lump

Fibroadenomas arise in the terminal duct lobular unit of the breast. They are the most common breast
tumor in adolescent women. They also occur in a small number of post-menopausal women. Their
incidence declines with increasing age, and they generally appear before the age of thirty years, probably
partly as a result of normal estrogenic hormonal fluctuation. Although fibroadenoma is considered a
neoplasm, some authors believe fibroadenoma arises from hyperplasia of normal breast lobule
components. The typical case is the presence of a painless, firm, solitary, mobile, slowly growing lump in
the breast of a woman of childbearing years

Granulomatous mastitis (choice E) are multinucleated giant cells and epithelioid histiocytes around lobules.
Often minor ductal and periductal inflammation is present. The lesion is in some cases very difficult to
distinguish from breast cancer. Patients usually present with a distinct firm mass mostly in the subareolar
region. It occurs on average 2 years and almost exclusively up to 6 years after pregnancy, usual age range
is 17 to 42 years. Use of hormonal contraceptives, prolactin raising medications and hyperprolactinemia

184
have been implicated in the pathogenesis or as predisposing factors.
157.

A 32 year old woman, para 2, gravida 3, is being evaluated at 16 weeks gestation. No physical
abnormalities are found on maternal physical examination and the uterine size is consistent with the
dates. The alpha-fetoprotein done as part of the maternal serum screen is found to be elevated to 3
multiples of the median for this gestational age.

Which of the following should be the next step in management of this patent?

ultrasound examination

chromosomal studies

serum acetyl-cholinesterase determination

amniotic fluid AFP determination

amniotic fluid electrophoresis

Explanation

The correct answer is choice A.

Alpha-fetoprotein (AFP) is part of the triple screen done on maternal serum to evaluate the fetus for
certain congenital conditions. In addition to AFP, the triple test measures estriol, and beta human chorionic
gonadotropin (beta-hCG) in the maternal blood. AFP tends to be low in cases of Down syndrome (Trisomy
21) and Edward syndrome (Trisomy 18), in which case chromosomal studies (choice B) on cells drawn via
amniocentesis would be the next step.

Amniocentesis is a test in which amniotic fluid is drawn through a needle under ultrasound guidance. The
fluid contains fetal cells whose chromosomes can be viewed as a karyotype and counted. Normally,
amniocentesis is recommended for women 35 years of age or older, but even for a younger mother it is
indicated if the result of her triple test suggests a higher than normal probability of a trisomy such as Down
syndrome. When chromosomal or genetic analysis is required earlier than the 15th gestational week,
another sampling procedure, chorionic villus sampling, may be conducted as early as the 10th week.

A higher than normal level of AFP may mean that the mother is carrying twins or triplets (or more), or it
can suggest that the fetus is at risk for conditions that develop when parts of the embryo do not close
when they should. Such conditions include omphalocele, gastroschisis, and neural tube defects, such as
spina bifida.

Gastroschisis is a defect in the abdominal wall, resulting in intestines developing outside of the abdomen of
the fetus. Other organs also may develop outside of the abdomen. Omphalocele is similar to gastroschisis,
except that in omphalocele intestines, and organs such as the liver, though outside the abdomen are
contained within a sac.

The neural tube is at the midline of the embryo and develops into the central nervous system. When it
does not close completely, parts of the spinal cord can be covered incompletely, usually in the lumbar and
sacral levels.

Whether multiple pregnancy or a detect in embryo structure, ultrasound examination will make it visible.
Measuring the level of AFP in the fluid drawn during amniocentesis (choice D) might be useful in confirming
whether a high AFP is indeed due to a problem with the fetus rather than the mother, but if this is the case
ultrasound would be needed anyway to determine the cause. Similarly, electrophoresis to characterize AFP
or other proteins in the amniotic fluid (choice E) is not the first priority when a technique as revealing as

185
ultrasound is available.

While measurement of serum acetyl-cholinesterase also is useful in diagnosing neural tube defects (choice
C), as in the case of the other tests, this is a priority which comes subsequent to ultrasound to rule out
other causes. In this case, the AFP being precisely triple the mean for this gestational age argues in favor
of tripleton pregnancy, while the uterine size for the age argues against it. For this reason, it is urgent that
ultrasound investigation be conducted as soon as possible.

158.

A 26 year old woman with type I diabetes on insulin is asking you about the possibility of being pregnant,
treatment of her diabetes during pregnancy, and if there are any baby risks.

Regarding diabetes and pregnancy, all of the following are true, except?

ideally, all patients should have pre-pregnancy counselling

ideally, diabetics should be controlled before pregnancy rather then during the course of
pregnancy

the objective of good diabetic control is to decrease the fetal congenital anomalies

oral hypoglycemics are useful in the management

insulin requirement varies through out the course of pregnancy

Explanation

The correct answer is choice D.

Women with type 1 diabetes are more likely to have problems with high blood pressure during pregnancy,
miscarriages and stillbirths, premature and cesarean deliveries, and birth injuries to both the mother and
the baby. Women with type 1 diabetes have more chances to have babies with high birth weights and
serious birth defects such as heart and spinal cord problems. Proper planning is the best way to safeguard
the health of the mother and her baby. A woman must begin pre- pregnancy planning, three to six months
prior to conception if possible. She should receive pre-pregnancy counseling by her obstetrician and an
endocrinologist to determine if it is safe to become pregnant (choice A).

Ideally, the woman should have her hemoglobin A1C within normal ranges prior to conception, to minimize
risk of defects or spontaneous abortion. Before a woman can determine she is pregnant, the fetus' heart,
brain, nervous system, and other organs have already begun forming during the first five weeks of
pregnancy. Unless a woman is already in control of her blood sugar at the time of inception, birth defects
can happen (choice B and C).

Diabetic complications like kidney and eye problems, heart disease, high blood pressure, thyroid problems,
high cholesterol and other preexisting medical problems must be evaluated before hand in women with
type 1 diabetes. She should be tested for retinopathy. If the results show even a touch of retinopathy, an
ophthalmologist with expertise in retinal disease should be added to her prenatal-care team. She should
undergo an exercise treadmill test and an EKG, and get assessed for a cardiovascular risk as she is at a
higher risk for pregnancy-related complications such as strokes and heart disease.

Management of type 1 diabetes is altered during the course of pregnancy even if blood glucose was under
good control before the woman become pregnant. Insulin requirements may go down in early pregnancy
and then increase in mid- to late pregnancy (choice E). There is a 10% decrease in maternal glucose levels
by the end of the first trimester especially in cases of morning sickness, therefore the daily dosage of

186
insulin decreases by 10%-20% during this period. But insulin needs rapidly increase later in pregnancy.
There is a 10%-30% increase in insulin demands during the rest of the pregnancy in type 1 diabetics.

It is necessary to control blood glucose in pregnant woman with type 1 DM to prevent hyperglycemia and
ketoacidosis. This is best achieved by frequent (usually 4-5 times a day) home glucose monitoring, diet
control (the diabetic should have the usual weight gain of pregnancy), and stabilization of exercise.

The safety of oral hypoglycemic during pregnancy has not been properly established (choice D). Insulin is
the preferred drug for blood sugar control as it cannot cross the placenta and provides more predictable
glucose control; it is used for types 1 and 2 diabetes and for some women with gestational diabetes.
Human insulin is used if possible because it minimizes antibody formation. The amount and type of insulin
should be individualized. 2/3 of total dose (60% NPH, 40% regular) is advised in the morning while the
remaining 1/3 (50% NPH, 50% regular) is taken in the evening.

159.

A 24 year old primigravida has been in labour for 6 hours. During the past 2 hours of observation the
contractions have become shorter and are weak. The membranes are bulging and she has remained at 5
cm dilation for this 2 hour period. The occiput is at station +1 and there is no molding of the fetal head.

Which of the following should be the next step in management?

administer intramuscular ergometrine

administer buccal oxytocin

apply intracervical prostaglandin gel

observe for 4 hours and reevalute

perform artifical rupture of the membranes

Explanation

The correct answer is choice E

Rupturing the membranes allows for some amniotic fluid to be delivered, making the uterine contractions
stronger. This will also give the opportunity for the physician to insert a fetal scalp electrode to better
assess the fetal well-being.

PROM is defined as rupture of membranes that occurs at term. It occurs in 3-8% of pregnancies. 50% of
patients deliver within 5 hours of membrane rupture and 95% of patients deliver within 28 hours. As the
time between the rupture of the membranes and the onset of labour increases, so may the risk of maternal
and fetal infection.

Friedman's original research in 1955 defined 3 stages of labor:

 The first stage starts with uterine contractions leading to complete cervical dilation and is divided
into latent and active phases. In the latent phase, irregular uterine contractions occur with slow
and gradual cervical effacement and dilation. The active phase is demonstrated by an increased
rate of cervical dilation and fetal descent. The active phase usually starts at 3-4 cm cervical
dilation and is subdivided into the acceleration, maximum slope, and deceleration phases.
 The second stage of labor is defined as complete dilation of the cervix to the delivery of the infant.

187
 The third stage of labor involves delivery of the placenta.

The reason for PROM is an abnormal progression of labor. The table below summarizes the finding of an
abnormal pregnancy.

*adapted
from http://emedicine.medscape.com - "Abnormal Labor"

Administering intramuscular ergometrine (Choice A) causes tetanic contractions which could harm the
fetus by possible cutting off its circulation leading to anoxia and is directly contraindicated intrapartum.

Buccal oxytocin (Choice B) is not administered because it can not be totally controlled and one would first
wait to see if labour is enhanced by the rupture of membranes alone.

Intracervical prostaglandin gel (Choice C) is used to dilate the cervix for terminating pregnancies (abortion)

Observing for four hours (Choice D) is not an option as one would take a more active role after no change
has occurred in 2 hours.

160.

A pregnant woman is concerned about the possibility of giving birth to a child with amaurotic familial
idiocy (tay sachs disease).

A study that will show whether the fetus will be affected is which of the following:

188
measurement of maternal serum alpha fetoprotein

measurement of glucose 6 phosphate dehydrogenase in amniotic cell

amniotic cell karyotype

hexasaminidase A from cultured amniotic cells

none of the above

Explanation

The correct answer is choice D

Tay-Sachs disease, also known as hexosaminidase A deficiency or GM2 gangliosidosis, is an autosomal


recessive genetic disorder which causes a deterioration of mental and physical abilities starting at 6
months of age and resulting in death by the age of four due to accumulation of gangliosides accumulating
in the brain's nerve cells. Symptoms stem from hexosaminidase A deficiency and so testing this will
diagnose the condition.

It is caused by a genetic defect in a single gene with one defective copy inherited from each parent.
Abnormally large quantities of gangliosides amass in the nerve cells of the brain, eventually leading to
premature death of those cells. There is no cure or treatment.

The disease is classified in variant forms, based on the onset timing of neurological symptoms.

Infantile:

 baby develops normally for first 6 months


 As nerve cells distend, deterioration of mental and physical abilities occur
 blindness
 deafness
 dysphagia
 paralysis

Juvenile:

 extremely rare, presenting between 2-10 years of age


 dysarthria
 dysphagia
 ataxia
 spasticity
 death between 5-15 years

Incidence is higher in the Ashkenazi Jewish population and a subset group of Cajuns in Louisiana.

Maternal serum alpha-fetoprotein (Choice A) is measured after the 14 week of gestation and suggests
whether there is an open neural tube defect (when elevated) or chromosomal abnormalities (such as Down
syndrome). It is not diagnostic, but merely indicates that an amniocentesis should be performed as a
further investigation.

Glucose 6-phosphate dehydrogenase is measured in amniotic cells (Choice B) and is an X-linked recessive
hereditary disease. Individuals withthe disease may show nonimmune hemolytic anemia in response to a
number of causes, most commonly infection or exposure to certain medications, foods or chemicals. An
example is fava beans causing a hemolytic reaction.

189
Amniotic cell karyotype (Choice C) is performed to determine a chromosomal abnormality ( eg. Down
Syndrome/ Trisomy 21)

161.

A 25-year-old sexually active woman presents to your clinic with a 3-day history of dysuria, polyuria, and
suprapubic tenderness but no comorbid conditions.

Which one of the following steps is NOT indicated?

Urinalysis

Urine culture

Treatment with TMP/SMX

Encouraging postcoital voiding

visualization of the urethra and surrounding tissue

Explanation

The correct answer is choice D

Sings and symptoms of Cystitis include:

 urinary pain
 urinary burning
 frequent or difficult urination
 urgency
 bladder pain or lower abdominal pain
 abdominal sensitivity
 dark or bloody urine
 cloudy or smelly urine

190
Although it is a good idea to get the patient to perform postcoital voiding on a regular basis to prevent
future increased risk of cystitis, it is not treatment or care for the current infection that she has.

Urinalysis (Choice A) is performed to confirm a urinary tract infection. One would expect to see white cells,
possibly red blood cells and possible cast cells within the specimen. Turbidity as opposed to a clear
specimen is also an indication.

It would be prudent to perform a Urine Culture (Choice B) so that you could tell if the antibiotic that you
have given the patient is appropriate or whether the bacterium is resistant to it.

The treatment option in Choice C is a broad spectrum antibiotic that is commonly chosen for urinary tract
infections. E.coli react to it.

Visualization of the tissue (Choice E) is wise to rule out other causes such as vulvitis or vaginitis. Herpes
simplex infections can induce dysuria.

Cystitis, caused by a urinary tract infection is the result of an invasion of bacteria into the bladder from
outside the body through the uretrha although it can occassionally come from the blood stream.

Interstitial cystitis is a condition that has recently been considered a combination of problems, such as
bladder pain syndrome(BPS) or painful bladder syndrome (PBS) and includes all cases of urinary pain that
cannot be associated with other causes such as bladder infection or stones. The bladder wall may be
irritated and become scarred or stiff. Glomerulations (Pinpoint bleeding) often appears on the wall of the
bladder. Ten percent of these cases have Hunner's ulcers- patches of broken skin on the bladder wall.

162.

An 18-year-old woman presents to your clinic with 3 days’ duration of lower abdominal pain and vaginal
discharge. She is in mild distress and has a temperature of 99.8°F. Physical examination reveals bilateral
lower quadrant tenderness, with guarding but no rebound. Pelvic examination reveals bilateral adnexal
tenderness, a friable cervix with thick yellow discharge, and significant cervical motion tenderness.

Which of the following statements concerning treatment of this condition is correct?

Hospitalization is always indicated for the treatment of this condition.

A parenteral antibiotic is required for treatment.

Treatment with fluoroquinolone antibiotics is not recommended in the management


of this condition.

If outpatient treatment is selected, a follow-up visit after one week is necessary.

HIV testing is not indicated in this condition.

Explanation

The correct answer is Choice C.

The patient is suffering from pelvic inflammatory disease (PID). The signs and symptoms described in the
question are typical for this condition. PID requires immediate and adequate antibiotic treatment with at
least two antibiotics. Outpatient treatment is acceptable in certain cases. However, hospitalization is
recommended in some cases of pelvic inflammatory disease, including cases that occur during pregnancy,
noncompliance or expected noncompliance with therapy, the inability to take oral medications, severe
illness, pelvic abscess, or need for surgery.

191
Both oral and parenteral regimens are safe and effective in the treatment of PID. At least two antibiotics
are recommended allowing broad-spectrum coverage. The two most important sexually transmitted
organisms associated with acute PID are Chlamydia trachomatis and Neisseria gonorrhoeae. Due to
increasing resistance, the 2007 CDC guidelines state that fluoroquinolones are no longer recommended as
therapy for PID if N. gonorrhoeae is a proven or suspect pathogen.

According to the U.S. Centers for Disease Control and Prevention, effective outpatient antibiotic regimens
for the treatment of PID include ceftriaxone or cefoxitin with doxycycline, with or without the addition of
metronidazole. Recommended parenteral therapies are either cefotetan or cefoxitime along with
doxycycline, or clindamycin plus gentamycin.

If outpatient therapy is administered, it is necessary to see the patient for follow-up within 48 to 72 hours
to ascertain clinical improvement. If there is no improvement after 72 hours, hospitalization and further
evaluation should be considered.

All patients with any diagnosis of an STD should be offered HIV testing.

163.

A 45 year-old gravida 2, para 2 mother gives birth to a male infant by cesarean section at 39 weeks
gestation. Although the woman's first pregnancy was uneventful, resulting in a healthy girl who now is 4
years of age, this pregnancy included a triple screen result at 16 weeks giving a 1/8 chance of Trisomy 21
(Down syndrome). Although the patient refused amniocentesis, ultrasonography at 20 and 24 weeks
revealed a femur length that was slightly, but not severely, shortened for the gestational age.

Despite this, the infant was born with an Apgar score of 8 at one minute and 10 at five minutes post birth,
and no heart murmurs were noted. Nevertheless, chromosomal studies on blood taken from the infant
reveal a karyotype of 47, XY, +21, confirming the suspected diagnosis.

This child will be at elevated risk for which of the following malignancies?:

testicular seminoma

chronic myelogenous leukemia

chronic lymphocytic leukemia

osteosarcoma

neuroblastoma

Explanation

The correct answer is choice A.

Down syndrome is associated with advanced maternal age. The risk of having a Down syndrome child rises
sharply after age 35 for a woman, and dramatically after age 40 compared to the risk for younger mothers.
Normally for a woman of advanced childbearing age, amniocentesis is conducted by the 20th gestational
week, especially if the triple test is abnormal, as it was in this case.

A triple test is a measure of the levels of alpha-fetoprotein (AFP), estriol, and beta human chorionic
gonadotropin (beta-hCG) in the maternal blood. When Down syndrome is present, AFP and estriol tend to
be too low, while beta-hCG tends to be too high for the gestational age. Based on these results, a
probability is calculated as to whether or not the fetus will have Down syndrome.

Amniocentesis is a test in which amniotic fluid is drawn through a needle under ultrasound guidance. The
fluid contains fetal cells whose chromosomes can be viewed as a karyotype and counted. When

192
chromosomal or genetic analysis is required earlier than the 15th gestational week, another sampling
procedure, chorionic villus sampling, may be conducted as early as the 10th week.

Afflicted with Down syndrome, this child will be at elevated risk for a plethora of conditions, including
certain malignancies, but not others. Down syndrome is predisposes patient's to hematologic malignancy,
namely acute lymphoblastic and acute myeloid leukemia. The association is so dramatic that for the first
five years of life the risk of developing acute leukemia is 56 times higher in Down syndrome children
compared to children of the same age group without Down syndrome. There is no association, however,
with chronic myelogenous leukemia (choice B) or chronic lymphocytic leukemia (choice C).

Interestingly, compared to the rest of the population, children with Down syndrome actually have a lower
risk of developing solid cancers, such as osteosarcoma (choice D) and neuroblastoma (choice E). It has
been suggested that this phenomenon may have to do with a high level of tumour suppressor genes in
Down syndrome. Germ cell tumors, however, are an exception to the low solid cancer incidence in Down
syndrome. Thus, studies have noted an increase in the incidence of testicular seminoma (choice A) as one
example.

164.

A 32-year-old G2 P1 woman has an uncomplicated vaginal delivery of a 30 week gestational age male
infant. Apgar scores of 5 and 7 at 1 minute and 5 minutes are recorded at the time of birth. The baby then
begins to exhibit increasing respiratory difficulty in the next hour, culminating in the need for intubation
and mechanical ventilation.

Which of the following laboratory tests during gestation is most likely to predict the complication observed
in this infant?

Maternal serum alpha fetoprotein

Chromosome analysis on fetal cells from amniocentesis

L:S ratio, phosphatidylglycerol levels

TORCH titers on maternal blood

Maternal cocaine screen on blood and urine

Explanation

The correct answer is choice C

The lecithin/sphingomyelin (L/S) ratio assesses fetal lung maturity. Sphingomyelin is present in amniotic
fluid in relatively constant amounts during the third trimester and is not contained in fluid produced by the
fetal lungs whereas lecithin which is produced by the lungs begins to increase in concentration midway
through the third trimester. An L/S ratio of 2.0 or more predicted the absence of RDS in about 98% of
neonates.

Respiratory distress syndrome begins shortly after birth and is manifest by tachypnea, tachycardia, chest
wall retractions (recession), expiratory grunting, flaring of the nostrils and cyanosis during breathing
efforts. Most cases of hyaline membrane disease can be ameliorated or prevented if mothers who are
about to deliver prematurely can be given glucocorticoids. This speeds the production of surfactant. For
very premature deliveries, a glucocorticoid is given without testing the fetal lung maturity.

Maternal serum alphafetoprotein (Choice A) is measured to screen for Down's Syndrome or a fetal neural
tube defect. The protein is elevated in the later and decreased with the former. The most accurate
screening is done between the 16th to 18th week gestation.

193
Chromosome analysis on fetal cells from amniocentesis (Choice B) are performed to detect a genetic or
chromosomal abnormality and bears no association with RDS. (respiratory distress syndrome)

TORCH titers (Choice D) test for toxoplasma, rubella, cytomegalovirus and herpes. This is done in the first
trimester to rule out viral infections that can affect the fetus. The TORCH syndrome manifest:

 microcephaly
 chorioretinitis
 cerebral calcification
 mental retardation

Cocaine (choice E) is associated with placental abruption and congenital cardiac defects due to usage.

165.

A 39-year old female presents to the office with unprovoked vulvodynia. She admits to being sexually
active. Other than this event, the patient has had no prior medical problems and is currently on no
medications.

Which of the following classes of medication is an initial option if behavioral strategies have failed?

short-acting benzodiazepines

selective serotonin reuptake inhibitors

antiseizure medications

tricyclic antidepressants

topical estrogen cream

Explanation

The correct answer is choice D

A thorough pain history should be taken to evaluate the intensity of the pain and impact on the patient's
life. Pelvic floor muscle dysfunction should not be ignored and self-help exercizes may be of help with
biofeedback and self-massage.

Localized vulvodynia includes vestibulodynia, cliterodynia and hemivulvodynia. It is characterized by


burning, stinging irritation that occurs in the vulva and lasting 3 months.

The following differential diagnosis must be ruled out:

 inflammation (cause-lichen planus)


 infection (cause-candidiasis,herpes,HPV)
 Neoplasm (cause-Paget's disease, vulvar carcinoma)
 Neurologic disorder (neuralgia secondary to spinal nerve injury or herpes)

The provoked type is due to sexual or nonsexual behavior. The diagnosis is clinical so biopsies, patch tests
and MRIs will not help confirm it.

According to recommended guidelines, tricyclic antidepressants would be the initial option for unprovoked

194
vulvodynia.

Topical creams (Choice E) may cause irritation. Capsaicin, nifedipine and ketoconazole cream have been
not shown to have any effect.

Figure 1: Vulvodynia algorithm.

166.

A 22 year old G2P3 woman presents for a routine perinatal visit. She is 42-1/2 weeks gestation by dates.
She is 170 cms tall and weighs 67 kgs. Her total weight gain is 19 kgs. She has a positive history of
chronic hypertension. She has had 5 ultrasounds performed at this pregnancy at 10, 20, 25, 30 and 40
weeks. Her pregnancy has other been uneventful and she denies any bleeding or gestational diabetes.

Which of the following ultrasounds gives the most accurate dates of pregnancy?

195
8 weeks

9 weeks

10 weeks

11 weeks

12 weeks

Explanation

The correct answer is choice C

As the pregnancy proceeds, the dating of the ultrasound can be skewed by variables such as large for
gestational age or small for gestational age babies as well as the volume of the amniotic fluid. At 10
weeks, one is measuring the crown-rump length of the fetus which tends to be quite accurate.

Dating by ultrasonographic biometry in the first half of pregnancy results in a more accurate prediction of
the delivery date than using menstrual data alone or in combination with ultrasonography.

In early pregnancy, the measurement from the top of the fetal head to the bottom of the pelvis is called
the "Crown-Rump" length. Before twelve weeks, it's difficult to measure much more. Nevertheless, this
measurement is fairly accurate. In the second trimester, (past the first 12 weeks), the other
measurements can be added. Most babies, except in cases of early IUGR and deformities, grow about the
same until 20 weeks or so. After that, the variations among babies' measurements cause increasing
inaccuracies. The femur (thigh bone) length seems to hold on to it's accuracy longer than the other
parameters, but after 36 weeks it isn't foolproof.

Naegele's Rule (Standard Pregnancy Dating Method)

To predict your due date using Naegele's Rule, take the date of your last LMP and add 7 days. Then count
back 3 months to get your due date. For example, if your LMP was June 22, adding 7 days comes up with a
date of June 29; subtracting 3 months then comes up with a due date of March 29th.

167.

Your 34 year old G0 patient has taken oral contraceptive pills for approximately five years. Currently, she
is contemplating discontinuing the birth control pills in order to conceive. She is concerned about her
fertility and asks you when she should anticipate the resumption of normal menstrual periods.

You counsel her that by 3 months after discontinuation of birth control pills, the following proportion of
women will resume normal menses:

99%

90%

80%

25%

Less than 10%

196
Explanation

The correct answer is choice A.

Approximately 99% of women experience either the resumption of spontaneous menses or pregnancy at
90 days after the discontinuation of oral contraceptive therapy. If the patient's memses have not returned
by this time, further diagnostic workup is indicated.

Women who do not menstruate after three months following cessation of OCP use should be evaluated in
the same manner as other women with secondary amenorrhea (assuming that she had regular menstrual
periods prior to OCP use). Studies have refuted the earlier belief that OCP use predisposes a woman to
irregular menstrual cycles after discontinuation of their use. Post-pill amenorrhea is a form of secondary
amenorrhea in previously-menstruating women.

After excluding pregnancy (the most common cause of secondary amenorrhea), other causes of the
condition must be ruled out. The most common causes of secondary amenorrhea in nonpregnant
women are as follows, in order of frequency and with specific examples:

 ovarian disease (PCOS, hyperandrogenism, ovarian failure)


 hypothalamic disease (functional disorders, inflitrative processes)
 pituitary disease (prolactinoma)
 uterine disease (ASherman's syndrome, or acquired scarring of the endometrium)

The intial laboratory examination of a woman with secondary amenorrhea should include measurement of
serum hCG to rule out pregnancy, serum prolactin, thyrotropin, and FSH to test for hyperprolactinemia,
thyroid disease, and ovarian failure, respectively.

168.

A 27-year-old G2P1 woman is in the 39th week of gestation. A screening ultrasound at 19 weeks revealed
no abnormalities. She goes into labor and 6 hours later has a routine vaginal delivery of a term male. An
intact placenta is delivered 5 minutes later. The infant's Apgar scores are 8 and 9 at 1 and 5 minutes.
However, within 10 minutes she develops severe dyspnea and cyanosis, followed by seizures and coma.

Which of the following is the most likely explanation for this series of events?

Disseminated intravascular coagulation

Metastatic choriocarcinoma

Cerebral infarction

Amniotic fluid embolism

Pulmonary infarction

Explanation

The correct answer is choice D

Amniotic fluid embolism is a rare obstetric emergency where upon amniotic fluid, fetal cells or other debris
enter the maternal circulation causing cardiorespiratory collapse (possibly due to anaphylactic reaction to
fetal antigens). This reaction then results in cardiorespiratory (heart and lung) collapse and coagulopathy.
Death usually occurs within a few minutes.

197
It is mostly agreed that this condition results from amniotic fluid entering the uterine veins and in order for
this to occur there are three prerequisites:

 Ruptured membranes (a term used to define the rupture of the amniotic sac)
 Ruptured uterine or cervical veins
 A pressure gradient from uterus to vein

DIC (disseminated intravascular coagulopathy) (Choice A) which is a failure of the blood to coagulate. It is
the result of a wide range of diseases. It is associated with:

 infections-streptococcus pneumoniae,malaria, histoplasmosis,aspergillosis, gram-negative sepsis


 cancers of pancreas, lung, stomach and acute myeloid leukemia
 abruptio placentae
 preeclampsia
 amniotic fluid embolism
 massive tissue injury with extensive surgery

Diagnosis is usually suggested by severe cases with hemorrhage. The PT and APTT are very prolonged and
the fibrinogen level is markedly reduced. In mild cases, there is increased synthesis of coagulation factors
and platelets, PT and APTT may be normal but fibrin degradation products are elevated.

Metastatic choriocarcinoma (Choice B) which stems from a conversion of trophoblastic disease or


hydatidiform mole or abortion or normal pregnancy, present as irregular bleeding with sudden
hemorrhage, pain and cough with bloody sputum from possible pulmonary metastasis.

Cerebral infarction (Choice C) which is an ischemic thype of stroke due to a blood vessel disturbance can
be atherthrombotic or embolic with contralateral hemiparesis found.

Pulmonary infarction (Choice E) which is localized necrosis resulting from obstruction of the blood supply.

169.

A 40-year-old nonsmoking woman with well-controlled type 1 diabetes mellitus diagnosed in childhood
and no vascular disease seeks contraception. She states that she wants “something convenient” that does
not require a daily reminder. She is denies smoking and has no allergies.

Which one of the following contraceptive methods is the safest option for this patient following generally
accepted clinical guidelines?

Progestin intrauterine device

Contraceptive ring

Copper intrauterine device

Combined estrogen-progestin oral agent.

Diaphragm

Explanation

The correct answer is choice C

The copper intrauterine device can be inserted in the office and is the most common of IUDs. Copper wire is

198
wound around the stem of the T-shaped device. The copper IUD is highly effective and can stay in place for at
least 10 years. The copper is toxic to sperm and thus the uterus and fallopian tubes produce fluid that kills
sperm in addition to preventing any fertilized egg from implanting in the endometrial wall.

Choices A, B, and D secrete hormones as their action to control contraception and is relatively contraindicated
in a diabetic and in her age group.

Choice E does not afford protection without a reminder which the patient was requesting.

Table 1: Review of Contraindication (absolute and relative) for OCP's

170.

A representative of the law firm of Flotsam, Flotsam & Jetsam enters your office one day and serves you
with a subpoena. The subpoena requires you to appear in regard to litigation by one of your former
patients, a 31-year-old woman who is claiming that your malpractice led to medical expenses, pain, and
suffering because she developed invasive cervical carcinoma. She was last seen in your office 10 years
ago. She had been your patient for 5 years, receiving Pap smears in 4 of those 5 years. Your records
indicate that she had no abnormal Pap smears.

After discussion with your malpractice carrier's attorney, which of the following conclusions is most
appropriate?

199
The patient should have continued to return for yearly Pap smears

You are at fault in this case and should avoid a trial

The laboratory to which the Pap smears were sent is at fault for missing abnormal cells

Nothing anyone could have done would have prevented this carcinoma

The patient's health insurer is at fault for not covering the full cost of Pap smear testing

Explanation

The correct answer is choice A

The patient should have continued to return for yearly Pap smears. A pap smear is an excellent screen for
abnormal cells and would have afforded the patient treatment once the cytology showed aberrant cells
prior to the progression of an invasive lesion.

As long as you have a well-documented chart stating no complaints of abnormal vaginal bleeding
(postcoital bleeding is a common symptom) or a malodorous discharge (another common complaint with
cervical cancer) along with a normal screening pap result, your case is defendable and you are not at fault
(Choice B).

A laboratory error (Choice C) is not culpability on your part. Copies of slides can be reviewed to verify the
result but if found to be read wrong, this does not shift the blame to you as the practitioner.Nothing
anyone could have done would have prevented this carcinoma is simply not true (choice D).

Detection of dysplastic or malignant cells on the pap smear would have allowed for successful treatment
with further prevention of cancer. Failure to comply with screening guidelines caused failure of treatment
to the cervix with a subsequent poor outcome. Health insurance companies do not incur liability for
interactions or decisions made by patients and healthcare providers (Choice E). Treatments and diagnostic
tests are ordered by necessity in lieu of costs. Decisions to cover the tests or not are based on economic
data, not medical factors.

Figure 1: Summarized procedure schedule for PAP smears

200
Figure 2: Summarized results and followup procedures for PAP smears

201
*adopted from www.acog.org

171.

As a male medical student, you find the female hormonal block a real challenge. With all the different
contraceptive methods, you are trying to memorize all of the important facts.

Regarding contraception for the woman over 35 years of age, all the following are true except:

Healthy women of 35 yrs without coronary heart disease risk factors, can use low dose oral
contraceptive pills with confidence

The use of combined oral contraceptive pills results in significant decrease in the incidence of
ovarian cysts

The ratio of ectopic pregnancy to total pregnancies conceived is higher with progestin only IUD's
than that with copper containing IUD's

Due to increased risk of breast cancer, the use of combined oral contraceptive pills should be
discouraged in women over 35

The NuvaRing contains the same hormones as many oral contraceptives

Explanation

202
The correct answer is choice C

35 year old women without heart problems can use the low dose oral contraceptive pills (choice A) as long
as they do not smoke. However, there is a higher risk of blood clots or cardiovascular events.

Oral contraception does result in a decrease in ovarian cyst formation (choice B) due to the lack of
ovulation which is responsible for some functional cyst development.

The Levonorgestrel IUD (also known as Mirena) (Choice C) releases levonorgestrel which is a form of
progestin is highly effective at preventing conception and unlike other IUDs may decrease the incidence of
pelvic inflammatory disease, thereby decreasing the incidence of ectopic pregnancies in comparison to the
copper IUD.

The risk of endometrial and ovarian cancers is reduced with the use of OCs, while the risk of breast and
cervical cancers is increased. Hormonal factors that increase the risk of breast cancer include conditions
that may allow high levels of hormones to persist for long periods of time, such as beginning menstruation
at an early age (before age 12), experiencing menopause at a late age (after age 55), having a first child
after age 30, and not having children at all. Thus, oral contraception usage in women over the age of 35 is
discouraged for breast cancer risk (choice D).

The NuvaRing, manufactured by Merck pharmaceuticals, is a small flexible contraceptive ring that releases
a low dose of estrogen and progestin over three weeks. . It is inserted manually into the vagina and left in
place for three weeks. It is then removed whereupon a menstrual flow ensues. The benefit of the ring
includes ease of use with a lower incidence of estrogenic side effects like breast tenderness and nausea.

However, the ring is not without side effects. Device-related adverse reactions include sexual problems
with expulsion, vaginitis, headache, yeast infections, leukorrhea, sinusitis, nausea and weight gain. It has
the same cardiovascular contraindications associated with combined oral contraceptives such as myocardial
infarction and stroke. Thees risks are increased with other risk factors like smoking, recent surgery or a
history of cardiovascular disease.

172.

An 32 year-old woman presents in the ER with fever, nausea, and severe pain in the right lower
abdominal quadrant. She is married, nulliparous with no major illnesses or surgery in her history, though
she is a former smoker who quit at age 25. One physical examination, her blood pressure, pulse rate, and
respiratory rates are normal, at 110/70 mmHg, 70 beats/min, and 16 breaths/min, respectively. Her
temperature is elevated at 38.3°C (normal 37.0). You note rebound tenderness in the abdomen which is
most severe around the right iliac fossa.

Highly relevant information from this patient's social history would include all of the following except:

her ethnic or national origin

the number of sexual partners she's had

her method of contraception

her religious practices

the number of years she's been married

Explanation

The correct answer is choice D.

Without more information than is given, the interpretation of this case is open to several possible

203
diagnoses. While much of the differentiation can be resolved through a laboratory workup, the social
history is vital in helping to guide such a workup.

With or without a social history, a patient with this type of presentation would be worked up for acute
appendicitis. While the incidence of appendicitis peaks in late adolescence, it does not decline until the
geriatric years. Thus, the presence of rebound tenderness, together with a fever in this patient should put
appendicitis high in the differential diagnosis. Additional information that would support this, if available, is
a white blood cell count (WBC) above 11,000 cells/ul (normal range is 4,500-10,000) and elevated levels
of C-reactive protein.

Ectopic pregnancy is another possibility. The incidence of ectopic pregnancy rises with age in women in
their child-bearing years. While the highest risk group is ages 35-44 years, smokers have been shown to
have 1.6-3.5 times the risk of non-smokers. The location of the ectopic pregnancy can vary, but one
common location is in the fallopian tube. If this occurs in the right tube, the presentation can be very
similar to that of appendicitis. An important component of the etiology of tubal pregnancy is a history of
pelvic inflammatory disease (PID) due to Chlamydia trachomatis which is transmitted sexually. For this
reason, knowing whether or not this woman has had a high number of sexual partners (choice B) can be
useful.

The method of contraception that she uses (choice C) is relevant as well, since certain types of intrauterine
device (IUD) have been linked to ectopic pregnancy in the past, specifically those using copper or
progesterone. While studies as of the year 2010 have shown that only progesterone IUD have any
association with ectopic pregnancy, overall the ectopic pregnancy incidence in IUD users is 3-4 percent. At
the same time, if this woman has used condoms frequently as a form of contraception, the likelihood of
having PID in the past would be reduced, even if she has had a high number of sexual partners.

Related to this is the amount of time that the patient has been married (choice E), because if she's been
married for many years yet nulliparous (has never been pregnant), it could mean that he has a fertility
issue. The reason for this is that ova traveling from the ovaries during ovulation tend to get slowed or
trapped in fallopian tubes that have narrowed due to fibrosis resulting from the past PID. This causes
infertility while at the same time can lead to a tubal pregnancy if a fertilized ovum does is not able to
traverse the tube and thus implants there. Thus, if she has been married for several years, or if she has
been in a relationship for many years, it may be important to know whether her nulliparity is by choice. If
not, then her infertility may be there result of PID, or to another condition such as endometriosis. In any,
case, any woman of child-bearing age presenting with abdominal pain should be given a pregnancy test
and if positive she must be worked up for an ectopic pregnancy.

In endometriosis, endometrium-like tissue is found outside of the uterus. This can cause a chronic
inflammatory response in areas of the pelvis such as on the vagina, cervix, ovaries, fallopian tubes,
uterosacral ligaments and the rectovaginal septum. Typically, endometriosis limited to the pelvis develops
in women ages 25-30, while outside of the pelvis it develops with a still older peak incidence of 35-40
years. It is another cause of infertility, but is a possible diagnosis in this case of abdominal pain, until more
is known.

Although it has other components, such as arthralgia (joint pain), Familial Mediterranean fever (FMF) tends
to manifest with episodes of peritonitis, often mimicking acute appendicitis, sometimes resulting in a
healthy appendix being removed. FMF in an hereditary disease whose incidence varies sharply between
ethnic and national groups (choice A).

An autosomal recessive condition, FMF manifests in people who are homozygous for an MEFV gene, of
which several are known. Some MEFV genes are common in Arab people, occuring with in an estimated
1/50 Arabs, resulting in an FMF incidence of 1/2600 in many Arabic-speaking countries. FMF is even more
common in Turkey with an incidence of 1/1000, partly because the incidence is extremely high among
Armenians, with a gene frequency of 1 in 7 people and a disease incidence of 1/500.

Almost as high is the incidence among Sephardic Jews. Descended from Jews expelled from Spain and
Portugal who migrated to North Africa, but also to Holland and southeast Europe, including Turkey,
Sephardic Jews have an estimated FMF incidence of 1/250-1,000; thus, the gene occurs in 1 out of every
8-16 Sephardic Jews. While Ashkenazic Jews (which make up most Jews in the US and Canada) previously
were thought to have a fairly low incidence of FMF genes, studies have revealed a very high incidence, 1/5,
of a particular FMF gene, but interestingly it has resulted in a disease incidence of only 1/73,000

204
suggesting a low penetrance for this particular gene.

Whether in a clinical setting, or on a standardized exam, it is important not to confuse ethnicity or


nationality with religion (choice D). Frequently on standardized exams, questions dealing with FMF involve
patients who are Arab, which should not be confusing, but also those who are Jewish, Armenian, or in
some cases "Armenian Christian". While Armenians tend to be Christian, a large fraction of Jews actually
do not practice any religion. Thus, asking about this patient's religious practices or beliefs would be
unnecessary.

173.

An ultrasound on a 22 year old G2P1 at 30 weeks of gestation reveals a single fetus with its head in the
right upper quadrant and back to the mothers left. Both fetal thighs are flexed and legs extended.

Which of the following is the correct fetal presentation of this baby:

Frank breech

complete breech

Incomplete breech

Double-footing breech

kneeling breech

Explanation

The correct answer is choice A

Certain conditions are more predisposed to breech presentations (when the baby enters the birth canal
with the buttocks or feet first as opposed to a head presentation) such as:

 multiple gestation
 abnormal amniotic fluid volume (both polyhydramnios and oligohydramnios)
 uterine abnormalities
 fetal anomalies
 prior Cesarean section
 prematurity
 polyhydramnios/Oligohydramnios
 prior Breech Delivery

There are several types of breech presentations depending on whether the feet or buttocks enter first.

Types of breech presentation

205
Frank breech (Choice A) (70% of the breeches)- bottom comes first and legs are flexed at the hip and
extended at the knees (feet are by the ears)

Complete breech (Choice B) (5-10%)- baby's hips and knees are flexed so baby is sitting cross-legged with
feet beside the bottom.

Incomplete breech (Choice C)- One leg is stretched down the birth canal.

Double footling breech (Choice D)- both feet present first. This is rare at term but common in prematurity.

Kneeling breech (Choice E)- baby is in a kneeling position with one or both legs extended at the hips and
flexed at the knees. This is very rare.

174.

A 60 year-old female presents to the urgent care clinic where you are moonlighting complaining of urinary
frequency, urgency, dysuria and low back pain along with nonspecific lower abdominal dyscomfort for the
past 3 days. Laboratory examination confirms urinary tract infection without evidence of renal
involvement. Lengthy discussion with her reveals that this is the third such episode for her in the past 3
years and she is frustrated and irritable: “Why has this started happening to me doctor? I never had these
problems when I was younger like some of my friends did. Is it because I’m growing old?”

Which of the following treatments is the best treatment recommendation for her urinary tract infection
concern?

Antibiotic prophylaxis with trimethoprim/sulfamethoxazole

Kegel exercises

Click on image to Zoom

206
Probiotic administration

Topical vaginal estrogen administration

Increase daily water intake and drink 2 glasses of cranberry


juice daily

Explanation

The correct answer is Choice D

One of the potential complications of menopause is development of urinary tract infections due to change
in the normal bacterial vaginal flora resulting from decreasing local concentration of estrogen. Reduction of
the protective flora allows for proliferation of pathogenic bacteria that can invade the urethra and progress
into the bladder, leading to cystitis. Numerous effective treatment methods for this condition exist,
including both systemic and topical estrogen therapy, but most women find that topical estradiol cream
applied to the urethral orifice on alternating days results in fairly rapid improvement and likely resolution of
this concern. One added benefit of the marked efficacy of topical therapy is that many women are anxious
about using systemic therapy because of possible side effect concerns.

Antibiotic prophylaxis with trimethoprim/sulfamethoxazole (Choice A) is incorrect because the only time
that prophylaxis should be considered is when infection is occurring on a very frequent basis (4-6
episodes/year) and when other possible causes (recurrence due to stone, tumor, or other anatomic issue,
resistant bacteria, etc.) have been eliminated as possible causes. A significant concern with antibiotic
prophylaxis is development of bacterial resistance and elimination of the prophylactic agent from the list of
potential compounds that could be used to treat a breakthrough infection. The usual possible side effects of
the agent employed are possible concerns, as well.

Kegel Exercises (Choice B) is incorrect because this is the proper name of the pelvic floor strengthening
exercises designed to aid men and women with stress urinary incontinence issues and other pelvic floor
instability issues. While they do tend to help reduce incontinence concerns and have been noted by some
to improve sexual satisfaction, their utility in urinary tract infection prevention is not nearly as effective as
that of estrogen administration.

Probiotic administration (Choice C) is not correct because this therapy has not been proven in well
designed prospective studies to reduce the incidence of urinary tract infection.

Increase daily water intake and drink 2 glasses of cranberry juice daily (Choice E) is not the best answer
for this question because both increasing daily water intake and adding cranberry juice to a persons’ diet
have not been demonstrated to have a great effect on either prevention or treatment of urinary tract
infection. The minimal data on cranberry juice suggests that it is the flushing effect caused by increased
liquid moreso than that urine acidification that may be of some help in helping prevent an early urinary
tract infection from progressing to a fully developed episode.

175.

A 25-year old woman in her first trimester presents to her obstetrician for an initial antenatal examination.
Her past history is positive for epilepsy and her seizures are currently controlled with valproic acid.

Which of the following congenital malformations is least apt to occur with first-trimester maternal use of
this drug?

207
Atrial septal defect

Polydactyly

Hypospadias

Ventricular septal defect

Cleft palate

Explanation

The correct answer is choice D

Valproic acid is a chemical compound used as an anticonvulsant and mood-stabilizing drug. It is used mainly in treatment of
epilepsy, bipolar disorder and occasionally for major depression, migraine headaches and schizophrenia. In the United States
and Canada, the Food and Drug Administration (FDA) assigns each medication to a Pregnancy Category according to whether it
has been proven to be harmful in pregnancy. Valproic acid is listed in Pregnancy Category D. This means that there is a risk to
the baby, but the benefits may outweigh the risk for some women.

When taken in pregnancy, valproic acid is associated with:

 neural tube defects (known folate antagonist)


 increased risk of autism
 epicanthic folds
 flat nasal bridge
 cognitive impairment in the newborn
 thick lower lip and small downturned mouth
 atrial septal defect
 cleft palate
 hypospadias
 polydactyly
 craniosynostosis

The risks for other malformations in case-control studies were not elevated. These include:

 clubfoot
 microcephaly
 tetralogy of Fallot
 ventricular septal defect
 gastroschisis
 diaphragmatic hernia

Figure 1: Review of Drug-Pregnancy Categories

208
176.

A 32 year old G2 P1 woman gives birth via normal spontaneous vaginal delivery at 38 weeks gestation.
The infant weighs 3550 grams and scores 9 and 10 at the 1 and 5 minute Apgar tests, respectively. Seven
minutes later, the placenta is delivered and weighs 500 grams. Examination reveals that the umbilical
cord is 100 centimeters long.

Which of the following abnormalities were most likely also observed during delivery?

Amnion nodosum

Nuchal cord

Velamentous insertion

Single umbilical artery

Umbilical cord prolapse

Explanation

The correct answer is choice B.

A nuchal cord occurs when the umbilical cord becomes wrapped around the fetal neck 360 degrees. Nuchal
cords are very common, with prevalence rates of 6% to 37%, though up to half of nuchal cords resolve
before delivery.

Umbilical cords over 69.85 centimeters are considered long. A long umbilical cord doesn't generally

209
indicate any birth defects, but can be a problem if the fetus gets tangled in the cord or manages to tie it
into a tight knot that prevents proper blood and nutrient flow. This kind of fetal entanglement is not
uncommon and can often be treated during birth as long as the medical practitioner is aware of it
beforehand. Some doctors and researchers believe that a long cord is a sign of a hyperactive baby, since
the length of the cord normally increases with increased movement of the fetus in the womb.

Amonion nodosum (choice A) is a rare occurrence that results from prolonged oligohydramnios, which
typically leads to a short umbilical cord.

In velamentous insertion (choice C), the umbilical cord inserts into the membranes, then travels within the
membranes to the placenta (between the amnion and chorion). The insertion point of the cord, however, is
not related to the length of the cord.

In single umbilical artery (choice D), the umbilical cord contains only two blood vessels, instead of the
normal three. The cause of this abnormality, called single umbilical artery, is unknown, but is unrelated to
umbilical cord length.

Umbilical cord prolapse (choice E) occurs when the cord slips into the vagina after the membranes (bag of
waters) have ruptured, before the baby descends into the birth canal. The baby can put pressure on the
cord as he passes through the cervix and vagina during labor and delivery. Pressure on the cord reduces or
cuts off blood flow from the placenta to the baby, decreasing the baby's oxygen supply. Umbilical cord
prolapse can result in stillbirth unless the baby is delivered promptly, usually by cesarean section. There is
no identified relationship between umbilical cord prolapse and a longer umbilical cord.

177.

A 20 year old woman gives a history of sharp pain in the lower abdomen for 2-3 days every month
approximately 2 weeks before the menses. Her menstrual cycles are typically 28 days in length and
regular. She began menstruating at the age of 12 and has normal sexual characteristic development. She
is otherwise healthy and on no medications. She is not currently pregnant but is sexually active.

The most probable etiology for her pain is which of the following:

Endometriosis

Dysmenorrhoea

Pelvic tuberculosis

Mittelschmerz

Pelvic Inflammatory disease

Explanation

The correct answer is choice D

Mittelschmerz (which originates from German meaning middle pain) is characterized by lower abdominal
and pelvic pain that occurs midway through a woman's menstrual cycle during ovulation. Approximately
20% of women experience mittelschmerz and is diagnosed when the pain is mid-cycle with no
abnormalities on pelvic examination. The pain can appear suddenly and usually subsides within hours,
although it may sometimes last two or three days. Because ovulation occurs on a random ovary each
cycle, the pain may switch sides or stay on the same side from one cycle to another. The pain is not
harmful and does not signify the presence of disease. No treatment is usually necessary. Pain relievers
(analgesics) may be needed in cases of prolonged or intense pain. Hormonal forms of contraception can be
taken to prevent ovulation -- and therefore ovulatory pain -- but otherwise there is no known prevention.

210
Although the pain is not harmful, it can be mistaken for appendicitis. If the duration lasts for more than 2-
3 days, a pelvic ultrasound should be performed to rule out any other etiology. Causes of this condition
have been associated with the following:

 Follicular swelling
 Ovarian wall rupture
 Fallopian tube contraction
 Smooth muscle cell contraction
 Irritation

The pain of endometriosis (choice A) can be quite severe and is associated with dysmenorrhea (choice B)
disabling cramps during menses. The pain may progress over time and also link to lower back pains. The
pain would not be for only 2 days and not regularly midcycle. Interesting to note is that the intensity of the
pain is not related to the extent of the endometriosis nor the size and amount of any of the seedlings or
endometriomas.

Pelvic tuberculosis (choice C) which is a chronic infection from the Mycobacterium tuberculosis, is
associated with infertility, pelvic pain, period abnormalities and poor general health. The pain here is not
cyclical.

Pelvic Inflammatory disease (choice E) can be very painful and is not cyclical. However, PID can also be
asymptomatic.

178.

A woman's pelvic mass is evaluated by computed tomography of the abdomen and pelvis. CT scan
reveals a 15-cm mass associated with the right ovary, enlarged pelvic and para-aortic nodes, and several
1-cm-sized hyper-dense liver regions. She also has pleural fluid present in both lungs confirmed with
chest X-ray in left and right decubitus positions.

What is the staging of this neoplasm according to the International Federation of Gynecology and
Obstetrics?

Stage IIa

Stage IIb

Stage IIIa

Stage IV

Staging is not possible with this information.

Explanation

The correct answer is Choice E.

Surgery is necessary for the diagnosis, staging, and treatment of intraabdominal or pelvic gynecological
cancers. Surgical staging includes peritoneal washings for cytology and biopsy of any areas where
metastases are suspected.

In this case, while an ovarian origin of the tumor may be suspected, a precise histological diagnosis of the
tumor, whether primary or metastatic, is required. The results from imaging studies presented in the
question are insufficient for diagnosis of the tumor type or for staging of the tumor.

211
The following is a summary of staging for epithelial cancers of the ovary. It is important to note that in this
case, without a biopsy and tissue disgnosis, it is impossible to say whether the neoplasm is ovarian in
origin and if so, whether it is indeed an epithelial neoplasm.

Stage I

 Stage I: ovarian cancer is limited to the ovaries.


 Stage IA: Tumor limited to one ovary; capsule intact, no tumor on ovarian surface. No malignant
cells in ascites or peritoneal washings.*
 Stage IB: Tumor limited to both ovaries; capsules intact, no tumor on ovarian surface. No
malignant cells in ascites or peritoneal washings.*
 Stage IC: Tumor limited to one or both ovaries with any of the following: capsule ruptured, tumor
on ovarian surface, malignant cells in ascites or peritoneal washings.

* [Note: The term, malignant ascites, is not classified. The presence of ascites does not affect staging
unless malignant cells are present.]

Stage II

 Stage IIA: Extension and/or implants on the uterus and/or fallopian tubes. No malignant cells in
ascites or peritoneal washings
 Stage IIB: Extension to and/or implants on other pelvic tissues. No malignant cells in ascites or
peritoneal washings
 Stage IIC: Pelvic extension and/or implants (stage IIA or stage IIB) with malignant cells in ascites
or peritoneal washings

Different criteria for allotting cases to stage IC and stage IIC have an impact on diagnosis. To assess this
impact, of value would be to know if rupture of the capsule was (1) spontaneous or (2) caused by the
surgeon; and, if the source of malignant cells detected was (1) peritoneal washings or (2) ascites.

Stage III ovarian cancer is tumor involving one or both ovaries with microscopically confirmed peritoneal
implants outside the pelvis. Superficial liver metastasis equals stage III. Tumor is limited to the true pelvis
but with histologically verified malignant extension to small bowel or omentum.

 Stage IIIA: Microscopic peritoneal metastasis beyond pelvis (no macroscopic tumor).
 Stage IIIB: Macroscopic peritoneal metastasis beyond pelvis no more than 2 cm in greatest
dimension.
 Stage IIIC: Peritoneal metastasis beyond pelvis more than 2 cm in greatest dimension and/or
regional lymph node metastasis.

Stage IV ovarian cancer is tumor involving one or both ovaries with distant metastasis. If pleural effusion
is present, positive cytologic test results must exist to designate a case to stage IV. Parenchymal liver
metastasis equals stage IV.

179.

A 14-year-old girl is brought to her primary care physician because of irregular menses. Menarche was at
age 13. She denies sexual activity. Initially, her menstruation was relatively painless, although she has
noted some cramping with her last several periods. Her menses have never been regular, with intervals of
4 to 10 weeks between periods. Bleeding has lasted from 2 to 5 days at a time. She is otherwise healthy
and without other complaints.

Which of the following is the most appropriate next step in this patients management?

212
Begin oral contraceptive pills

Dilatation and curettage

Obtain a pelvic ultrasound

Perform a speculum examination

Reassurance

Explanation

The correct answer is choice E

The patient had been experiencing anovulation. While the normal menstrual cycle in woman typically last
28 days with a follicular phase, ovulation and a luteal phase, the anovulatory cycle has cycle lengths of
varying degrees. The menstrual interval is prolonged and exceeds 35 days with no menstrual pattern.
Bleeding occurs due to the inability of estrogen that is needed to stimulate the endometrium, to maintain
the lining. Anovulatory cycles with just estrogen progresses with increased progesterone manufacture as
puberty is reached and progresses.

This patient has just started to have ovulatory cycles which can present with midcycle cramping or
dysmenorrhea. At this time, an explanation of the menstrual cycle and condition to the patient will be very
helpful. After several cycles, the patient may adjust and the problem resolve spontaneously.

Observation at this point is key as most females will regulate their cycles within 6 months. If a problem
persists, blood tests for prolactin (which can be elevated during stress) or thyroid studies may be initiated
as well as FSH,LH and estrogen levels.

A more complete history also needs to be taken. Cigarette smoking is an agent that can shorten menses
by decreasing the follicular phase. As little as 10 cigarettes a day can shorten menstrual cycles and make
them variable in lengths.

Acute alcohol ingestion or getting drunk on an irregular basis may also cause menses irregularity.

Prescribing oral contraceptives (Choice A) will suppress ovulation and this patient is trying to adjust to
having ovulation become regular. Since the pills are not being given for contraceptive purposes, one
should wait and see what the next few menstrual cycles are like.

Performing a dilatation and curettage (Choice B) is directly contraindicated because you may:

 Create Asherman's syndrome (endometrial adhesions)


 Break the integrity of the hymen
 Cause emotional trauma
 no medical indication for the procedure

A pelvic sonogram (choice C) can be performed in the future if the problem persists but a simple pelvic
examination (by rectal) should be sufficient at this point. Polycystic ovarian syndrome may cause irregular
delayed menses due to a problem with egg release despite follicular growth and estrogen increase. The
high estrogen state may cause endometrial proliferation and once too thick causes sloughing. Increase
hair growth and insulin resistance are often associated with this condition.

A speculum examination (Choice D) would cause trauma to the hymen and would only be performed after
visual inspection of the area to denote its integrity.

180.

213
A 40-year-old woman has had a feeling of abdominal discomfort for the past 8 months. On pelvic
examination, there is a right adnexal mass. An abdominal CT scan demonstrates a 7 cm cystic mass
involving the right ovary with small areas of calcification. The uterus is normal in size. The right fallopian
tube and ovary are removed surgically for histopathological diagnosis. Grossly, the mass on sectioning is
filled with abundant hair and sebum. Microscopically, the mass has glandular spaces lined by columnar
epithelium, squamous epithelium with hair follicles, cartilage, and dense connective tissue.

Which of the following statements regarding this tumor is most appropriate?

A sarcomatous element is usually present

A human papillomavirus infection preceded development of this mass

Metastases are unlikely to be present

Primitive immature tissues resembling those from an embryo can usually be seen

Such a mass is often seen to arise in the testis

Explanation

The correct answer is Choice C.

The patient's tumor is a mature cystic teratoma. Also known as a dermoid cyst, the mature cystic teratoma
is the most common ovarian tumor in women in the second and third decades of life.

The characteristic appearance of dermoid cysts is a multicystic mass that contains hair, teeth, and/or skin
that is mixed into sebaceous, thick, often foul-smelling material. The tissues present are mature,
differentaited tissues rather than embryonal tissue. Malignant transformation occurs in only 0.2 to 2
percent of mature cystic teratomas. Risk factors for malignancy in a mature cystic teratoma include patient
age over 45 years, tumor diameter greater than 10 cm, and rapid growth of the lesion.

Beacuse of their characteristic appearance, ultrasound and other imaging studies can accurately suggest
the diagnosis in most cases. Treatment is by surgical removal. Surgical management is necessary in order
to establish a definitive diagnosis as well as to potentially preserve ovarian tissue and avoid complications
such as torsion, rupture, or development of malignancy.

Human papillomaviruses are not related to the pathogenesis of mature cystic teratoma.

While mature cystic teratomas are common in the ovary, they are rarely found in the testis.

214

Вам также может понравиться